Everything EMT

¡Supera tus tareas y exámenes ahora con Quizwiz!

Blood pressure is usually not assessed in children younger than _____ years. 3 5 4 6

3

__________ rays easily penetrate through the human body and require lead or several inches of concrete to prevent penetration. Neutron Gamma Beta Alpha

Gamma

All of the following conditions should make you suspect shock, EXCEPT: anaphylaxis. severe infection. spinal injury. ischemic stroke.

ischemic stroke.

A productive cough, fever, and chills in an 80-year-old patient with a compromised immune system should make you most suspicious for: emphysema. bronchitis. pneumonia. heart failure.

pneumonia.

The leading cause of death in the geriatric patient is: hypertension. heart disease. altered mental status. arthritis.

heart disease.

Common signs and symptoms of a chest injury include all of the following, except: hematemesis. tachypnea. chest wall ecchymosis. localized pain.

hematemesis.

A sign of kidney damage after blunt trauma is: hematemesis. hematochezia. hematuria. hemoptysis.

hematuria.

After applying a tourniquet, the injury from a patient's leg stops bleeding. This is called: hemiplegia. hemolysis. hematemesis. hemostasis.

hemostasis.

Enlargement of the liver is called: hydrocephalus. pneumonitis. nephritis. hepatomegaly.

hepatomegaly.

As a woman approaches menopause: she usually experiences abdominal cramping without vaginal bleeding. she cannot become pregnant because of fluctuating hormone levels. her menstrual periods may become irregular and vary in severity. her risk of developing pelvic inflammatory disease lowers significantly.

her menstrual periods may become irregular and vary in severity.

A 33-year-old woman who is 36 weeks pregnant is experiencing vaginal bleeding. During transport, you note that she suddenly becomes diaphoretic, tachycardic, and hypotensive. You should: place her in a left lateral recumbent position. position her supine and elevate her legs 12 inches. carefully place sterile gauze into her vagina. ventilate her with a bag-mask device.

place her in a left lateral recumbent position.

A 26-year-old female who is 34 weeks pregnant is experiencing a seizure. Her husband tells you that her blood pressure has been high and that she had been complaining of a headache for the past few days. You should: elevate her legs to improve brain perfusion and keep her warm. administer oral glucose for presumed hypoglycemia and transport. insert an oral airway and ventilate her with a bag-valve mask. place her on her side, administer high-flow oxygen, and transport.

place her on her side, administer high-flow oxygen, and transport.

After recognizing that an incident involves a hazardous material, you should contact the hazardous materials team and then: take measures to ensure the safety of yourself and others. don standard equipment before gaining access to any patients. identify the chemical using the Emergency Response Guidebook. not allow anyone within 25′ to 50′ of the incident scene.

take measures to ensure the safety of yourself and others.

After sizing up the scene of a patient with a possible infectious disease, your next priority should be to: quickly access the patient. take standard precautions. notify law enforcement. contact medical control.

take standard precautions.

Airway management can be challenging in patients with Down syndrome because their: teeth are misaligned, and they have a large tongue. tongue is relatively small and falls back in the throat. occiput is round, which causes flexion of the neck. mandible is large, which inhibits a mask-to-face seal.

teeth are misaligned, and they have a large tongue.

Elevation of the rib cage during inhalation occurs when: abdominal contents descend. intrathoracic pressure decreases. the intercostal muscles contract. the diaphragm descends.

the intercostal muscles contract.

Pleural fluid is contained between the: visceral pleura and the lung. visceral and parietal pleurae. parietal pleura and the chest wall. parietal pleura and the heart.

visceral and parietal pleurae.

Common causes of syncope in older patients include all of the following, except: vasoconstriction. acute hypotension. blood volume loss. venous pooling.

vasoconstriction.

External bleeding from a vein is relatively easy to control because: blood typically oozes from a vein. veins carry deoxygenated blood. veins are under a lower pressure. veins hold smaller blood volume.

veins are under a lower pressure.

A female patient with a suspected head injury has slow, shallow breathing. The most appropriate treatment for her includes: ventilation assistance to maintain an ETCO 2 of 30 to 35 mm Hg. ventilation assistance to maintain an oxygen saturation of 90%. hyperventilating her at 30 breaths/min. administering oxygen via a nonrebreathing mask.

ventilation assistance to maintain an ETCO 2 of 30 to 35 mm Hg.

You may not be able to determine whether a person has a mental illness, but you can predict the person's likelihood of becoming: psychotic. diabetic. suicidal. violent.

violent.

The adult epinephrine auto-injector delivers ______ mg of epinephrine, and the infant-child auto-injector delivers ______ mg. 0.1; 0.01 0.01; 0.1 0.03; 0.3 0.3; 0.15

0.3; 0.15

According to the rule of palm method for estimating the extent of a patient's burns, the palm of the patient's hand is equal to _____ of his or her total body surface area. 6% 1% 4% 2%

1%

hat is the minimum number of chest compressions that should be delivered per minute to a 4-month-old infant? 90 110 100 120

100

Trench collapses usually involve large areas of falling dirt that weigh approximately _______ per cubic foot. 50 lbs 200 lbs 150 lbs 100 lbs

100 lbs

A 5-year-old female pulled a pot of boiling water from the stove. She has superficial and partial-thickness burns to her head, face, and anterior trunk. What percentage of her body surface area has been burned? 18% 36% 27% 30%

30%

In two-rescuer adult CPR, you should deliver a compression to ventilation ratio of: 15:2. 5:1. 30:2. 5:2.

30:2.

After ________ minutes without oxygen, permanent brain damage is possible. 4 to 6 2 to 3 1 to 2 7

4 to 6

At a flow rate of 6 L/min, a nasal cannula can deliver an approximate oxygen concentration of up to: 52%. 35%. 44%. 24%.

44%.

A patient in unstable condition should be reassessed at least every: 10 minutes. 20 minutes. 15 minutes. 5 minutes.

5 minutes.

An oxygen cylinder should be taken out of service and refilled when the pressure inside it is less than: 500 psi. 1,000 psi. 1,500 psi. 200 psi.

500 psi.

The average adult has approximately ___________ of blood in his or her vascular system. 3 L 6 L 4 L 5 L

6 L

A child might begin to show signs of separation anxiety as early as: 6 months. 12 months. 2 months. 18 months.

6 months.

Capillary refill time is most reliable as an indicator of end-organ perfusion in children younger than: 4 years. 6 years. 8 years. 10 years.

6 years.

The cervical spine is composed of _____ vertebrae. 8 5 6 7

7

The average pulse rate of persons between 19 and 40 years of age is typically: 60 beats/min. 80 beats/min. 70 beats/min. 90 beats/min.

70 beats/min.

The pulse rate of a child from ages 6 to 12 years is approximately: 90 to 140 beats/min. 60 to 100 beats/min. 100 to 150 beats/min. 70 to 120 beats/min.

70 to 120 beats/min.

A critical incident stress debriefing should be conducted no longer than ________ hours following the incident. six 72 24 12

72

If a baby is born at 7:52, the second Apgar score should be calculated at: 7:53. 7:57. 7:59. 8:00.

7:57.

The anterior fontanelle fuses together between the ages of: 6 and 8 months. 3 and 4 months. 9 and 18 months. 7 and 14 months.

9 and 18 months.

A person's ability to shiver is lost when his or her body temperature falls below: 90°F (32°C). 92°F (33°C). 95°F (35°C). 94°F (34°C).

90°F (32°C).

In which of the following patients would the head tilt-chin lift maneuver be the MOST appropriate method of opening the airway? A 24-year-old male who is found unconscious at the base of a tree A 45-year-old male who is semiconscious after falling 20 feet A 37-year-old female who is found unconscious in her bed A 50-year-old male who is unconscious following head trauma

A 37-year-old female who is found unconscious in her bed

10. Calming and reassuring an anxious patient can be facilitated by: A) maintaining eye contact with the patient whenever possible. B) using medical terminology to ensure the patient understands. C) positioning yourself at a level that is higher than the patient. D) withholding unpleasant information until arrival at the hospital

A) maintaining eye contact with the patient whenever possible

70. What is the alveolar minute volume of a patient with a tidal volume of 500 mL, a dead space volume of 150 mL, and a respiratory rate of 16 breaths/min? A) 5,600 mL B) 6,000 mL C) 7,400 mL D) 8,000 mL

A) 5,600 mL

86. The thoracic cavity is separated from the abdominal cavity by the: A. diaphragm. B. anterior rib cage. C. intercostal margin. D. costovertebral angle.

A. diaphragm.

89. All of the following are hollow abdominal organs, EXCEPT for the: A. liver. B. bladder. C. ureters. D. stomach.

A. liver.

What type of poisoning will cause burns around the mouth in children? Plant poisoning. Injected poison. Alkaline poison. Inhaled poison.

Alkaline poison.

Common causes of acute psychotic behavior include all of the following, EXCEPT: Alzheimer's disease. schizophrenia. mind-altering substance use. intense stress.

Alzheimer's disease.

Which of the following is not considered a basic technique when controlling bleeding? Apply a cervical collar and place the patient on a long backboard. Apply direct pressure over the wound with a dry, sterile dressing. Pack the wound with a hemostatic impregnated gauze. Apply a tourniquet to an extremity above the level of the bleeding.

Apply a cervical collar and place the patient on a long backboard.

While auscultating an elderly woman's breath sounds, you hear low-pitched "rattling" sounds at the bases of both of her lungs. This finding is MOST consistent with which of the following conditions? Early pulmonary edema Acute asthma attack Aspiration pneumonia Widespread atelectasis

Aspiration pneumonia

Which of the following situations requires you to notify the appropriate authorities? Cardiac arrest Accidental knife wound Attempted suicide Drug overdose

Attempted suicide

154. In infants and children, a capillary refill time (CRT) that is greater than ______ second(s) is a sign of poor peripheral perfusion. A) 1 B) 2 C) 3 D) 4

B) 2

153. What is the MOST common cause of airway obstruction in an unconscious patient? A) Vomitus B) The tongue C) Blood clots D) Aspirated fluid

B) The tongue

94. Bone marrow produces: A. platelets. B. blood cells. C. lymphocytes. D. electrolytes.

B. blood cells.

143. The right coronary artery supplies blood to the: A. left ventricle and inferior wall of the right atrium. B. right ventricle and inferior wall of the left ventricle. C. right atrium and posterior wall of the right ventricle. D. left ventricle and posterior wall of the right ventricle.

B. right ventricle and inferior wall of the left ventricle.

91. Skeletal muscle is also referred to as __________ muscle. A. smooth B. striated C. connective D. involuntary

B. striated

Which of the following MOST accurately describes the cause of an ischemic stroke? Rupture of a cerebral artery Acute atherosclerotic disease Blockage of a cerebral artery Narrowing of a carotid artery

Blockage of a cerebral artery

A 5-year-old child has had severe vomiting and diarrhea for 4 days. Which of the following assessment findings would be the most indicative of decompensated shock? Blood pressure of 70/40 mm Hg Pulse rate greater than 120/min Respiratory rate of 30 breaths/min Capillary refill time of 4 seconds

Blood pressure of 70/40 mm Hg

28. Which of the following is NOT a BLS intervention? A) Abdominal thrusts B) Chest compressions C) Cardiac monitoring D) Automated defibrillation

C) Cardiac monitoring

199. Your partner, a veteran EMT with whom you have worked regularly for the past 4 years, seems unusually agitated during a call involving an elderly patient. Upon arrival back at your station, you note the obvious smell of alcohol on his breath. What should you do? A) Remain quiet and simply request another partner. B) Report the incident to your EMS medical director. C) Discreetly report your suspicions to your supervisor. D) Tell your partner that he must seek professional help.

C) Discreetly report your suspicions to your supervisor.

13. Which of the following conditions would be LEAST likely to cause an altered level of consciousness? A. drug overdose B. inadequate perfusion C. acute anxiety D. poisoning

C. acute anxiety

Which of the following incidents does not require a report to be filed with local authorities? Animal bites Cardiac arrest Spousal abuse Gunshot wounds

Cardiac arrest

A 33-year-old male sustained an abdominal evisceration to the left lower quadrant of his abdomen after he was cut with a large knife. After appropriately managing his ABCs and assessing him for other life-threatening injuries, how should you care for his wound? Cover it with moist, sterile gauze and secure with an occlusive dressing. Carefully replace the exposed bowel into the abdomen and transport. Cover the exposed bowel and keep his legs in a straight position. Irrigate it with sterile water and cover it with a dry dressing.

Cover it with moist, sterile gauze and secure with an occlusive dressing.

Which of the following conditions is more common in women than in men? Pancreatitis Cystitis Hepatitis Cholecystitis

Cystitis

118. A 60-year-old man complains of chest pain. He is conscious and alert and denies shortness of breath. Which of the following questions would be the MOST appropriate to ask him? A) "Were you exerting yourself when the chest pain began?" B) "Does the pain in your chest move to either of your arms?" C) "Does the pain in your chest feel like a stabbing sensation?" D) "Do you have any heart problems or take any medications?"

D) "Do you have any heart problems or take any medications?"

197. Which of the following statements regarding the EMS medical director and an EMT's scope of practice is correct? A) The EMS medical director can expand the EMT's scope of practice but cannot limit it without state approval. B) The EMS medical director can expand or limit an individual EMT's scope of practice without state approval. C) An EMT's scope of practice is exclusively regulated by the state EMS office, not the EMS medical director. D) An EMT's scope of practice may be expanded by the medical director after proper training and state approval.

D) An EMT's scope of practice may be expanded by the medical director after proper training and state approval.

81. Which of the following steps is NOT proper procedure when performing an emergency move? A) Using a long-axis body drag during the move B) Pulling the patient on a blanket or similar object C) Pulling the patient's clothing in the shoulder area D) Lifting the patient by the belt to move him or her

D) Lifting the patient by the belt to move him or her

190. Which of the following statements regarding communications with the elderly is correct? A) The majority of elderly patients are hearing or visually impaired. B) Attempt to calm the elderly patient by using his or her first name. C) Explain the justification for a procedure after it has been completed. D) Older patients have difficulty understanding when they are stressed

D) Older patients have difficulty understanding when they are stressed.

151. How does respiration differ from ventilation? A) Ventilation occurs when air is forced into the lungs, whereas respiration occurs when air is drawn or sucked into the lungs. B) Respiration occurs when the diaphragm and intercostal muscles contract, whereas ventilation occurs when those same muscles relax. C) Respiration occurs when oxygen is delivered to the cells of the body, whereas ventilation occurs when carbon dioxide is removed. D) Respiration is the process of gas exchange, whereas ventilation is the movement of air between the lungs and the environment.

D) Respiration is the process of gas exchange, whereas ventilation is the movement of air between the lungs and the environment.

66. When a person is exposed to a cold environment: A. sweat is produced and is warmed when the vessels constrict. B. blood vessels dilate and divert blood to the core of the body. C. the skin becomes flushed secondary to peripheral vasodilation. D. peripheral vessels constrict and divert blood away from the skin.

D. peripheral vessels constrict and divert blood away from the skin.

16. A patient who does not respond to your questions but moves or cries out when his or her trapezius muscle is pinched, is said to be: A. conscious and alert. B. completely unresponsive. C. responsive to verbal stimuli. D. responsive to painful stimuli.

D. responsive to painful stimuli.

A 40-year-old man is in cardiac arrest. Your partner is performing CPR. You are attaching the AED when the patient's wife tells you that he has an automatic implantable cardiac defibrillator (AICD). The AED advises that a shock is indicated. What should you do? Continue CPR and transport the patient to the closest appropriate hospital. Deliver the shock followed by immediate resumption of CPR. Avoid defibrillation because this will damage the patient's AICD. Contact medical control and request permission to defibrillate.

Deliver the shock followed by immediate resumption of CPR.

Which of the following is an effective stress management strategy? Developing a social network outside of EMS Frequently reflecting on troublesome calls Requesting overtime to occupy your mind Modestly increasing caffeine consumption

Developing a social network outside of EMS

Following a stab wound to the left anterior chest, a 25-year-old male presents with a decreased level of consciousness and signs of shock. Which of the following additional assessment findings should increase your index of suspicion for a cardiac tamponade? Widening pulse pressure A rapid, irregular pulse Engorged jugular veins Diminished breath sounds

Engorged jugular veins

Which type of consent is involved when a 39-year-old, mentally competent female with a severe headache asks you to take her to the hospital? Formal Implied Informed Expressed

Expressed

A 66-year-old male presents with dark red rectal bleeding and abdominal pain. He is conscious and alert; however, his skin is cool and clammy and his pulse is rapid. Further assessment reveals that his blood pressure is 112/60 mm Hg. Which of the following questions would be most pertinent to ask him? Have you experienced recent abdominal trauma? Do you take any over-the-counter medications? What does your blood pressure normally run? Has blood soaked through your undergarments?

Have you experienced recent abdominal trauma?

Which of the following types of consent allows treatment of a patient who is unconscious or mentally incapacitated? Actual Expressed Implied Informed

Implied

Which of the following is the MOST rapidly acting medication administration route? Subcutaneous (SC) Intramuscular (IM) Intravenous (IV) Sublingual (SL)

Intravenous (IV)

Which of the following would the paramedic be LEAST likely to ask the EMT to do? Intubate a patient Assess blood glucose Apply a tourniquet Obtain vital signs

Intubate a patient

Your patient's symptoms include high fever, cough, vomiting, bloody diarrhea, and shortness of breath. Which of the following should you suspect? Influenza Whooping cough Hepatitis B MERS-CoV

MERS-CoV

A 29-year-old pregnant woman has had severe vomiting for the last 2 days. Today, she is vomiting large amounts of blood. Her skin is cool and pale, and she is tachycardic. The EMT should suspect: esophageal varices. acute pancreatitis. Mallory-Weiss tear. esophagitis.

Mallory-Weiss tear.

Which step in EMS decision making comes after a patient has been transferred? Team communication Data gathering Outcome evaluation Planning

Outcome evaluation

A young male jumped from a tree and landed feet first. Which aspect of his body has sustained the initial injury? Plantar Ventral Dorsal Palmar

Plantar

Which of the following anatomic terms is synonymous with the word "dorsal"? Anterior Medial Palmar Posterior

Posterior

A 25-year-old unrestrained female struck the steering wheel with her chest when her car hit a tree while traveling at a high rate of speed. She has signs and symptoms of shock, which you suspect are the result of intrathoracic bleeding. Which of the following interventions will provide this patient with the greatest chance for survival? Rapid transport to a trauma center High-flow oxygen administration Intravenous fluid administration Full immobilization of her spine

Rapid transport to a trauma center

Which of the following drugs is commonly referred to as "roofies"? GHB MDMA Rohypnol Ketamine

Rohypnol

Which of the following is an example of a functional behavioral disorder? Alzheimer's disease Schizophrenia Head trauma Drug addiction

Schizophrenia

Which of the following conditions would be LEAST likely to result in hypoxia? Pulmonary edema Severe anxiety Narcotic overdose Pleural effusion

Severe anxiety

Which of the following statements regarding Good Samaritan laws is correct? Such laws do not protect EMTs who are off duty. Such laws will not protect the EMT in cases of gross negligence. Such laws guarantee that the EMT will not be held liable if he or she is sued. Such laws provide the EMT with absolute immunity from a lawsuit.

Such laws will not protect the EMT in cases of gross negligence.

A 69-year-old female was involved in a motor vehicle crash. She is semiconscious with a blood pressure of 80/50 mm Hg and a heart rate of 74 beats/min that is weak. Her daughter, who was uninjured in the crash, tells you that her mother has a history of hypertension and takes beta-blockers. Because this patient is probably in shock, what is the most likely explanation for the absence of tachycardia? Failure of the parasympathetic nervous system Deterioration of the cardiac conduction system The effects of her antihypertensive medication Intrathoracic bleeding and cardiac compression

The effects of her antihypertensive medication

Which of the following would MOST likely provide clues regarding the source of a patient's allergic reaction? The patient's family history The patient's general physical appearance The environment in which the patient is found The time of year in which the exposure occurred

The environment in which the patient is found

Which of the following medication routes delivers a drug through the skin over an extended period of time, such as a nitroglycerin or fentanyl patch? Intraosseous Transcutaneous Sublingual Subcutaneous

Transcutaneous

From what internal female organ is the fetus expelled during delivery? Vagina Uterus Cervix Perineum

Uterus

Abnormal variations in pupil size and reaction would most likely be observed in a patient with: retinitis. contact lenses. conjunctivitis. a brain injury.

a brain injury.

The term "supraventricular tachycardia" means: a rapid heart rate that originates from above the ventricles. a rapid heart rate that originates from within the ventricles. a slow heart rate that originates from within the ventricles. a slow heart rate that originates from above the ventricles.

a rapid heart rate that originates from above the ventricles.

Common signs and symptoms of a serious head injury include all of the following, except: a rapid, thready pulse. decerebrate posturing. widening pulse pressure. cerebrospinal fluid leakage from the ears.

a rapid, thready pulse.

The most prominent symptom of decompression sickness is: abdominal or joint pain. difficulty with vision. dizziness and nausea. tightness in the chest.

abdominal or joint pain.

A 37-year-old female with a history of diabetes presents with excessive urination and weakness of 2 days' duration. Her blood glucose level reads 320 mg/dL. If this patient's condition is not promptly treated, she will MOST likely develop: acidosis and dehydration. hypoxia and overhydration. irreversible renal failure. severe insulin shock.

acidosis and dehydration.

A 35-year-old obese woman is complaining of localized pain in the right upper quadrant with referred pain to the right shoulder. The MOST likely cause of her pain is: appendicitis. pancreatitis. acute cystitis. acute cholecystitis.

acute cholecystitis.

A 56-year-old male is found semiconscious by his wife. Your assessment reveals that his respirations are rapid and shallow, his pulse is rapid and irregular, and his blood pressure is low. The patient's wife states that he complained of left arm pain and nausea the day before, but would not allow her to call 9-1-1. The MOST likely cause of this patient's present condition is: acute myocardial infarction. severe septic hypoperfusion. dehydration from GI virus. a ruptured aortic aneurysm.

acute myocardial infarction.

A 59-year-old male presents with sudden-onset severe lower back pain. He is conscious and alert, but very restless and diaphoretic. Your assessment reveals a pulsating mass to the left of his umbilicus. You should: place the patient in a sitting position and transport at once. vigorously palpate the abdomen to establish pain severity. administer oxygen and prepare for immediate transport. request a paramedic unit to give the patient pain medication.

administer oxygen and prepare for immediate transport.

After taking diphenhydramine (Benadryl) for an allergic reaction, a person begins experiencing drowsiness and a dry mouth. These findings are an example of a(n): therapeutic effect. unpredictable effect. adverse effect. untoward effect.

adverse effect.

In the presence of oxygen, the cells convert glucose into energy through a process called: respiration. aerobic metabolism. perfusion. anaerobic metabolism.

aerobic metabolism.

Vector-borne transmission of an infectious organism occurs via: smoke or dust. direct contact. animals or insects. inanimate objects.

animals or insects.

As an EMT, the performance of your duties will be compared to that of: the general public. the medical director. a paramedic supervisor. another EMT.

another EMT.

A disease vector is defined as: the ability of a virus or bacterium to be spread. the period of time between exposure and illness. any agent that acts as a carrier or transporter. the spectrum of signs that define a disease.

any agent that acts as a carrier or transporter.

A 60-year-old female presents with a tearing sensation in her lower back. Her skin is sweaty, and she is tachycardic. The EMT should suspect: kidney stones. aortic aneurysm. strangulated hernia. acute pancreatitis.

aortic aneurysm.

A 17-year-old male was shot in the right anterior chest during an altercation. As your partner is applying oxygen, you perform a rapid assessment and find an open chest wound with a small amount of blood bubbling from it. You should: control the bleeding from the wound and prepare to transport at once. place a sterile dressing over the wound and apply direct pressure. direct your partner to assist the patient's ventilations with a bag-mask device. apply an occlusive dressing to the wound and continue your assessment.

apply an occlusive dressing to the wound and continue your assessment.

A 39-year-old male sustained a stab wound to the groin during an altercation at a bar. As you approach the patient, you note that he is conscious, is screaming in pain, and is attempting to control the bleeding, which is bright red and spurting from his groin area. You should: elevate his legs and keep him warm. ensure that his airway is patent. administer 100% supplemental oxygen. apply direct pressure to the wound.

apply direct pressure to the wound.

Febrile seizures: often result in permanent brain damage. are usually benign but should be evaluated. occur when a child's fever rises slowly. are also referred to as absence seizures.

are usually benign but should be evaluated.

A 30-year-old male was rescued after being lost in the woods for approximately 18 hours. The outside temperature is 30°F (-1°C). He is immediately placed in the warmed ambulance, where you perform a primary assessment. He is unresponsive, pale, and apneic. You should: apply an automated external defibrillator and assess his cardiac rhythm. open his airway and give two rescue breaths. assess for a carotid pulse for up to 60 seconds. apply chemical heat packs to his groin and axillae.

assess for a carotid pulse for up to 60 seconds.

According to the JumpSTART triage system, if a pediatric patient has a respiratory rate of 40 breaths/min, you should: assign an immediate category. assess neurologic status. assess for a distal pulse. look for posturing.

assess for a distal pulse.

After establishing that an adult patient is unresponsive, you should: apply the AED and deliver a shock, if needed. manually open the airway. immediately begin chest compressions. assess for breathing and a pulse.

assess for breathing and a pulse.

A 31-year-old male with a history of diabetes had a seizure that stopped prior to EMS arrival. He is unresponsive and has rapid, shallow breathing. His pulse is rapid and weak, and his skin is cyanotic. The EMT should: place a thin layer of oral glucose between his cheek and gums. administer oxygen by nasal cannula and assess his blood glucose level. position the patient on his side and give oxygen via nonrebreathing mask. assist the patient's ventilations with a bag-valve mask.

assist the patient's ventilations with a bag-valve mask.

A young male was shot in the abdomen by an unknown type of gun. Law enforcement personnel have ensured that the scene is safe. The patient is semiconscious, has shallow breathing, and is bleeding externally from the wound. As you control the external bleeding, your partner should: obtain baseline vital signs. apply a nonrebreathing mask. perform a secondary assessment. assist the patient's ventilations.

assist the patient's ventilations.

If suctioning of the tracheostomy tube is necessary, the EMT should: attempt to use the patient's suction device first because it is probably already sized correctly. instill 20 mL of saline into the tracheostomy tube and suction for no longer than 20 seconds. insert the suction catheter to a depth of no more than 15 cm and set the suction unit to 140 mm Hg. attach a bag-valve mask to the tracheostomy tube and hyperventilate the patient for 2 minutes.

attempt to use the patient's suction device first because it is probably already sized correctly.

When documenting a call involving a female patient who was sexually assaulted, the EMT should: avoid speculation and document only factual data. theorize as to why the sexual assault occurred. include the results of the internal vaginal exam. include a description of the suspected perpetrator.

avoid speculation and document only factual data.

An EMT might injure his or her back, even if it is straight, if the: shoulder is aligned over the pelvis. back is bent forward at the hips. hands are held close to the legs. force is exerted straight down the spine.

back is bent forward at the hips.

A 47-year-old male presents with severe abdominal pain of 3 hours' duration. His abdomen is distended and guarded. Your MOST important consideration for this patient should be to: transport him in a supine position. assess his blood pressure to determine perfusion adequacy. be alert for signs and symptoms of shock. determine the exact location and cause of his pain.

be alert for signs and symptoms of shock.

You and your partner arrive at the side of a 60-year-old woman who collapsed about 7 minutes ago. She is unresponsive, apneic, and pulseless. You should: apply the AED if there is no response after 10 cycles of CPR. immediately apply the AED and analyze her cardiac rhythm. begin CPR and apply the AED as soon as it is available. begin CPR at a compression to ventilation ratio of 15:2.

begin CPR and apply the AED as soon as it is available.

Cerebral palsy is a condition that results from damage or injury to the: brain. peripheral nervous system. voluntary muscles. spinal cord.

brain.

The onset of menstruation usually occurs in females who are: between 8 and 10 years of age. between 25 and 28 years of age. between 11 and 16 years of age. between 18 and 23 years of age.

between 11 and 16 years of age.

A patient has fractured both femurs. Anatomically, these injuries would be described as being: proximal. medial. unilateral. bilateral.

bilateral.

A 40-year-old male intentionally cut his wrist out of anger after losing his job. Law enforcement has secured the scene prior to your arrival. As you enter the residence and visualize the patient, you can see that he has a towel around his wrist and a moderate amount of blood has soaked through it. You should: calmly identify yourself to the patient. tell the patient that you want to help. approach the patient with caution. quickly tend to the bleeding wound.

calmly identify yourself to the patient.

Clouding of the lenses of the eyes is called: glaucoma. retinitis. cataracts. conjunctivitis.

cataracts.

In addition to looking for severe bleeding, assessment of circulation in the conscious patient should involve: taking a blood pressure and determining if the patient is alert and oriented or confused. applying a pulse oximeter probe to the finger to determine if peripheral perfusion is adequate. palpating the carotid pulse to determine the approximate rate and checking capillary refill time. checking the radial pulse and noting the color, temperature, and condition of the skin.

checking the radial pulse and noting the color, temperature, and condition of the skin.

Exposure to _________ would most likely result in immediate respiratory distress. soman tabun lewisite chlorine

chlorine

The three bones that make up the shoulder girdle are the: acromion, humerus, and clavicle. acromion, clavicle, and scapula. acromion, scapula, and humerus. clavicle, scapula, and humerus.

clavicle, scapula, and humerus.

A 30-year-old female presents with redness, inflammation, and pain to her left eye. During your assessment, you note that she is having difficulty keeping her eyes open. You should suspect that she is experiencing: acute retinitis. conjunctivitis. a detached retina. a corneal abrasion.

conjunctivitis.

General guidelines for carrying a patient on a stretcher include: slightly twisting your body when carrying. constant communication with your partners. maintaining slight flexion of your back. leaning back from your waist when lifting.

constant communication with your partners.

A 39-year-old male accidentally cut his wrist while sharpening his hunting knife. He is conscious and alert with adequate breathing but is bleeding significantly from the wound. You should: apply oxygen with a nonrebreathing mask. control the bleeding with direct pressure. ensure the patient has a patent airway. apply a tourniquet proximal to the wound.

control the bleeding with direct pressure.

Phosgene (CG) has an odor that resembles: almonds. cut grass. bleach. garlic.

cut grass.

The process of removing or neutralizing and properly disposing of a hazardous material is called: decontamination. chemical containment. antidotal treatment. neutralization.

decontamination.

During your assessment of a 70-year-old woman, she tells you that she takes blood-thinning medication and has to wear compression stockings around her legs. This information should make you suspect that she has: hemophilia sickle-cell disease severe hyperglycemia. deep vein thrombosis.

deep vein thrombosis.

The effectiveness of pit crew CPR is dependent on: a team leader who is capable of performing all of the patient care tasks. defining clear roles and responsibilities before the call is received. rapidly assessing the patient before assigning roles and responsibilities. protocols that allow the EMT to function without medical control.

defining clear roles and responsibilities before the call is received.

Common signs and symptoms of a sprain include all of the following, except: ecchymosis. guarding. deformity. swelling.

deformity.

A patient with profuse sweating is referred to as being: edematous. plethoric. diaphoretic. flushed.

diaphoretic.

Nitroglycerin relieves cardiac-related chest pain by: constricting the coronary arteries and improving cardiac blood flow. contracting the smooth muscle of the coronary and cerebral arteries. dilating the coronary arteries and improving cardiac blood flow. increasing the amount of stress that is placed on the myocardium.

dilating the coronary arteries and improving cardiac blood flow.

Bleeding from soft-tissue injuries to the face is most effectively controlled with: pressure dressings and chemical ice packs. digital pressure to an adjacent pulse point. direct pressure using dry, sterile dressings. ice packs and elevation of the patient's head.

direct pressure using dry, sterile dressings.

A fracture of the humerus just above the elbow would be described as a: proximal humerus fracture. distal forearm fracture. distal humerus fracture. proximal elbow fracture.

distal humerus fracture.

When working as an independent health care group member, the EMT should expect that he or she: does not have to wait for an assignment before performing a task. will receive no support or guidance from an EMS supervisor. will be specifically instructed on how to perform a specific task. will rely on the group leader for making virtually all decisions.

does not have to wait for an assignment before performing a task.

Common side effects of epinephrine include all of the following, EXCEPT: drowsiness. tachycardia. headache. dizziness.

drowsiness.

Patients with uncontrolled diabetes experience polyuria because: excess glucose in the blood is excreted by the kidneys. high blood sugar levels cause permanent kidney damage. low blood glucose levels result in cellular dehydration. they drink excess amounts of water due to dehydration.

excess glucose in the blood is excreted by the kidneys.

Geriatric patients, newborns, and infants are especially prone to hyperthermia because they: have less body fat. exhibit poor thermoregulation. have relatively smaller heads. have smaller body surface areas.

exhibit poor thermoregulation.

Posttraumatic stress disorder can happen after: alcohol withdrawal. a bout with depression. exposure to or injury from a traumatic occurrence. extended periods of hyperthermia or hypothermia.

exposure to or injury from a traumatic occurrence.

Most cases of anthrax begin with: respiratory distress. signs of shock. flulike symptoms. pulmonary edema.

flulike symptoms.

When a woman presents with abdominal pain or other vague symptoms, the EMT is often unable to determine the nature of the problem until he or she: has formed a general impression of the patient. ascertains if the patient was ever pregnant. has obtained a complete set of vital signs. has gathered patient history information.

has gathered patient history information.

Common safety equipment carried on the ambulance includes all of the following, except: turnout gear. hazardous materials gear. safety goggles. face shields.

hazardous materials gear.

Shock is the result of: hypoperfusion to the cells of the body. the body's maintenance of homeostasis. temporary dysfunction of a major organ. widespread constriction of the blood vessels.

hypoperfusion to the cells of the body.

After performing a primary assessment, a rapid exam of the body should be performed to: look specifically for signs and symptoms of inadequate perfusion. determine the need for spinal motion restriction precautions. find and treat injuries or conditions that do not pose a threat to life. identify less-obvious injuries that require immediate treatment.

identify less-obvious injuries that require immediate treatment.

Gastric distention will MOST likely occur: when you deliver minimal tidal volume. in patients who are intubated. when the airway is completely obstructed. if you ventilate a patient too quickly.

if you ventilate a patient too quickly.

In late adults, the amount of air left in the lungs after expiration of the maximum amount of air: decreases, which increases diffusion in the lungs and causes an accumulation of carbon dioxide. remains unchanged because the lungs have become accustomed to years of breathing pollution. decreases, resulting in widespread collapsing of the alveoli and impaired diffusion of gases. increases, which hampers diffusion of gases because of the stagnant air that remains in the alveoli.

increases, which hampers diffusion of gases because of the stagnant air that remains in the alveoli.

All of the following will help minimize the risk of gastric distention when ventilating an apneic patient with a bag-mask device, EXCEPT: ensuring the appropriate airway position. increasing the amount of delivered tidal volume. delivering each breath over 1 second. ventilating the patient at the appropriate rate.

increasing the amount of delivered tidal volume.

You are assisting an asthma patient with his prescribed metered-dose inhaler. After the patient exhales, and before inhaling, the patient should put his or her lips around the inhaler, take a deep breath, and depress the inhale. You should: advise him to exhale forcefully to ensure medication absorption. allow him to breathe room air and assess his oxygen saturation. immediately reapply the oxygen mask and reassess his condition. instruct him to hold his breath for as long as he comfortably can.

instruct him to hold his breath for as long as he comfortably can.

Once a cervical collar has been applied to a patient with a possible spinal injury, it should not be removed unless: sensory and motor functions remain intact. lateral immobilization has been applied. the patient adamantly denies neck pain. it causes a problem managing the airway.

it causes a problem managing the airway.

You respond to the residence of a 70-year-old male who complains of weakness and severe shortness of breath. His wife tells you that he is a dialysis patient, but has missed his last two treatments. After applying oxygen, you auscultate his lungs and hear diffuse rhonchi. The patient is conscious, but appears confused. His blood pressure is 98/54 mm Hg, his pulse rate is 120 beats/min and irregular, and his respirations are 24 breaths/min and labored. You should: leave him in a sitting position, keep him warm, and prepare for immediate transport. place him in a supine position, elevate his lower extremities, and transport at once. perform a detailed secondary assessment and then transport him to a dialysis center. treat for shock and request a paramedic unit to respond to the scene and assist you.

leave him in a sitting position, keep him warm, and prepare for immediate transport.

When caring for a woman who is experiencing a gynecologic emergency, the EMT's main focus should be to: ask questions related to her gynecologic history. determine the underlying cause of her problem. maintain her ABCs and transport without delay. keep assessment and treatment to a minimum.

maintain her ABCs and transport without delay.

A 50-year-old female is entrapped in her passenger car after it struck a tree. As the rescue team is preparing to extricate her, you quickly assess her and determine that she is breathing shallowly and her radial pulse is absent. You should: stabilize her condition before extrication begins. begin CPR as the rescue team begins extrication. secure her with a short backboard or vest device. maintain spinal stabilization as she is extricated.

maintain spinal stabilization as she is extricated.

As you assess the head of a patient with a suspected spinal injury, your partner should: prepare the immobilization equipment. assess the rest of the body for bleeding. maintain stabilization of the head. look in the ears for gross bleeding.

maintain stabilization of the head.

You and your EMT partner arrive at the residence of a 50-year-old man who complains of weakness. Your primary assessment reveals that he is critically ill and will require aggressive treatment. The closest hospital is 25 miles away. You should: manage all threats to airway, breathing, and circulation and consider requesting an ALS unit. administer oxygen via nonrebreathing mask and obtain as much of his medical history as possible. perform a detailed secondary assessment, assess his vital signs, and then transport rapidly. load him into the ambulance, begin transport, and perform all treatment en route to the hospital.

manage all threats to airway, breathing, and circulation and consider requesting an ALS unit.

A 4-year-old, 16-kg female ingested an unknown quantity of liquid drain cleaner. Your assessment reveals that she is conscious and alert, is breathing adequately, and has skin burns around her mouth. You should: place her supine and elevate her legs. administer 16 g of activated charcoal. monitor her airway and give oxygen. determine why the ingestion occurred.

monitor her airway and give oxygen.

A rapid, irregular pulse following blunt trauma to the chest is most suggestive of a: pericardial tamponade. tension pneumothorax. myocardial contusion. ruptured aorta.

myocardial contusion.

Anterior to the knee is a specialized bone called the: patella. femur. tibia. calcaneus.

patella.

Assessment of a patient's blood pressure with an automatic BP cuff reveals that it is 204/120 mm Hg. The patient is conscious and alert and denies any symptoms. The EMT should: prepare for immediate transport. obtain a manual blood pressure. conclude that she has hypertension. reassess her blood pressure in 5 minutes.

obtain a manual blood pressure.

An abdominal evisceration: is most commonly the result of blunt force trauma. occurs when organs protrude through an open wound. should be covered with bulky dry, sterile dressings. often causes severe hypothermia because of heat loss.

occurs when organs protrude through an open wound.

The capacity of an individual to cope with and recover from stress is called: wellness. distress. eustress. resilience.

resilience.

Fractures of the pelvis in older patients often occur as the result of a combination of: osteoporosis and low-energy trauma. increased bone density and car crashes. acetabular separation and severe falls. arthritic joints and high-energy trauma.

osteoporosis and low-energy trauma.

Potentially life-threatening consequences of pelvic inflammatory disease (PID) include: bacterial vaginosis and chlamydia. ovarian abscess and ectopic pregnancy. uterine rupture with severe bleeding. ovarian cysts and gonorrhea.

ovarian abscess and ectopic pregnancy.

EMTs arrive at the scene of a patient who was "found down." A family member states that she gave naloxone (Narcan) to the patient before the ambulance arrived. The EMTs should suspect that the patient: overdosed on an opioid drug. experienced a prolonged seizure. has a history of allergic reactions. has a low blood glucose level.

overdosed on an opioid drug.

For every emergency request, the dispatcher should routinely gather and record all of the following information, except the: caller's phone number. patient's medical history. nature of the call. location of the patient(s).

patient's medical history.

A 77-year-old woman slipped and fell on a throw rug and landed on her left hip. She denies striking her head or losing consciousness. Assessment of her left leg reveals that it is shortened and externally rotated. Distal pulses, sensory, and motor functions are intact. You should: place her onto a scoop stretcher, pad around her left hip with pillows, and secure her to the scoop with straps. bind both of her legs together with triangular bandages and carefully secure her onto the ambulance stretcher. manually stabilize her left leg, apply a traction splint, and then secure her to a long backboard or scoop. carefully slide a long backboard underneath her, keep her in a supine position, and apply a splint to her leg.

place her onto a scoop stretcher, pad around her left hip with pillows, and secure her to the scoop with straps.

Hypovolemic shock caused by severe burns is the result of a loss of: plasma. red blood cells. platelets. whole blood.

plasma.

A 22-year-old male was kicked in the abdomen multiple times during an assault. He is conscious but restless and his pulse is rapid. His skin is cold and moist. Your assessment reveals a large area of bruising to the right upper quadrant of his abdomen. The most appropriate treatment for this patient includes: applying oxygen via nasal cannula. preparing for immediate transport. performing a focused physical exam. a detailed assessment of his abdomen.

preparing for immediate transport.

The purpose of a ventricular peritoneum shunt is to: prevent excess cerebrospinal fluid from accumulating in the brain. divert excess cerebrospinal fluid to the ventricles of the brain. monitor pressure within the skull in patients with a head injury. remove fluid from the abdomen of patients with right-sided heart failure.

prevent excess cerebrospinal fluid from accumulating in the brain.

Following blunt trauma to the chest, an 18-year-old female presents with respiratory distress, shallow breathing, and cyanosis. Her blood pressure is 80/50 mm Hg and her pulse is 130 beats/min and thready. You should: perform a rapid head-to-toe physical assessment. provide ventilation assistance with a bag valve mask. apply 100% oxygen via a nonrebreathing mask. place her supine and elevate her lower extremities.

provide ventilation assistance with a bag valve mask.

A 29-year-old female presents with confusion and disorientation. Her respirations are rapid and shallow, and her pulse is 120 beats/min and thready. She is markedly diaphoretic and has an oxygen saturation of 89%. You should: treat her for hyperglycemia. provide ventilatory support. transport immediately. administer oral glucose.

provide ventilatory support.

In relation to the wrist, the elbow is: lateral. medial. distal. proximal.

proximal.

Temporary, widespread vasodilation and syncope caused by a sudden nervous system reaction MOST accurately describes: neurologic shock. neurogenic shock. psychogenic shock. vasovagal shock.

psychogenic shock.

After assisting your patient with prescribed nitroglycerin, you should: perform a secondary assessment before administering further doses. reassess his or her blood pressure within 5 minutes to detect hypotension. avoid further dosing if the patient complains of a severe headache. place the patient in a recumbent position in case of fainting.

reassess his or her blood pressure within 5 minutes to detect hypotension.

A 31-year-old female is experiencing an acute asthma attack. She is conscious and alert, but in obvious respiratory distress. After assisting her with her prescribed MDI, you should: check the drug's expiration date to ensure that it is still current. reassess the patient and document her response to the medication. contact medical control and apprise him or her of what you did. administer another treatment in 30 seconds if she is still in distress.

reassess the patient and document her response to the medication.

After applying medical restraints to a combative patient, you should: position the patient prone in order to further prevent injury to yourself. inform the patient that the restraints are punishment for their behavior. remove them only after hospital personnel have requested you to do so. remove them only if the patient verbally commits to calming down.

remove them only after hospital personnel have requested you to do so.

Blood levels of medications might rise in the elderly, sometimes to toxic levels. This is most likely due to: renal insufficiency. intentional overdose. splenic dysfunction. pancreatic failure.

renal insufficiency.

Most AEDs are set up to adjust the voltage based on the impedance, which is the: distance between the two AED pads on the chest. resistance of the body to the flow of electricity. actual amount of energy that the AED will deliver. direction that the electrical flow takes in the body.

resistance of the body to the flow of electricity.

In most cases, cardiopulmonary arrest in infants and children is caused by: respiratory arrest. severe chest trauma. a drug overdose. a cardiac dysrhythmia.

respiratory arrest.

As you are wheeling your patient through the emergency department doors, you receive another call for a major motor vehicle crash. You should: place the patient in a high-visibility area and then respond to the call. inform the admissions clerk of the situation and then respond at once. respond only after giving a verbal patient report to a nurse or physician. leave a copy of the run form with a nurse and then respond to the call.

respond only after giving a verbal patient report to a nurse or physician.

Eclampsia is most accurately defined as: high levels of protein in the patient's urine. hypertension in the 20th week of pregnancy. seizures that result from severe hypertension. a blood pressure greater than 140/90 mm Hg.

seizures that result from severe hypertension.

Clinical signs of labored breathing include all of the following, EXCEPT: shallow chest movement. supraclavicular retractions. gasping attempts to breathe. use of accessory muscles.

shallow chest movement.

When the EMT assists a paramedic with an advanced intervention, he or she should recall that the focus of the intervention is on: following local protocol. completing the procedure. learning to perform the skill. solving a clinical problem.

solving a clinical problem.

Activities such as walking, talking, and writing are regulated by the: involuntary nervous system. central nervous system. autonomic nervous system. somatic nervous system.

somatic nervous system.

Skeletal muscle is also referred to as __________ muscle. connective smooth striated involuntary

striated

Patients with acute abdominal pain should not be given anything to eat or drink because: digestion prevents accurate auscultation of bowel sounds. substances in the stomach increase the risk of aspiration. food will rapidly travel through the digestive system. it will create referred pain and obscure the diagnosis.

substances in the stomach increase the risk of aspiration.

A burn that is characterized by redness and pain is classified as a: second-degree burn. superficial burn. partial-thickness burn. full-thickness burn.

superficial burn.

While caring for a patient, the EMT states to her partner, "Why even splint the patient's leg if they're only going to remove it in the ED?" This statement indicates that: the patient's leg does not require splinting. the EMT does not trust the hospital staff. the EMT's focus is not on the common goal. the EMT is being realistic in her thinking.

the EMT's focus is not on the common goal.

A dissecting aortic aneurysm occurs when: all layers of the aorta suddenly contract. the inner layers of the aorta become separated. the aorta ruptures, resulting in profound bleeding. a weakened area develops in the aortic wall.

the inner layers of the aorta become separated.

A "silent" heart attack occurs when: the usual chest pain is not present. a sudden dysrhythmia causes death. sweating is the only presentation. the patient minimizes the chest pain.

the usual chest pain is not present.

Infants are often referred to as "belly breathers" because: their rib cage is less rigid, and the ribs sit horizontally. their intercostal muscles are not functional. their diaphragm does not receive impulses from the brain. an infant's ribs are brittle and are less able to expand.

their rib cage is less rigid, and the ribs sit horizontally.

A person who routinely misuses a substance and requires increasing amounts to achieve the same effect is experiencing: addiction. dependence. withdrawal. tolerance.

tolerance.

You are transporting an immobilized patient with severe facial trauma. As you are preparing to give your radio report to the hospital, the patient begins vomiting large amounts of blood. You should: quickly suction his oropharynx. turn the backboard onto its side. reassess his breathing adequacy. alert the hospital of the situation.

turn the backboard onto its side.

Upon arriving at the scene of a major motor vehicle crash at night, you find that the safest place to park your ambulance is in a direction that is facing oncoming traffic. You should: place a flare near the crash. quickly access the patient. turn your headlights off. turn all warning lights off.

turn your headlights off.

You receive a call to a residence where a man found his wife unresponsive on the couch. The patient's respiratory rate is 8 breaths/min, her breathing is shallow, her heart rate is 40 beats/min, and her pulse is weak. The husband hands you an empty bottle of hydrocodone (Vicodin), which was refilled the day before. You should: ventilate her with a bag-mask device. contact the poison control center. perform a rapid head-to-toe exam. apply oxygen via a nonrebreathing mask.

ventilate her with a bag-mask device.

Capillaries link the arterioles and the: venules. cells. aorta. veins.

venules.

Gloves, a mask, eye protection, and a face shield should be used: when performing endotracheal intubation. while handling needles or other sharps. during routine cleaning of the ambulance. whenever you touch non-intact skin.

when performing endotracheal intubation.

When you are obtaining medical history from the family of a suspected stroke patient, it is MOST important to determine: if there is a family history of a stroke. when the patient last appeared normal. the patient's overall medication compliance. if the patient has been hospitalized before.

when the patient last appeared normal.

The carpal bones form the: wrist. hand. ankle. foot.

wrist.

A 60-year-old man complains of chest pain. He is conscious, alert, and denies shortness of breath. Which of the following questions would be the most appropriate to ask him? "Does the pain in your chest move to either of your arms?" "Does the pain in your chest feel like a stabbing sensation?" "Were you exerting yourself when the chest pain began?" "Do you have any heart problems or take any medications?"

"Do you have any heart problems or take any medications?"

Which of the following statements is not appropriate to document in the narrative section of a PCR? "After oxygen was administered, the patient's breathing improved." "The patient admits to smoking marijuana earlier in the day." "Significant damage was noted to the front end of the vehicle." "General impression revealed that the patient was intoxicated."

"General impression revealed that the patient was intoxicated."

An infant with a total blood volume of 800 mL would start showing signs of shock when as little as ______ of blood is lost. 150 mL 100 mL 50 mL 200 mL

100 mL

To ensure that you will deliver the appropriate number of chest compressions during one-rescuer adult CPR, you should compress the patient's chest at a rate of: at least 120 compressions per minute. 100 to 120 compressions per minute. 80 to 100 compressions per minute. no greater than 100 compressions per minute.

100 to 120 compressions per minute.

A 29-year-old male with a head injury opens his eyes when you speak to him, is confused as to the time and date, and is able to move all of his extremities on command. His Glasgow Coma Scale (GCS) score is: 10. 13. 12. 14.

13.

Following a head injury, a 20-year-old female opens her eyes spontaneously, is confused, and obeys your commands to move her extremities. You should assign her a GCS score of: 15. 13. 12. 14.

14.

A newborn infant will usually begin breathing spontaneously within _______ seconds following birth. 3 to 5 5 to 10 15 to 30 30 to 60

15 to 30

What is the correct ratio of compressions to ventilations when performing two-rescuer child CPR? 30:2 15:2 3:1 5:1

15:2

When pulling a patient, you should extend your arms no more than ________ in front of your torso. 15″ to 20″ 5″ to 10″ 10″ to 15″ 20″ to 30″

15″ to 20″

A mother who is pregnant with her first baby is typically in the first stage of labor for approximately: 4 hours. 8 hours. 10 hours. 16 hours.

16 hours.

A 21-year-old male was working in an auto repair shop and sustained radiator burns to the anterior aspect of both arms and to his anterior chest. According to the rule of nines, this patient has burns that cover _____ of his body surface area. 36% 18% 45% 27%

18%

The pressure of gas in a full cylinder of oxygen is approximately _______ pounds per square inch (psi). 3,000 1,000 500 2,000

2,000

An abortion occurs when the fetus and placenta deliver before: 20 weeks. 24 weeks. 26 weeks. 28 weeks.

20 weeks.

The air you breathe is _______ oxygen, and the air you exhale is _______ oxygen. 25%; 32% 21%; 35% 21%; 16% 16%; 25%

21%; 16%

A patient with a head injury presents with abnormal flexion of his extremities. What numeric value should you assign to him for motor response? 2 4 5 3

3

In which of the following situations does a legal duty to act clearly exist? The EMT hears of a cardiac arrest after his or her shift ends. A call is received 15 minutes prior to shift change. The EMT witnesses a vehicle crash while off duty. A bystander encounters a victim who is not breathing.

A call is received 15 minutes prior to shift change.

In which of the following situations would a direct ground lift be the most appropriate method of moving a patient? An unconscious patient with a possible ischemic stroke A conscious patient complaining of abdominal pain A patient who complains of hip pain following a fall A pedestrian with back pain after being struck by a car

A conscious patient complaining of abdominal pain

Shortly after you load your patient, a 50-year-old man with abdominal pain, into the ambulance, he tells you that he changed his mind and does not want to go to the hospital. The patient is conscious and alert and has no signs of mental incapacitation. You are suspicious that the man has a significant underlying condition and feel strongly that he should go to the hospital. Which of the following statements regarding this situation is correct? A mentally competent adult can withdraw his or her consent to treat at any time. Once the patient is in the ambulance, he cannot legally refuse EMS treatment. Because of your suspicions, the best approach is to transport him to the hospital. Any patient who refuses EMS treatment must legally sign a patient refusal form.

A mentally competent adult can withdraw his or her consent to treat at any time.

Which of the following statements regarding a patient refusal is correct? Documentation of proposed care is unnecessary if the patient refuses treatment. A mentally competent adult has the legal right to refuse EMS care and transport. Advice given to a patient who refuses EMS treatment should not be documented. A patient who consumed a few beers will likely be able to refuse EMS treatment.

A mentally competent adult has the legal right to refuse EMS care and transport.

Which of the following scenarios most accurately depicts abandonment? A paramedic transfers patient care to an EMT. A physician assumes patient care from an EMT. An EMT gives a verbal report to an emergency room nurse. An AEMT transfers patient care to a paramedic.

A paramedic transfers patient care to an EMT.

Which of the following general statements regarding consent is correct? Expressed consent is valid only if given in writing by a family member. A patient can consent to transport but can legally refuse treatment. All patients older than 18 years can legally refuse treatment or transport. Patients who are intoxicated are generally allowed to refuse treatment.

A patient can consent to transport but can legally refuse treatment.

Which of the following situations would require the use of a specialized rescue team? A patient in a badly damaged car, not entrapped A patient trapped in a cave or a confined space An obese patient who must be moved to the ambulance A patient found floating face down in a swimming pool

A patient trapped in a cave or a confined space

You are dispatched to a residence for a 40-year-old female who complains of lower abdominal pain, fever and chills, and a foul-smelling vaginal discharge. Which of the following additional assessment findings would increase your index of suspicion for pelvic inflammatory disease? Bright red blood in the urine A shuffling gait when walking Vaginal passage of blood clots A history of ectopic pregnancy

A shuffling gait when walking

EMTs are dispatched for a teenage male who is "not acting right." When they arrive, they are informed that the patient was huffing. Several cans of Freon are found near the patient. Which of the following is a unique consideration for this patient? The pulse oximeter will yield a falsely elevated reading. A sudden adrenaline release can cause a fatal dysrhythmia. Oxygen will not increase the patient's oxygen saturation. Encouraging walking will help eliminate Freon from the body.

A sudden adrenaline release can cause a fatal dysrhythmia.

189. Which of the following statements regarding a decreased level of consciousness in the elderly patient is correct? A) A decreased level of consciousness is not a normal part of the aging process. B) Most elderly patients have some deterioration in their level of consciousness. C) A decreased level of consciousness is most often the result of chronic dementia. D) The AVPU scale is an ineffective tool when assessing an elderly patient's level of consciousness.

A) A decreased level of consciousness is not a normal part of the aging process.

61. Which of the following statements regarding smooth muscle is correct? A) A person has no voluntary control over smooth muscle. B) Smooth muscle is found exclusively within blood vessels. C) The biceps and quadriceps are examples of smooth muscle. D) Smooth muscle is under control of the voluntary nervous system.

A) A person has no voluntary control over smooth muscle.

168. A 40-year-old unrestrained female impacted the steering wheel of her vehicle with her chest when she hit a tree while traveling at 45 mph. She is conscious and alert, but is experiencing significant chest pain and shortness of breath. Which of the following injuries is the LEAST likely? A) Head injury B) Cardiac contusion C) Pulmonary contusion D) Multiple rib fractures

A) Head injury

39. Which of the following statements regarding hepatitis A is correct? A) Hepatitis A can only be transmitted by a patient who has an acute infection. B) Infection with hepatitis A causes chronic illness with a high mortality rate. C) Hepatitis A is primarily transmitted via contact with blood or other body fluids. D) Although there is no vaccine against hepatitis A, treatment is usually successful.

A) Hepatitis A can only be transmitted by a patient who has an acute infection.

42. Which of the following medication routes has the slowest rate of absorption? A) Oral B) Rectal C) Inhalation D) Sublingual

A) Oral

188. The components of the pediatric assessment triangle (PAT) are: A) appearance, work of breathing, and skin circulation. B) mental status, heart rate, and systolic blood pressure. C) skin condition, respiratory rate, and level of alertness. D) activity, respiratory quality, and level of consciousness

A) appearance, work of breathing, and skin circulation.

54. External bleeding from an extremity can usually be controlled initially by: A) applying direct pressure. B) elevating the extremity. C) applying a tourniquet. D) applying chemical ice packs.

A) applying direct pressure.

156. You receive a call for a 70-year-old female with respiratory distress. Her husband tells you that she has congestive heart failure; however, he does not think that she has been taking her medications as prescribed. The patient is laboring to breathe, appears tired, and has cyanosis around her lips. You should: A) assist her ventilations with a bag-valve mask. B) apply a pulse oximeter and assess her vital signs. C) administer oxygen via a nonrebreathing mask. D) obtain a complete list of all of her medications

A) assist her ventilations with a bag-valve mask

59. When faced with a situation in which a patient is in cardiac arrest and a valid living will or DNR order cannot be located, you should: A) begin resuscitation at once. B) contact medical control first. C) determine the patient's illness. D) notify the coroner immediately.

A) begin resuscitation at once.

41. You are treating a middle-aged man with chest discomfort. He has a history of three previous heart attacks and takes nitroglycerin as needed for chest pain. You have standing orders to administer aspirin to patients with suspected cardiac-related chest pain or discomfort. While your partner is preparing to give oxygen to the patient, you should: A) confirm that the patient is not allergic to aspirin, give him the appropriate dose of aspirin, and document the time and dose given. B) contact medical control, apprise him or her of the patient's chief complaint and vital signs, and request permission to give him aspirin. C) ensure that the patient's systolic blood pressure is at least 100 mm Hg as aspirin dilates the blood vessels and can cause a drop in blood pressure. D) assist the patient in taking one of his prescribed nitroglycerin, assess his vital signs, and give him aspirin if he is still experiencing chest discomfort.

A) confirm that the patient is not allergic to aspirin, give him the appropriate dose of aspirin, and document the time and dose given.

148. You are dispatched to an office building for a 49-year-old male with chest pain. When you arrive at the scene, you find the patient to be conscious and alert, but in obvious pain. He tells you that he did not call 9-1-1; a coworker did. He further states that he does not want to be treated or transported to the hospital. You should: A) ensure that he is aware of the risks of refusing medical care. B) err in the best interest of the patient and transport him at once. C) have him sign a refusal of care form and then return to service. D) tell him that he is having a heart attack and needs medical care.

A) ensure that he is aware of the risks of refusing medical care.

68. When the shoulder girdle is aligned over the pelvis during liftinghe driver, a young female, sitting on the curb. She is confused; is in obvious respiratory distress; and has pale, moist skin. As your partner manually stabilizes her head, you perform a primary assessment. After performing any immediate life-saving treatment, you should: A) perform a detailed head-to-toe exam and prepare for immediate transport. B) assess her vital signs, secure her to a backboard, and transport her immediately. C) fully immobilize her spine, load her into the ambulance, and assess her vital signs. D) identify the specific areas of her injuries and focus your assessment on those areas.

A) perform a detailed head-to-toe exam and prepare for immediate transport.

50. An absence seizure is also referred to as a: A) petit mal seizure. B) grand mal seizure. C) total body seizure. D) generalized motor seizure.

A) petit mal seizure.

17. A 70-year-old female was recently discharged from the hospital following a total hip replacement. Today, she presents with restlessness, tachycardia, and a blood pressure of 100/64 mm Hg. Her skin is hot and moist. You should be MOST suspicious that she is experiencing: A) septic shock. B) pump failure. C) a local infection. D) decompensated shock.

A) septic shock.

200. The standards for prehospital emergency care and the individuals who provide it are typically regulated by the: A) state office of EMS. B) regional trauma center. C) American Heart Association. D) National Registry of EMTs.

A) state office of EMS.

152. With increasing age, the heart must work harder to move the blood effectively because: A) the blood vessels become stiff. B) the arteries dilate significantly. C) diastolic blood pressure decreases. D) the blood thickens as a person ages.

A) the blood vessels become stiff.

64. Prompt transport of a patient with a suspected AMI is important because: A) the patient may be eligible to receive thrombolytic therapy. B) 90% of the cardiac cells will die within the first 30 minutes. C) nitroglycerin can only be given in the emergency department. D) many patients with an AMI die within 6 hours.

A) the patient may be eligible to receive thrombolytic therapy

67. When the shoulder girdle is aligned over the pelvis during lifting: A) the weight is exerted straight down the vertebrae. B) the hands can be held further apart from the body. C) the muscles of the back experience increased strain. D) the risk of back injuries is significantly increased.

A) the weight is exerted straight down the vertebrae.

99. Infants are often referred to as "belly breathers" because: A) their rib cage is less rigid and the ribs sit horizontally. B) an infant's ribs are brittle and are less able to expand. C) their intercostal muscles are not functional. D) their diaphragm does not receive impulses from the brain.

A) their rib cage is less rigid and the ribs sit horizontally.

18. Abdominal thrusts in a conscious child or adult with a severe upper airway obstruction are performed: A) until he or she loses consciousness. B) in sets of five followed by reassessment. C) about 1 inch below the xiphoid process. D) until he or she experiences cardiac arrest.

A) until he or she loses consciousness.

167. During your assessment of a patient who experienced a blast injury, you note that he has a depressed area to the front of his skull. This injury MOST likely occurred: A) when the patient was hurled against a stationary object. B) as a result of flying debris. C) by inhaling toxic gases. D) as a direct result of the pressure wave.

A) when the patient was hurled against a stationary object.

104. Which of the following statements would NOT be appropriate to say to the family of a dying patient? A. "Things will get better in time." B. "It is okay to be angry and sad." C. "This must be painful for you." D. "Tell me how you are feeling."

A. "Things will get better in time."

15. Which of the following patients does NOT have an altered mental status? A. a patient with an acute allergic reaction and dizziness B. a diabetic who opens his eyes when you ask questions C. a patient with a head injury who is slow to answer questions D. a patient who overdosed and moans when he is touched

A. a patient with an acute allergic reaction and dizziness

127. In what area of the lungs does respiration occur? A. alveoli B. trachea C. bronchi D. capillaries

A. alveoli

97. An EMT may injure his or her back, even if it is straight, if the: A. back is bent forward at the hips. B. hands are held close to the legs. C. shoulder is aligned over the pelvis. D. force is exerted straight down the spine.

A. back is bent forward at the hips.

114. You and your partner arrive at the side of a 60-year-old woman who suddenly collapsed about 7 minutes ago. She is unresponsive, apneic, and pulseless. You should: A. begin CPR and apply the AED as soon as it is available. B. immediately apply the AED and analyze her cardiac rhythm. C. begin CPR at a compression to ventilation ratio of 15:2. D. apply the AED if there is no response after 10 cycles of CPR

A. begin CPR and apply the AED as soon as it is available.

107. When faced with a situation in which a patient is in cardiac arrest, and a valid living will or DNR order cannot be located, you should: A. begin resuscitation at once. B. contact medical control first. C. determine the patient's illness. D. notify the coroner immediately.

A. begin resuscitation at once.

105. Carbon monoxide blocks the ability of the blood to oxygenate the body because it: A. binds with the hemoglobin in the red blood cells. B. fills the lungs with secretions. C. causes the body to expel too much carbon dioxide. D. destroys the number of circulating red blood cells.

A. binds with the hemoglobin in the red blood cells.

140. When an electrical impulse reaches the AV node, it is slowed for a brief period of time so that: A. blood can pass from the atria to the ventricles. B. blood returning from the body can fill the atria. C. the impulse can spread through the Purkinje fibers. D. the SA node can reset and generate another impulse

A. blood can pass from the atria to the ventricles.

37. When perfusion to the core of the body decreases: A. blood is shunted away from the skin. B. decreased cardiac contractility occurs. C. blood is diverted to the gastrointestinal tract. D. the voluntary nervous system releases hormones

A. blood is shunted away from the skin.

92. The musculoskeletal system refers to the: A. bones and voluntary muscles of the body. B. nervous system's control over the muscles. C. connective tissue that supports the skeleton. D. involuntary muscles of the nervous system.

A. bones and voluntary muscles of the body.

82. The _________ is the best-protected part of the CNS and controls the functions of the cardiac and respiratory systems. A. brain stem B. cerebellum C. spinal cord D. cerebral cortex

A. brain stem

77. The _________ contain(s) about 75% of the brain's total volume. A. cerebrum B. cerebellum C. brain stem D. meninges

A. cerebrum

3. What three bones make up the shoulder girdle? A. clavicle, scapula, humerus B. acromion, clavicle, scapula C. acromion, scapula, humerus D. acromion, humerus, clavicle

A. clavicle, scapula, humerus

19. When you inspect a patient's pupils with a penlight, the pupils should normally react to the light by: A. constricting. B. enlarging. C. dilating. D. fluttering.

A. constricting.

30. To select the proper size oropharyngeal airway, you should measure from the: A. corner of the mouth to the earlobe. B. center of the mouth to the posterior ear. C. corner of the mouth to the superior ear. D. angle of the jaw to the center of the mouth.

A. corner of the mouth to the earlobe.

103. Burns are classified according to: A. depth and extent. B. location and pain. C. degree and location. D. extent and location.

A. depth and extent.

129. Weakening of the airway in patients with chronic bronchitis is the result of: A. destruction of protective mechanisms that remove foreign particles. B. loss of the lubricating substance that facilitates alveolar expansion. C. airway irritation caused by a marked decrease in mucus production. D. acute constriction of the bronchioles caused by an external irritant.

A. destruction of protective mechanisms that remove foreign particles.

11. When approaching a 32-year-old male who is complaining of traumatic neck pain, you should: A. ensure that the patient can see you approaching him. B. approach him from behind and ask him not to move. C. stand behind him and immediately stabilize his head. D. assess his mental status by having him move his head.

A. ensure that the patient can see you approaching him.

43. Neurogenic shock occurs when: A. failure of the nervous system causes widespread vasodilation. B. the spinal cord is severed and causes massive hemorrhaging. C. there is too much blood to fill a smaller vascular container. D. massive vasoconstriction occurs distal to a spinal cord injury.

A. failure of the nervous system causes widespread vasodilation.

123. When assessing a patient with a medical complaint, which of the following would MOST likely reveal the cause of his or her problem? A. history taking B. rapid body scan C. baseline vital signs D. primary assessment

A. history taking

35. Shock is the result of: A. hypoperfusion to the cells of the body. B. the body's maintenance of homeostasis. C. temporary dysfunction of a major organ. D. widespread constriction of the blood vessels.

A. hypoperfusion to the cells of the body.

34. Which of the following organs or tissues can survive the longest without oxygen? A. muscle B. heart C. liver D. kidneys

A. muscle

88. Irritation or damage to the pleural surfaces that causes sharp chest pain during inhalation is called: A. pleurisy. B. dyspnea. C. pneumonitis. D. pneumothorax.

A. pleurisy.

116. CPR will NOT be effective if the patient is: A. prone. B. supine. C. horizontal. D. on a firm surface

A. prone.

53. The systemic veins function by: A. returning deoxygenated blood back to the heart. B. delivering oxygen-poor blood to the capillaries. C. returning oxygen-rich blood back to the left atrium. D. delivering deoxygenated blood to the capillaries.

A. returning deoxygenated blood back to the heart.

56. Hypoperfusion is another name for: A. shock. B. cyanosis. C. hypoxemia. D. cellular death.

A. shock.

102. The heart's primary pacemaker is the: A. sinoatrial (SA) node. B. internodal pathway. C. atrioventricular (AV) node. D. ventricular bundle of His.

A. sinoatrial (SA) node.

If you could only take two pieces of equipment with you to a patient's side, you should choose the: cervical collar and long backboard. BVM and portable oxygen. oral airways and sterile dressings. AED and portable suction unit.

AED and portable suction unit.

A patient with a left ventricular assist device (LVAD) tells you that the device's pump flow is continuous. Which of the following should you expect to encounter during your assessment? High systolic blood pressure Low diastolic blood pressure Distention of the jugular veins Absence of a palpable pulse

Absence of a palpable pulse

Under what circumstances is a left ventricular assist device used? To reduce ventricular pumping force in patients with aortic aneurysms To ensure that the ventricles contract at an adequate and consistent rate To permanently replace the function of one or both of the ventricles As a bridge to heart transplantation while a donor heart is being located

As a bridge to heart transplantation while a donor heart is being located

A 23-year-old male experienced severe head trauma after his motorcycle collided with an oncoming truck. He is unconscious, has agonal gasps, and has copious bloody secretions in his mouth. How should you manage his airway? Suction his oropharynx with a rigid catheter until all secretions are removed. Insert a nasopharyngeal airway and provide suction and assisted ventilations. Provide continuous ventilations with a bag-mask device to minimize hypoxia. Alternate oropharyngeal suctioning and ventilation with a bag-mask device.

Alternate oropharyngeal suctioning and ventilation with a bag-mask device.

Which of the following scenarios is an example of informed consent? An EMT advises a patient of the risks of receiving treatment. A patient advises an EMT of why he or she is refusing care. An EMT initiates immediate care for an unconscious adult. A patient is advised by an EMT of the risks of refusing care.

An EMT advises a patient of the risks of receiving treatment.

Which of the following scenarios most accurately depicts a posttraumatic stress disorder (PTSD) reaction? A newly certified EMT becomes extremely nauseated and diaphoretic at the scene of an incident involving grotesque injuries. An EMT becomes distracted at the scene of a motor vehicle crash involving the same type of car in which a child was previously killed. An EMT is emotionally exhausted and depressed after a school bus crash involving critical injuries and the death of several children. An EMT with many years of field experience becomes irritable with her coworkers and experiences headaches and insomnia.

An EMT becomes distracted at the scene of a motor vehicle crash involving the same type of car in which a child was previously killed.

Which of the following statements regarding the metered-dose inhaler (MDI) is correct? MDIs are contraindicated for patients with asthma or emphysema. MDIs are most commonly used by patients with cardiovascular disease. Shaking an MDI prior to use will cause deactivation of the medication. An MDI delivers the same amount of medication every time it is used.

An MDI delivers the same amount of medication every time it is used.

Which of the following statements regarding an infant's vital signs is correct? An infant's normal body temperature is typically higher than a preschooler's normal body temperature. An infant's heart rate generally ranges between 70 and 110 beats/min. By six months of age, an infant's normal tidal volume is 2 to 4 mL/kg. An infant's normal heart rate increases by 10 beats/min each month.

An infant's normal body temperature is typically higher than a preschooler's normal body temperature.

Which of the following statements regarding anaphylaxis is correct? Patients with asthma are at lower risk of developing anaphylaxis. The signs of anaphylaxis are caused by widespread vasoconstriction. Anaphylaxis is characterized by airway swelling and hypotension. Most anaphylactic reactions occur within 60 minutes of exposure.

Anaphylaxis is characterized by airway swelling and hypotension.

Premature diagnosis during a call can be due to what error? Overconfidence Anchoring Bias Streamlining

Anchoring

A patient is bleeding severely from a severed femoral artery high in the groin region. Which of the following would most likely control the bleeding? Apply chemical ice packs to the wound and transport. Apply a topical hemostatic agent with direct pressure. Position the patient with his injured side down. Apply a pelvic binder device to stabilize the pelvis.

Apply a topical hemostatic agent with direct pressure.

During your visual inspection of a 25-year-old woman in labor, you see the baby's head crowning at the vaginal opening. What should you do? Apply gentle pressure to the baby's head as it delivers. Tell the mother not to push and transport her immediately. Place your fingers in the vagina to assess for a nuchal cord. Maintain firm pressure to the head until it completely delivers.

Apply gentle pressure to the baby's head as it delivers.

You are assessing a 45-year-old female who is severely depressed. She states that it seems as though her entire world is crashing down around her. She further states that she has had frequent thoughts of suicide, but is not sure if she can actually go through with it. How should you manage this situation? Have law enforcement place her in protective custody. Leave the scene and have a neighbor check in on her. Ask the patient if she has developed a suicidal plan. Encourage the patient to remain quiet during transport.

Ask the patient if she has developed a suicidal plan.

A 58-year-old male presents with confusion, right-sided weakness, and slurred speech. His airway is patent, and his breathing is adequate. His wife is present and is very upset. Which of the following has the MOST immediately priority? Asking his wife when she noticed the symptoms Obtaining a complete set of baseline vital signs Documenting all of his current medications Administering glucose to rule out hypoglycemia

Asking his wife when she noticed the symptoms

Which of the following will MOST reliably allow you to determine the nature of a patient's illness? Refraining from asking open-ended questions Trending of the patient's vital signs over time Focusing solely on how the call is dispatched Asking questions related to the chief complaint

Asking questions related to the chief complaint

95. Which of the following scenarios is an example of informed consent? A) A patient advises an EMT of why he or she is refusing care. B) An EMT advises a patient of the risks of receiving treatment. C) An EMT initiates immediate care for an unconscious adult. D) A patient is advised by an EMT of the risks of refusing care

B) An EMT advises a patient of the risks of receiving treatment.

196. Which of the following skills or interventions is included at every level of prehospital emergency training? A) Oral glucose for hypoglycemia B) Automated external defibrillation C) Intranasal medication administration D) Use of a manually triggered ventilator

B) Automated external defibrillation

26. Which of the following statements regarding gastrointestinal bleeding is correct? A) In the majority of cases, bleeding within the gastrointestinal tract occurs acutely and is severe. B) Bleeding within the gastrointestinal tract is a symptom of another disease, not a disease itself. C) Lower gastrointestinal bleeding results from conditions such as Mallory-Weiss syndrome. D) Chronic bleeding within the gastrointestinal tract is usually more severe than bleeding that occurs acutely.

B) Bleeding within the gastrointestinal tract is a symptom of another disease, not a disease itself.

195. Which of the following statements regarding a "dirty bomb" is correct? A) The effectiveness of a dirty bomb is solely dependent on the amount of alpha radiation that it emits. B) Dirty bombs could injure victims with both radioactive material and the explosive material used to deliver it. C) Dirty bombs contain significant amounts of radioactive material and are capable of being delivered via a missile. D) The dirty bomb, because of its ability to cause massive damage over a large geographic area, is an effective WMD.

B) Dirty bombs could injure victims with both radioactive material and the explosive material used to deliver it.

84. You have two patients who were involved in a motor vehicle crash when their SUV struck a tree—one with neck and back pain, and the other with a deformed left femur. The patient with the deformed femur states that he does not want to be placed on a hard board, nor does he want a collar around his neck. What is the MOST appropriate and practical method of securing these patients and placing them into the ambulance? A) Immobilize both patients with a cervical collar and long backboard based on the mechanism of injury; place one on the wheeled stretcher and the other on the squad bench. B) Immobilize the patient with neck and back pain on a long backboard and place him on the wheeled stretcher; place the patient with the deformed femur on a folding stretcher secured to the squad bench. C) Immobilize the patient with neck and back pain on a long backboard and place him on the squad bench; allow the patient with the deformed femur to sit on the wheeled stretcher. D) Apply a traction splint to the patient with the deformed femur and place him on the wheeled stretcher; place the patient with neck and back pain on the squad bench immobilized with a cervical collar and scoop stretcher.

B) Immobilize the patient with neck and back pain on a long backboard and place him on the wheeled stretcher; place the patient with the deformed femur on a folding stretcher secured to the squad bench.

75. When gathering a patient's medications, you find the following: Isordil, Lasix, Nexium, and digoxin. Which of these medications can be obtained over-the-counter (OTC)? A) Lasix B) Nexium C) Isordil D) Digoxin

B) Nexium

180. Which of the following statements regarding abdominal trauma is correct? A) Most of the vital abdominal organs lie within the retroperitoneal space. B) The absence of abdominal pain does not rule out intra-abdominal bleeding. C) The liver is well protected and is rarely injured during a traumatic event. D) Hollow abdominal organs are vascular and bleed profusely when injured.

B) The absence of abdominal pain does not rule out intra-abdominal bleeding

45. You are assessing a 49-year-old man who, according to his wife, experienced a sudden, severe headache and then passed out. He is unresponsive and has slow, irregular breathing. His blood pressure is 190/94 mm Hg and his pulse rate is 50 beats/min. His wife tells you that he has hypertension and diabetes. He has MOST likely experienced: A) acute hypoglycemia. B) a ruptured cerebral artery. C) a complex partial seizure. D) an occluded cerebral artery.

B) a ruptured cerebral artery.

193. Which of the following is a typical function of the rescue team? A) Initial triage and patient prioritization B) Shutting off power to downed electrical lines C) Providing safe entry and access to patients D) Providing emergency care during extrication

C) Providing safe entry and access to patients

98. When assessing an 80-year-old patient in shock, it is important to remember that: A) it is common to see a more significant increase in heart rate than what would be expected in younger adults. B) age-related changes in the cardiovascular system may make the patient less able to compensate for decreased perfusion. C) the patient's cardiac output is able to increase by nearly 200% in response to the decrease in perfusion. D) in older adults, it is especially common to observe a significant decrease in heart rate in response to shock.

B) age-related changes in the cardiovascular system may make the patient less able to compensate for decreased perfusion.

24. Erosion of the protective layer of the stomach or duodenum secondary to overactivity of digestive juices results in: A) ileus. B) an ulcer. C) appendicitis. D) cholecystitis.

B) an ulcer.

52. A 58-year-old man complains of chest discomfort and nausea. He is conscious and alert; his blood pressure is 140/90 mm Hg, his pulse is 104 beats/min, and his respirations are 16 breaths/min. Your partner has applied supplemental oxygen. Prior to assisting the patient with one of his prescribed nitroglycerin tablets, you ask him if he takes medication to treat erectile dysfunction (ED) and he tells you that he does. You should: A) avoid giving him nitroglycerin and transport him at once. B) ask him what he takes, how much, and when he last took it. C) recall that erectile ED drugs can cause significant hypertension. D) administer his nitroglycerin and then reassess his blood pressure.

B) ask him what he takes, how much, and when he last took it.

155. After establishing that an adult patient is unresponsive, you should: A) manually open the airway. B) assess for breathing and a pulse. C) immediately begin chest compressions. D) apply the AED and deliver a shock, if needed

B) assess for breathing and a pulse.

78. The areas of the spinal column, in descending order, are: A) cervical, thoracic, sacral, lumbar, and coccyx. B) cervical, thoracic, lumbar, sacral, and coccyx. C) cervical, thoracic, coccyx, lumbar, and sacral. D) cervical, lumbar, thoracic, sacral, and coccyx.

B) cervical, thoracic, lumbar, sacral, and coccyx.

149. All information recorded on the PCR must be: A) typewritten or printed. B) considered confidential. C) a matter of public record. D) reflective of your opinion.

B) considered confidential.

159. The principal symptom in both infectious and noninfectious gastroenteritis is: A) vomiting. B) diarrhea. C) dysuria. D) high fever

B) diarrhea.

170. An effective team leader should: A) command his or her team. B) help the team accomplish goals. C) perform all difficult interventions. D) refrain from any direct patient care

B) help the team accomplish goals.

71. You are dispatched to a residence where a middle-aged man was found unconscious in his front yard. There are no witnesses who can tell you what happened. You find him in a prone position; his eyes are closed and he is not moving. Your FIRST action should be to: A) palpate for the presence of a carotid pulse. B) log roll him as a unit to a supine position. C) assess the rate and quality of his breathing. D) open his airway with a jaw-thrust maneuver.

B) log roll him as a unit to a supine position.

175. When a patient experiences a severe spinal injury, he or she: A) will likely be paralyzed from the neck down. B) may lose sensation below the level of the injury. C) most commonly has a palpable spinal deformity. D) often loses motor function on one side of the body.

B) may lose sensation below the level of the injury.

136. A 71-year-old male is semiconscious following a sudden, severe headache. There is vomitus on his face and his respirations are slow and shallow. The EMT must immediately: A) insert a nasopharyngeal airway. B) perform oropharyngeal suctioning. C) apply oxygen via a nonrebreathing mask. D) begin assisting the patient's ventilations.

B) perform oropharyngeal suctioning.

79. Relative to an adult's airway anatomy, the child's: A) tongue takes up less space in the pharynx. B) pharynx is smaller and less deeply curved. C) trachea is smaller, softer, and less flexible. D) mouth and nose are proportionately larger

B) pharynx is smaller and less deeply curved.

55. Observations made when forming a general impression of a patient would include all of the following, EXCEPT: A) appearance. B) pulse strength. C) race and gender. D) level of distress.

B) pulse strength.

163. You respond to a call for an unknown emergency. When you arrive at the scene, the patient's husband meets you at the door and states that his wife has been depressed and has locked herself in an upstairs bedroom. He further tells you that he keeps his handgun in the bedroom. You should: A) ask the husband to attempt to reason with his wife. B) remain in a safe place and request law enforcement. C) get in your ambulance and leave the scene immediately. D) go upstairs with caution and attempt to talk to the patient.

B) remain in a safe place and request law enforcement.

57. You have been working at the scene of a major building collapse for 8 hours. Many injured people are still being removed, and everyone is becoming frustrated and losing focus. This situation is MOST effectively managed by: A) providing large amounts of caffeine to the rescue workers. B) requesting a CISM team to provide on-scene peer support. C) conducting a critical incident stress debriefing the next day. D) allowing each worker to sleep in 15- to 30-minute increments.

B) requesting a CISM team to provide on-scene peer support.

184. Rough handling of a hypothermic patient with a pulse may cause: A) profound bradycardia. B) ventricular fibrillation. C) ventricular tachycardia. D) pulseless electrical activity.

B) ventricular fibrillation

138. What is the function of the left atrium? A. It ejects oxygenated blood into the aorta. B. It receives oxygenated blood from the lungs. C. It receives blood from the pulmonary arteries. D. It receives oxygenated blood from the vena cava.

B. It receives oxygenated blood from the lungs.

7. Which of the following statements regarding the mechanism of injury (MOI) is correct? A. A nonsignificant MOI rules out the possibility of serious trauma. B. The MOI may allow you to predict the severity of a patient's injuries. C. The exact location of a patient's injuries can be determined by the MOI. D. A significant MOI always results in patient death or permanent disability.

B. The MOI may allow you to predict the severity of a patient's injuries.

65. Which of the following statements regarding the dermis is correct? A. The dermis produces a substance that provides color to the skin. B. The dermis contains hair follicles, sweat glands, and nerve endings. C. The cells of the dermis are worn away and are constantly replaced. D. The dermis lies above the germinal layer and provides protection.

B. The dermis contains hair follicles, sweat glands, and nerve endings.

113. Most prehospital cardiac arrests occur as the result of: A. severe blunt trauma. B. a cardiac arrhythmia. C. an acute ischemic stroke. D. obstruction of the airway

B. a cardiac arrhythmia.

101. Vasovagal reactions that occur in patients receiving IV therapy are MOST often the result of: A. the pain associated with venipuncture. B. a fear of needles or the sight of blood. C. too much IV fluid being administered. D. an unexpected reaction to the IV fluid.

B. a fear of needles or the sight of blood.

73. The MOST significant complication associated with facial injuries is: A. damage to the eyes. B. airway compromise. C. cervical spine injury. D. mandibular immobility.

B. airway compromise.

164. Which of the following is an example of a functional behavioral disorder? A) Head trauma B) Drug addiction C) Schizophrenia D) Alzheimer disease

C) Schizophrenia

124. A 58-year-old man complains of chest discomfort and nausea. He is conscious and alert; his blood pressure is 140/90 mm Hg, his pulse is 104 beats/min, and his respirations are 16 breaths/min. Your partner has applied supplemental oxygen. Prior to assisting the patient with one of his prescribed nitroglycerin tablets, you ask him if he takes medication to treat erectile dysfunction and he tells you that he does. You should: A. avoid giving him nitroglycerin and transport him at once. B. ask him what he takes, how much, and when he last took it. C. recall that erectile dysfunction drugs can cause hypertension if given with nitroglycerin. D. administer his nitroglycerin and then reassess his blood pressure.

B. ask him what he takes, how much, and when he last took it.

74. The central nervous system (CNS) is composed of the: A. cerebellum and brain. B. brain and spinal cord. C. cerebrum and meninges. D. meninges and spinal cord.

B. brain and spinal cord.

40. Which of the following injuries would MOST likely cause obstructive shock? A. liver laceration B. cardiac tamponade C. simple pneumothorax D. spinal cord injury

B. cardiac tamponade

63. What layer of the skin forms a watertight, protective seal for the body? A. dermis B. epidermis C. muscular layer D. subcutaneous layer

B. epidermis

69. The eyeball itself is referred to as the: A. orbit. B. globe. C. sclera. D. cornea.

B. globe.

1. Which of the following is an example of a symptom? A. cyanosis B. headache C. tachycardia D. hypertension

B. headache

32. The MOST significant complication associated with oropharyngeal suctioning is: A. oral abrasions from vigorous suctioning. B. hypoxia due to prolonged suction attempts. C. clogging of the catheter with thick secretions. D. vomiting from stimulating the anterior airway.

B. hypoxia due to prolonged suction attempts.

115. Gastric distention will MOST likely occur: A. in patients who are intubated. B. if you ventilate a patient too fast. C. when you deliver minimal tidal volume. D. when the airway is completely obstructed.

B. if you ventilate a patient too fast

128. The two processes that occur during respiration are: A. ventilation and diffusion. B. inspiration and expiration. C. diffusion and oxygenation. D. oxygenation and ventilation.

B. inspiration and expiration.

110. In contrast to inhalation, exhalation: A. requires muscular effort to effectively expel air from the lungs. B. is a passive process caused by increased intrathoracic pressure. C. occurs when the diaphragm lowers and expels air from the lungs. D. is an active process caused by decreased intrathoracic pressure.

B. is a passive process caused by increased intrathoracic pressure.

48. The energy of a moving object is called: A. latent energy. B. kinetic energy. C. potential energy. D. converted energy

B. kinetic energy

111. The hypoxic drive—the primary stimulus to breathe for patients with certain chronic respiratory diseases—is influenced by: A. high blood oxygen levels. B. low blood oxygen levels. C. low blood carbon dioxide levels. D. high blood carbon dioxide levels.

B. low blood oxygen levels.

8. Observations made when forming a general impression of a patient would include all of the following, EXCEPT: A. appearance. B. pulse strength. C. race and gender. D. level of distress

B. pulse strength.

46. When a motor vehicle strikes a tree while traveling at 40 mph, the unrestrained occupant: A. will most likely be thrown over the steering column. B. remains in motion until acted upon by an external force. C. will decelerate at the same rate as the motor vehicle. D. is thrust under the steering column onto the floorboard.

B. remains in motion until acted upon by an external force.

29. A 19-year-old female is found unconscious by her roommate. Your primary assessment reveals that her breathing is inadequate. As you insert an oropharyngeal airway, she begins to gag violently. You should: A. continue to insert the airway as you suction her oropharynx. B. remove the airway and be prepared to suction her oropharynx. C. insert the airway no further but leave it in place as a bite block. D. select a smaller oropharyngeal airway and attempt to insert it.

B. remove the airway and be prepared to suction her oropharynx.

120. In MOST cases, cardiopulmonary arrest in infants and children is caused by: A. a drug overdose. B. respiratory arrest. C. severe chest trauma. D. a cardiac dysrhythmia.

B. respiratory arrest.

135. Harsh, high-pitched inspiratory sounds are characteristic of: A. rales. B. stridor. C. rhonchi. D. wheezing.

B. stridor

36. When the body senses a state of hypoperfusion, the sympathetic nervous system releases epinephrine, the effects of which include: A. tachypnea. B. tachycardia. C. vasodilation. D. restlessness.

B. tachycardia.

93. Skeletal muscle is attached to the bone by tough, ropelike fibrous structures called: A. fascia. B. tendons. C. cartilage. D. ligaments.

B. tendons.

51. You arrive at the home of a 50-year-old female with severe epistaxis. As you are treating her, it is MOST important to recall that: A. the patient may be significantly hypertensive. B. the patient is at risk for vomiting and aspiration. C. a detailed exam is needed to determine the cause. D. many medications interfere with blood clotting.

B. the patient is at risk for vomiting and aspiration

25. What is the MOST common cause of airway obstruction in an unconscious patient? A. vomitus B. the tongue C. blood clots D. aspirated fluid

B. the tongue

87. Immediate death from blunt chest trauma following a motor vehicle crash is MOST often the result of: A. a tension pneumothorax. B. traumatic aortic rupture. C. penetrating lung injuries. D. a massive cardiac contusion.

B. traumatic aortic rupture.

198. Which of the following statements regarding the different stages of the grieving process is correct? A) The grieving process typically begins with severe depression. B) It is rare that people will jump back and forth between stages of the grieving process. C) The stages of the grieving process may occur simultaneously. D) Bargaining is the most unpleasant stage of the grieving process.

C) The stages of the grieving process may occur simultaneously

122. An index of suspicion is MOST accurately defined as: A. the EMT's prediction of the type of illness a patient has based on how the call is dispatched. B. your awareness and concern for potentially serious underlying and unseen injuries or illness. C. ruling out specific medical conditions based on the absence of certain signs and symptoms. D. determining the underlying cause of a patient's medical condition based on signs and symptoms.

B. your awareness and concern for potentially serious underlying and unseen injuries or illness.

After assuming care of a cardiac arrest patient from an EMT, the paramedic should remember that: ALS interventions are the core interventions around which BLS care is provided. BLS efforts must continue throughout the patient care continuum. the BLS care provided by the EMT is the "first steps" of ALS care. ALS interventions are fundamentally more critical than BLS interventions.

BLS efforts must continue throughout the patient care continuum.

Which of the following sets of vital signs would the EMT most likely encounter in a patient with acute cocaine overdose? BP, 60/40 mm Hg; pulse, 140 beats/min BP, 190/90 mm Hg; pulse, 40 beats/min BP, 200/100 mm Hg; pulse, 150 beats/min BP, 180/100 mm Hg; pulse, 50 beats/min

BP, 200/100 mm Hg; pulse, 150 beats/min

76. Which of the following clinical signs is unique to anaphylactic shock? A) Pallor B) Dizziness C) Wheezing D) Hypotension

C) Wheezing

6. You are dispatched to a residence for a 67-year-old female who was awakened by shortness of breath and sharp chest pain. Her husband tells you that she was recently discharged from the hospital after having hip surgery. Your assessment reveals dried blood around her mouth, facial cyanosis, and an oxygen saturation of 88%. You should suspect: A) acute pulmonary edema. B) right-sided heart failure. C) acute pulmonary embolism. D) spontaneous pneumothorax.

C) acute pulmonary embolism.

186. You are assessing a 25-year-old woman who is 39 weeks pregnant. She is experiencing regular contractions that are approximately 3 minutes apart and states that her amniotic sac broke 2 hours ago. After taking the standard precautions, you should: A) apply high-flow oxygen. B) place her on her left side. C) assess her for crowning. D) transport her immediately.

C) assess her for crowning.

147. If a person is partially immune to a particular disease, he or she: A) is not protected from a new infection if exposed to another individual. B) must be revaccinated at least every 18 months to avoid infection. C) may develop illness from germs that lie dormant from the initial infection. D) will not experience future illness, even if his or her immune system is stressed.

C) may develop illness from germs that lie dormant from the initial infection.

20. The left ventricle has the thickest walls because it: A) pumps blood to the lungs to be reoxygenated. B) uses less oxygen than other chambers of the heart. C) pumps blood into the aorta and systemic circulation. D) receives blood directly from the systemic circulation.

C) pumps blood into the aorta and systemic circulation

9. Typical components of an oral patient report include all of the following, EXCEPT: A) the chief complaint or mechanism of injury. B) important medical history not previously given. C) the set of baseline vital signs taken at the scene. D) the patient's response to treatment you provided.

C) the set of baseline vital signs taken at the scene.

58. If you are exposed to a patient's blood or other bodily fluid, your first action should be to: A) report the incident to the infection control officer. B) abandon patient care and seek medical attention. C) transfer care of the patient to another EMS provider. D) vigorously clean the area with soap and water.

C) transfer care of the patient to another EMS provider.

23. An adult at rest should have a respiratory rate that ranges between: A. 8 and 15 breaths/min. B. 10 and 18 breaths/min. C. 12 and 20 breaths/min. D. 16 and 24 breaths/min.

C. 12 and 20 breaths/min.

33. A 23-year-old male experienced severe head trauma after his motorcycle collided with an oncoming truck. He is unconscious, has rapid and shallow breathing, and has copious bloody secretions in his mouth. How should you manage his airway? A. Suction his oropharynx with a rigid catheter until all secretions are removed. B. Insert a nasopharyngeal airway and provide suction and assisted ventilations. C. Alternate 15 seconds of oral suctioning with 2 minutes of assisted ventilation. D. Provide continuous ventilations with a bag-mask device to minimize hypoxia.

C. Alternate 15 seconds of oral suctioning with 2 minutes of assisted ventilation.

49. If direct pressure with a sterile dressing fails to immediately stop severe bleeding from an extremity, you should apply: A. additional sterile dressings. B. a splint and elevate the extremity. C. a tourniquet proximal to the injury. D. digital pressure to a proximal artery.

C. a tourniquet proximal to the injury.

125. The primary prehospital treatment for most medical emergencies: A. typically does not require the EMT to contact medical control. B. focuses on definitive care because a diagnosis can usually be made. C. addresses the patient's symptoms more than the actual disease process. D. involves transport only until treatment can be performed at the hospital.

C. addresses the patient's symptoms more than the actual disease process.

85. The body's functions that occur without conscious effort are regulated by the _________ nervous system. A. sensory B. somatic C. autonomic D. voluntary

C. autonomic

108. A patient has fractured both femurs. Anatomically, these injuries would be described as being: A. medial. B. proximal. C. bilateral. D. unilateral.

C. bilateral.

38. Pulmonary edema and impaired ventilation occur during: A. septic shock. B. neurogenic shock. C. cardiogenic shock. D. anaphylactic shock.

C. cardiogenic shock.

80. Coordination of balance and body movement is controlled by the: A. medulla. B. cerebrum. C. cerebellum. D. brain stem.

C. cerebellum.

27. The jaw-thrust maneuver is used to open the airway of patients with suspected: A. mandibular fractures. B. upper airway swelling. C. cervical spine injuries. D. copious oral secretions

C. cervical spine injuries.

90. Contraction or tensing of the abdominal muscles in an effort to ease pain is called: A. flexing. B. referring. C. guarding. D. withdrawing.

C. guarding.

31. The MOST serious complication associated with using a nasopharyngeal airway in a patient with trauma to the head or face is: A. fracturing the septum. B. damaging the turbinates. C. penetrating the cranium. D. causing severe bleeding.

C. penetrating the cranium.

72. The white portion of the eye is called the: A. iris. B. retina. C. sclera. D. cornea.

C. sclera.

141. In contrast to the sympathetic nervous system, the parasympathetic nervous system: A. prepares the body to handle stress. B. causes an increase in the heart rate. C. slows the heart and respiratory rates. D. dilates the blood vessels in the muscles.

C. slows the heart and respiratory rates.

121. Which of the following conditions is NOT categorized as a psychiatric condition? A. depression B. schizophrenia C. substance abuse D. Alzheimer's disease

C. substance abuse

A man was found unresponsive in his bed at home. There is no evidence of injury, and the patient's medical history is not known. The patient's face is cherry red, yet the pulse oximeter reads 98%. Which of the following would MOST likely explain this? Carbon monoxide poisoning Severe pulmonary edema Increased body temperature Cold extremities

Carbon monoxide poisoning

According to the "E" in the DOPE mnemonic, which of the following actions should you perform to troubleshoot inadequate ventilation in a patient with a tracheostomy tube? Check the mechanical ventilator for malfunction. Attempt to pass a suction catheter into the tube. Look for blood or other secretions in the tube. Listen to breath sounds to assess for a pneumothorax.

Check the mechanical ventilator for malfunction.

Which of the following conditions would MOST likely lead to pelvic inflammatory disease if left untreated? Chlamydia Ovarian cysts Ectopic pregnancy Genital herpes

Chlamydia

Minimum airway and ventilation equipment that should be carried on every ambulance include all of the following, except: Combitubes, or laryngeal mask airways. various sizes of oral and nasal airways. mounted and portable suctioning units. adult and pediatric bag-valve masks.

Combitubes, or laryngeal mask airways.

Which of the following activities occurs in the warm zone? Personnel staging Command Medical monitoring Decontamination

Command

Which of the following statements regarding interaction with the caregiver of a child or adult with special health care needs is correct? In most cases, it is more appropriate for the EMT to contact medical control prior to speaking with the patient's primary caregiver. Before performing an assessment of the patient's ABCs, the EMT should ask the caregiver about the patient's medical condition. Communication with the patient's caregiver or family members is important because they are the most familiar with the patient's condition. In general, the EMT should only speak with a certified home health care provider because he or she is the expert on the patient's illness.

Communication with the patient's caregiver or family members is important because they are the most familiar with the patient's condition.

47. Which of the following statements regarding ventricular fibrillation (V-fib) is correct? A) V-fib is a state of rapid ventricular contraction. B) Most patients in V-fib have a weak carotid pulse. C) Defibrillation is only indicated for witnessed V-fib. D) V-fib results in an absence of forward blood flow.

D) V-fib results in an absence of forward blood flow.

191. As many as 40% of patients with Down syndrome have: A) diabetes mellitus. B) intracranial bleeding. C) unilateral paralysis. D) a heart condition.

D) a heart condition.

150. The term "supraventricular tachycardia" means: A) a slow heart rate that originates from within the ventricles. B) a rapid heart rate that originates from within the ventricles. C) a slow heart rate that originates from above the ventricles. D) a rapid heart rate that originates from above the ventricles.

D) a rapid heart rate that originates from above the ventricles.

162. It is MOST important to determine a patient's weight when asking questions pertaining to a toxic ingestion because: A) you may need additional lifting and moving assistance. B) this will allow you to determine if the exposure is lethal. C) naloxone is contraindicated if the patient is very obese. D) activated charcoal is given based on a patient's weight.

D) activated charcoal is given based on a patient's weight.

187. The MOST ominous sign of impending cardiopulmonary arrest in infants and children is: A) pallor. B) retractions. C) nasal flaring. D) bradycardia.

D) bradycardia.

182. The disruption of a joint in which the bone ends are no longer in contact is called a: A) strain. B) sprain. C) fracture. D) dislocation.

D) dislocation.

146. The EMT certification exam is designed to: A) rank EMTs based on performance on the certification exam. B) identify those EMTs who are prepared for advanced levels of training. C) provide EMTs with the best possible wage once certification is achieved. D) ensure that EMTs are competent and have the same level of knowledge and skills.

D) ensure that EMTs are competent and have the same level of knowledge and skills.

165. When documenting a call in which a female was sexually assaulted, you should: A) only use quotation marks when recording any statements made by witnesses. B) translate the patient's words or statements using proper medical terminology. C) record your opinion only if you have reasonable proof to justify the statement. D) keep the report concise and record only what the patient stated in her own words.

D) keep the report concise and record only what the patient stated in her own words.

119. Enlargement of the liver is called: A) nephritis. B) hepatomegaly. C) hydrocephalus. D) pneumonitis.

D) pneumonitis.

60. When assessing for fluid collection in the lungs during auscultation of lung sounds, you should: A) note the presence of a high-pitched whistling sound, which is an indicator of fluid in the lungs. B) pay special attention to the exhalation phase because this is when you will likely hear rales or rhonchi. C) auscultate the posterior chest first and compare the apex of one lung to the base of the opposite lung. D) start at the lower lung fields and determine at which level you start hearing clear breath sounds.

D) start at the lower lung fields and determine at which level you start hearing clear breath sounds.

173. A 30-year-old female was robbed and assaulted as she was leaving a nightclub. She has massive facial trauma and slow, gurgling respirations. As your partner manually stabilizes her head, you should: A) begin immediate ventilatory assistance. B) visualize her mouth for obvious wounds. C) apply oxygen via a nonrebreathing mask. D) suction her oropharynx for 15 seconds.

D) suction her oropharynx for 15 seconds

176. Common signs of a skull fracture include all of the following, EXCEPT: A) mastoid process bruising. B) ecchymosis around the eyes. C) noted deformity to the skull. D) superficial scalp lacerations.

D) superficial scalp lacerations.

194. You are triaging patients at the scene of a multiple-vehicle crash when you encounter a young male who is unresponsive and is not breathing. After you open his airway, he begins to breathe at a rapid rate. According to the START triage system, you should: A) assist his ventilations with a bag-valve mask and perform a rapid scan of his entire body. B) move him to the treatment area so he can receive a more comprehensive assessment of his status. C) move to the other patients, but reassess him in 5 minutes to determine if he is still breathing. D) tag him as immediate (red), place him in the recovery position, and move to the next patient.

D) tag him as immediate (red), place him in the recovery position, and move to the next patient.

132. Your BEST protection against legal liability when a patient with decision-making capacity refuses EMS care and transport is to: A) advise medical control of the situation. B) err on the side of caution and transport. C) ensure that the family is aware of the risks. D) thoroughly document the entire event.

D) thoroughly document the entire event.

117. In two-rescuer adult CPR, you should deliver a compression to ventilation ratio of: A. 5:1. B. 5:2. C. 15:2. D. 30:2.

D. 30:2.

112. After ________ minutes without oxygen, brain damage is likely. A. 2 B. 4 C. 5 D. 6

D. 6

21. Which of the following pupillary changes would indicate depressed brain function? A. Both pupils dilate when a bright light is removed. B. Both pupils constrict when a bright light is introduced. C. Both pupils react briskly to light instead of sluggishly. D. Both pupils dilate with introduction of a bright light.

D. Both pupils dilate with introduction of a bright light.

44. Which of the following statements regarding anaphylactic shock is MOST correct? A. Anaphylactic shock occurs immediately after a person is sensitized to an allergen. B. Sensitized people will experience less severe reactions upon subsequent exposure. C. Anaphylactic shock is the result of immune system failure due to a toxic exposure. D. Each subsequent exposure following sensitization often produces a more severe reaction.

D. Each subsequent exposure following sensitization often produces a more severe reaction.

139. The ability of cardiac muscle cells to contract spontaneously without a stimulus from a nerve source is called: A. excitability. B. contractility. C. impulsivity. D. automaticity

D. automaticity.

62. Perfusion is MOST accurately defined as the: A. effective transfer of oxygen from the venules across the systemic capillary membrane walls. B. ability of the systemic arteries to constrict as needed to maintain an adequate blood pressure. C. effective removal of carbon dioxide and other metabolic waste products from the body's cells. D. circulation of blood within an organ in adequate amounts to meet the body's metabolic needs.

D. circulation of blood within an organ in adequate amounts to meet the body's metabolic needs.

142. The myocardium receives oxygenated blood from the __________, which originate(s) from the __________. A. coronary sinus, vena cava B. aorta, inferior vena cava C. vena cava, coronary veins D. coronary arteries, aorta

D. coronary arteries, aorta

5. Upon arriving at a potentially unsafe scene, you should: A. remove all bystanders. B. request another ambulance. C. move the patient to safety. D. ensure that you are safe.

D. ensure that you are safe.

106. What type of consent is involved when a 39-year-old mentally competent female with a severe headache asks you to take her to the hospital? A. formal B. implied C. informed D. expressed

D. expressed

109. The brain connects to the spinal cord through a large opening at the base of the skull called the: A. foramen ovale. B. vertebral foramen. C. spinous foramen. D. foramen magnum.

D. foramen magnum.

100. To minimize the risk of injuring yourself when lifting or moving a patient, you should: A. flex at the waist instead of the hips. B. avoid the use of log rolls or body drags. C. use a direct carry whenever possible. D. keep the weight as close to your body as possible

D. keep the weight as close to your body as possible.

96. Bones are connected to other bones by bands of tough fibrous tissues called: A. bursa. B. tendons. C. cartilage. D. ligaments.

D. ligaments

83. Which of the following nerves carry information from the body to the brain via the spinal? A. motor B. central C. somatic D. sensory

D. sensory

134. When assessing for fluid collection in the lungs during auscultation of lung sounds, you should: A. note the presence of a high-pitched whistling sound, which is an indicator of fluid in the lungs. B. pay special attention to the exhalation phase since this is when you will likely hear rales or rhonchi. C. auscultate the posterior chest first and compare the apex of one lung to the base of the opposite lung. D. start at the lower lung fields and determine at which level you start hearing clear breath sounds.

D. start at the lower lung fields and determine at which level you start hearing clear breath sounds.

Which comes first in EMS decision making? Planning Data gathering Team communication Data interpretation

Data gathering

Why does the incidence of diabetes mellitus increase with age? Decreased physical activity, increased weight gain, and decreased insulin production Decreased physical activity, increased weight gain, and decreased blood sugar levels Decreased food intake, decreased weight gain, and decreased blood sugar levels Increased physical activity, increased food intake, and increased insulin production

Decreased physical activity, increased weight gain, and decreased insulin production

Which of the following is NOT a characteristic of epinephrine? Secreted naturally by the adrenal glands Constricts the blood vessels Decreases heart rate and blood pressure Dilates passages in the lungs

Decreases heart rate and blood pressure

A 50-year-old man with diabetes has an altered mental status and is unable to tell you when he last ate or took his insulin. Your glucometer keeps malfunctioning and you are unable to determine his blood glucose level. Which of the following clinical signs would MOST likely lead you to the correct diagnosis? Restlessness and irritability Rapid and weak pulse Deep and rapid breathing Hypotension and tachycardia

Deep and rapid breathing

Which of the following medications blocks the release of histamines? Albuterol (Ventolin) Epinephrine (Adrenalin) Acetaminophen (Tylenol) Diphenhydramine (Benadryl)

Diphenhydramine (Benadryl)

A 66-year-old woman presents with a stabbing pain in the middle of her chest that radiates to her back. She tells you that the pain suddenly began about 30 minutes ago and has been severe since the onset. She has a history of hypertension, but admits to being noncompliant with her antihypertensive medications. When you assess her, you find that her blood pressure is significantly higher in her left arm than it is in her right arm. What do these signs and symptoms MOST likely indicate? Acute myocardial infarction Hypertensive emergency Dissecting aortic aneurysm Unstable angina

Dissecting aortic aneurysm

Which is the most appropriate method to use when moving a patient from his or her bed to a wheeled stretcher? Log roll Direct carry Draw sheet method Extremity carry

Draw sheet method

Based on current guidelines, in which of the following situations should supplemental oxygen be administered? Signs of myocardial infarction and an oxygen saturation of 97% Exposure to carbon monoxide and an oxygen saturation of 94% Any elderly patient whose oxygen saturation is less than 95% Any diabetic patient whose oxygen saturation is less than 98%

Exposure to carbon monoxide and an oxygen saturation of 94%

You suspect that a pregnant 16-year-old girl has a broken leg after she was hit by a car. You explain that you plan to splint her leg, and she agrees to treatment. Which of the following types of consent describes her agreement? Minor's Informed Implied Expressed

Expressed

A 56-year-old female is found supine in a narrow hallway of her mobile home. She complains of severe weakness and dizziness, and states that she is unable to walk. There is no evidence of trauma, and the patient states that she did not fall. How should you and your partner move this patient to a more spacious area? Emergency move Direct carry Scoop stretcher Extremity lift

Extremity lift

In which of the following situations would external bleeding be the most difficult to control? Antecubital vein laceration and a blood pressure of 138/92 mm Hg Femoral artery laceration and a blood pressure of 140/90 mm Hg Jugular vein laceration and a systolic blood pressure of 90 mm Hg Carotid artery laceration and a systolic blood pressure of 60 mm Hg

Femoral artery laceration and a blood pressure of 140/90 mm Hg

Which of the following statements regarding fire ants is correct? Fire ants often bite a person repeatedly. Fire ant bites rarely cause anaphylaxis. Fire ant bites typically occur on the face. Most people are allergic to fire ant toxin.

Fire ants often bite a person repeatedly.

A patient in a semi-reclined position with the head elevated to facilitate breathing is in the ___________ position. recovery prone supine Fowler

Fowler

What medication form does oral glucose come in? Suspension Liquid Gel Fine powder

Gel

Which of the following statements regarding glucose is correct? Glucose is given to patients who are suspected of being hyperglycemic. Glucose is usually administered by the EMT via the intravenous route. Glucose is a complex sugar that rapidly absorbs into the bloodstream. Glucose is a simple sugar that is readily absorbed by the cells.

Glucose is a simple sugar that is readily absorbed by the cells.

Which of the following statements regarding hepatitis A is correct? Although there is no vaccine against hepatitis A, treatment is usually successful. Hepatitis A is primarily transmitted via contact with blood or other body fluids. Hepatitis A can only be transmitted by a patient who has an acute infection. Infection with hepatitis A causes chronic illness with a high mortality rate.

Hepatitis A can only be transmitted by a patient who has an acute infection.

During your assessment of a 50-year-old male who was found unresponsive in an alley, you note that he has slow, shallow respirations; bradycardia; facial cyanosis; and pinpoint pupils. As your partner begins assisting the patient's ventilations, he directs your attention to the patient's arms, which have multiple needle tracks on them. Which of the following would most likely explain the patient's presentation? Heroin overdose Alcohol intoxication Closed-head injury Delirium tremens

Heroin overdose

You receive a call for a domestic dispute. When you arrive at the scene, you find a young male standing on the front porch of his house. You notice that an adjacent window is broken. The patient has a large body, is clenching his fists, and is yelling obscenities at you. Which of the following findings is LEAST predictive of this patient's potential for violence? His large body size His shouting of obscenities His clenched fists The broken window

His large body size

A 75-year-old woman complains of shortness of breath. Which of the following findings should alert the EMT to the possibility of a pulmonary embolism? The patient's abdomen is swollen The patient is prescribed an inhaler Frequent urinary tract infections History of deep venous thrombosis

History of deep venous thrombosis

Which of the following is not a common factor that would affect a 75-year-old patient's vital signs? Increased weight Overall health Medications Medical conditions

Increased weight

Which of the following questions would be the MOST pertinent to ask a patient who recently returned from Europe and is now ill? Will you be traveling again in the future? How much time did you spend in Europe? Is anyone else in your travel party sick? What was the purpose of your travel?

Is anyone else in your travel party sick?

How does positive-pressure ventilation affect cardiac output? It causes pressure in the chest to decrease, which increases stroke volume and cardiac output. It increases intrathoracic pressure, which decreases venous return to the heart and causes a decrease in cardiac output. It decreases intrathoracic pressure, which facilitates venous return to the heart and increases cardiac output. There is no effect on cardiac output because positive-pressure ventilation is the act of normal breathing.

It increases intrathoracic pressure, which decreases venous return to the heart and causes a decrease in cardiac output.

Ketone production is the result of: blood glucose levels higher than 120 mg/dL. rapid entry of glucose across the cell membrane. acidosis when blood glucose levels are low. Correct! fat metabolization when glucose is unavailable.

Ketone production is the result of:

An 81-year-old female fell and struck her head. You find the patient lying on her left side. She is conscious and complains of neck and upper back pain. As you are assessing her, you note that she has a severely kyphotic spine. What is the most appropriate method of immobilizing this patient? Apply a cervical collar and place her in a sitting position on the wheeled stretcher. Move her to a supine position and immobilize her with a scoop stretcher and padding. Leave her on her side and use blanket rolls to immobilize her to the long backboard. Immobilize her in a supine position on a long backboard and secure her with straps.

Leave her on her side and use blanket rolls to immobilize her to the long backboard.

Which of the following organs would most likely bleed profusely when injured? Liver Bladder Stomach Intestine

Liver

For which of the following patients is spinal immobilization clearly indicated? Woman who fell from a standing position and has a deformed shoulder Man with altered mental status after being exposed to blunt force trauma Woman in a minor motor-vehicle collision who complains of severe knee pain Man with an arrow impaled in his leg and no pulse distal to the injury

Man with altered mental status after being exposed to blunt force trauma

After a head injury, which of the following is more common in children than in adults? Loss of consciousness Nausea and vomiting Seizures and hypoxia Spinal cord injury

Nausea and vomiting

In which of the following situations would the EMT be the least likely to immobilize a patient's spine? No spinal pain, but tingling in the extremities Unresponsive, but moving all extremities equally Pain to the c-spine, but no numbness or tingling. No distracting injuries or evidence of intoxication

No distracting injuries or evidence of intoxication

A team of EMTs and paramedics are attempting to resuscitate a man who is in cardiac arrest while his wife and son are present. Which of the following should occur during the resuscitation attempt? A law enforcement officer should prepare the family for the patient's death and contact the funeral home. One EMT should update the family on the interventions that have been performed and how the patient has responded. Each member of the resuscitation team should update the family at various intervals throughout the attempt. Communication with the family should be minimal until the final outcome of the resuscitation attempt is known.

One EMT should update the family on the interventions that have been performed and how the patient has responded.

According to the START triage system, what should you do if you encounter an unresponsive patient who is not breathing? Triage the patient as "expectant" and move on. Ventilate the patient for 2 minutes and reassess. Open the airway and reassess breathing status. Assign the patient in the "immediate" category.

Open the airway and reassess breathing status.

A 60-year-old male presents with acute respiratory distress. He is conscious and alert, has pink and dry skin, and has respirations of 22 breaths/min with adequate depth. Which of the following treatments is MOST appropriate for this patient? Assisted ventilation with a bag-valve mask and a head-to-toe exam Oxygen via nonrebreathing mask and a focused secondary assessment Oxygen via a nasal cannula, vital signs, and prompt transport to the hospital Positive-pressure ventilations and immediate transport to the closest hospital

Oxygen via nonrebreathing mask and a focused secondary assessment

A tour bus has overturned, resulting in numerous patients. When you arrive, you are immediately assigned to assist in the triage process. Patient 1 is a middle-aged male with respiratory distress, chest pain, and a closed deformity to his right forearm. Patient 2 is a young female who is conscious and alert but has bilateral femur fractures and numerous abrasions to her arms and face. Patient 3 is an older woman who complains of abdominal pain and has a history of cardiovascular disease. Patient 4 is unresponsive, is not breathing, has a weak carotid pulse, and has a grossly deformed skull. What triage categories should you assign to these patients? Patient 1, immediate (red); Patient 2, minimal (green); Patient 3, delayed (yellow); Patient 4, expectant (black) Patient 1, delayed (yellow); Patient 2, delayed (yellow); Patient 3, minimal (green); Patient 4, immediate (red) Patient 1, immediate (red); Patient 2, delayed (yellow); Patient 3, immediate (red); Patient 4, expectant (black) Patient 1, delayed (yellow); Patient 2, immediate (red); Patient 3, delayed (yellow); Patient 4, immediate (red)

Patient 1, immediate (red); Patient 2, delayed (yellow); Patient 3, immediate (red); Patient 4, expectant (black)

Patients who have experienced even minor-appearing head injuries should be suspected of having a brain injury, especially if they: have minor abrasions to the head area. do not have deformities to the skull. Correct! are taking blood-thinning medications. have a history of Alzheimer disease.

Patients who have experienced even minor-appearing head injuries

Which of the following statements regarding gastrostomy (gastric) tubes is correct? Patients with a gastrostomy tube might still be at risk for aspiration. Gastrostomy tubes are used for patients who cannot digest food. Most gastrostomy tubes are temporary and are not sutured in place. Gastrostomy tubes are placed directly into the small intestine.

Patients with a gastrostomy tube might still be at risk for aspiration.

Which of the following are noticeable characteristics of a 9-month-old infant? Places objects in the mouth, pulls himself or herself up Walks without help, becomes frustrated with restrictions Responds to his or her name, crawls around efficiently Knows his or her name, can walk without any assistance

Places objects in the mouth, pulls himself or herself up

CPR is in progress on a pregnant woman. Shortly after manually displacing her uterus to the left, return of spontaneous circulation occurs. Which of the following would MOST likely explain this? Displacement of her uterus caused blood to flow backward, which increased blood flow to her heart. Pressure was relieved from her aorta and vena cava, which improved chest compression effectiveness. Displacement of her uterus allowed her lungs to expand more fully, which restored her pulse. Increased blood flow to her heart caused her ventricles to stop fibrillating, which restored her pulse.

Pressure was relieved from her aorta and vena cava, which improved chest compression effectiveness.

An intoxicated 40-year-old male is found lying face down. How would you document his body's position? Recumbent Dorsal Prone Supine

Prone

A 66-year-old woman experienced a sudden onset of difficulty breathing. She has a history of type 2 diabetes and deep vein thrombosis (DVT). On the basis of her medical history, which of the following should the EMT suspect? Severe hypoglycemia Pulmonary embolism Diabetic ketoacidosis Congestive heart failure

Pulmonary embolism

Which of the following statements regarding pulse oximetry is correct? Pulse oximetry measures the percentage of hemoglobin that is saturated with oxygen but does not measure the actual hemoglobin content of the blood. The pulse oximeter is a valuable assessment tool that measures the percentage of red blood cells that contain hemoglobin molecules. Caution must be exercised when using the pulse oximeter on a patient with carbon monoxide poisoning because falsely low readings are common. Most otherwise healthy patients can maintain adequate oxygenation and good skin color with oxygen saturation readings as low as 70% to 80%.

Pulse oximetry measures the percentage of hemoglobin that is saturated with oxygen but does not measure the actual hemoglobin content of the blood.

Which of the following statements regarding rape is correct? Rape causes more physical than emotional harm. The EMT should try to determine if rape occurred. Rape is a legal term, not a medical diagnosis. Only a licensed physician can make a diagnosis of rape.

Rape is a legal term, not a medical diagnosis.

Which of the following statements regarding the rapid extrication technique is correct? The only indication for performing a rapid extrication is if the patient is not entrapped and is in cardiac arrest. Rapid extrication involves the use of heavy equipment to disentangle a patient from his or her crashed vehicle. It involves rapidly removing a patient from his or her vehicle after immobilizing him or her with a short backboard. Rapid extrication is indicated if the scene is unsafe and the patient is not entrapped in his or her vehicle.

Rapid extrication is indicated if the scene is unsafe and the patient is not entrapped in his or her vehicle.

You are assessing a conscious 55-year-old male with a sudden change in behavior. Which of the following clinical findings would be MOST suggestive of dysfunction of this patient's central nervous system? Rapid eye movement Excessive tearing or crying An irregular pulse Consistent eye contact

Rapid eye movement

In addition to obtaining a SAMPLE history and asking questions related to the chief complaint, what else should you specifically inquire about when assessing a patient with a potentially infectious disease? Recent travel HIV status Sexual practices Drug allergies

Recent travel

Assuming that no obvious signs of intra-abdominal injury are present, which of the following injuries would most likely cause an injury to the liver or spleen to be overlooked? Femur fracture Pelvic fracture Lumbar spine fracture Shoulder fracture

Shoulder fracture

Which of the following clinical signs would necessitate the administration of naloxone (Narcan) in a suspected narcotic overdose? Hypertension Extreme agitation Tachycardia Slow respirations

Slow respirations

A 59-year-old male with a history of emphysema complains of an acute worsening of his dyspnea and pleuritic chest pain following a forceful cough. Your assessment reveals that he has a barrel-shaped chest, unilaterally diminished breath sounds, and tachycardia. What is the MOST likely cause of this patient's condition? Spontaneous pneumothorax Exacerbation of his COPD Rupture of the diaphragm Acute pulmonary embolism

Spontaneous pneumothorax

A ____________ is a musculoskeletal injury in which there is partial or temporary separation of the bone ends as well as partial stretching or tearing of the supporting ligaments. Sprain Strain Fracture Dislocation

Sprain

Which of the following statements regarding standing orders is correct? Standing orders have less legal authority than orders given via radio. Standing orders only highlight the care that you may provide. Standing orders should be followed when physician contact is not possible. Standing orders require you to contact medical control first before providing an intervention.

Standing orders should be followed when physician contact is not possible.

Which of the following statements regarding communication with a child is correct? Standing over a child often increases his or her level of anxiety. The EMT should give the child minimal information to avoid scaring him or her. Most children are intrigued by strangers wearing uniforms. Unlike adults, children cannot see through lies or deceptions.

Standing over a child often increases his or her level of anxiety.

How is nitroglycerin usually given by the EMT? Injected Inhaled Orally Sublingually

Sublingually

In which position should you restrain a physically uncooperative patient? Prone With arms and legs bound together With hands tied behind the back Supine

Supine

Which of the following is an example of a rules-based medication error? The EMT administers the correct drug, but gives it by the wrong route. The EMT accidentally gives a higher drug dose than what is indicated. The EMT administers a drug that is not approved by the medical director. The EMT administers a drug that is contraindicated for the patient.

The EMT administers a drug that is not approved by the medical director.

In which of the following situations is an emergency patient move indicated? A patient has an altered mental status or is in shock. The EMT has to gain access to lesser-injured patients in a vehicle. A significant mechanism of injury is involved. The EMT is unable to protect the patient from scene hazards.

The EMT is unable to protect the patient from scene hazards.

Which of the following statements regarding the Salmonella bacterium is correct? Refrigeration of food will prevent salmonellosis. Symptoms of salmonellosis appear within 12 hours. The Salmonella bacterium produces toxins that cause food poisoning. The Salmonella bacterium itself causes food poisoning.

The Salmonella bacterium itself causes food poisoning.

What happens when blood volume is lost from the body? The veins dilate to increase systemic perfusion. Widespread vasodilation causes blood pressure to decrease. The arteries contract to increase the blood pressure. Arterial blood is diverted to the skin and muscles.

The arteries contract to increase the blood pressure.

Which of the following statements regarding the epinephrine auto-injector is correct? The auto-injector delivers epinephrine via the subcutaneous route. The adult auto-injector delivers 0.5 to 1 mg of epinephrine. EMTs do not need physician authorization to use the auto-injector. The epinephrine auto-injector delivers a preset amount of the drug.

The epinephrine auto-injector delivers a preset amount of the drug.

Which of the following is an example of closed-loop communication? The EMT corrects the team leader, who states that chest compressions should be greater than 3 inches deep. The team leader assigns the EMT a task, and the EMT repeats the request back to the team leader. EMTs decide not to attempt resuscitation because the patient has rigor mortis and is cold to the touch. The EMT requests permission from medical control to assist a patient with his prescribed nitroglycerin.

The team leader assigns the EMT a task, and the EMT repeats the request back to the team leader.

Why do middle adults commonly experience financial concerns? The majority of middle adults still have small children who live at home with them. They are typically receiving social security and must budget with a fixed income. Most people in the middle adult age group have chronic illnesses and cannot work. They are preparing for retirement but must still manage everyday financial demands.

They are preparing for retirement but must still manage everyday financial demands.

Which of the following statements regarding toddlers and preschoolers is correct? Toddlers and preschoolers commonly experience upper respiratory infections because of the loss of passive immunity. Muscle mass and bone density decrease in toddlers and preschoolers because of increased physical activity. Toddlers and preschoolers have well-developed lung musculature, even though they have less lung tissue. The normal respiratory rate in toddlers and preschoolers is between 12 and 20 breaths/min.

Toddlers and preschoolers commonly experience upper respiratory infections because of the loss of passive immunity.

A 71-year-old female slipped on a rug and fell. She is conscious but confused and complains of severe pelvic pain. Her respirations are 22 breaths/min and her heart rate is 120 beats/min. What should you do? Transport her on her left side Gently palpate her pelvis Treat her for possible shock Transport her in a seated position

Treat her for possible shock

Which of the following would MOST likely facilitate an accurate and effective verbal handoff report at the hospital? Clearly identifying your EMS certification level. Use of a mutually agreed-upon handoff format. Providing the handoff report only to a physician. Brief pause in care to provide the verbal report.

Use of a mutually agreed-upon handoff format.

Albuterol is a generic name for: Alupent. Atrovent. Ventolin. Singulair.

Ventolin.

A 19-year-old male is unresponsive, apneic, and pulseless after being struck in the center of the chest with a softball. Based on the mechanism of injury, what most likely occurred? Collapse of both lungs due to fractured ribs that perforated the lung tissue and caused cardiac arrest Asystole secondary to massive intrathoracic hemorrhage due to traumatic rupture of the aorta Ventricular fibrillation when the impact occurred during a critical portion of the cardiac cycle Fracture of the sternum that caused a rupture of the myocardium and led to a cardiac dysrhythmia

Ventricular fibrillation when the impact occurred during a critical portion of the cardiac cycle

Which of the following clinical signs is unique to anaphylactic shock? Hypotension Dizziness Wheezing Pallor

Wheezing

When is forcible restraint permitted? Only if consent to restrain is given by a family member Only if law enforcement personnel have witnessed threatening behavior When the patient poses a significant threat to self or others Anytime that the EMT feels threatened

When the patient poses a significant threat to self or others

Which of the following questions would be least pertinent during the initial questioning of a patient who ingested a substance? How much of the substance was taken? How long ago was the substance taken? What type of substance was taken? Why was the substance ingested?

Why was the substance ingested?

Following blunt trauma to the face, a 21-year-old male complains of a severe headache and decreased ability to move his eyes. This patient's clinical presentation is most consistent with: a lacerated globe. a blow-out fracture. a ruptured eyeball. optic vessel compression.

a blow-out fracture.

EMTs are dispatched for a fall. The patient, a 16-year-old female with Down syndrome, is experiencing difficulty walking and says that her fingers "feel like needles." The EMTs should suspect: an intracranial hemorrhage. hydrocephalus. a seizure. a cervical spine injury.

a cervical spine injury.

The most reliable sign of a head injury is: a pulse that is rapid and thready. an abnormally low blood pressure. decreased sensation in the extremities. a decreased level of consciousness.

a decreased level of consciousness.

A patient who is complaining of seeing flashing lights, specks, or "floaters" in his or her field of vision has most likely experienced: conjunctivitis. acute hyphema. a blow-out fracture. a detached retina.

a detached retina.

Negative-pressure breathing involves: pushing or forcing air into the lungs. increasing airway resistance during breathing. a drop in pressure within the chest cavity. relaxing the respiratory muscles.

a drop in pressure within the chest cavity.

Acute coronary syndrome (ACS) is a term used to describe: a severe decrease in perfusion caused by changes in heart rate. a group of symptoms that are caused by myocardial ischemia. the warning signs that occur shortly before a heart attack. the exact moment that a coronary artery is completely occluded.

a group of symptoms that are caused by myocardial ischemia.

When administering naloxone (Narcan) via the intranasal route, the EMT should administer: at least 2 mL into each nostril. a half dose into each nostril. a maximum of 0.5 mL. a minimum of 2.5 mg.

a half dose into each nostril.

A 75-year-old male with type 1 diabetes presents with chest pain and a general feeling of weakness. He tells you that he took his insulin today and ate a regular meal approximately 2 hours ago. You should treat this patient as though he is experiencing: hypoglycemia hyperglycemia. an acute stroke. a heart attack.

a heart attack.

An infectious disease is MOST accurately defined as: any disease that enters the body via the bloodstream and renders the immune system nonfunctional. a disease that can be spread from one person or species to another through a number of mechanisms. a medical condition caused by the growth and spread of small, harmful organisms within the body. the invasion of the human body by a bacterium that cannot be destroyed by antibiotics or other drugs.

a medical condition caused by the growth and spread of small, harmful organisms within the body.

A partial-thickness burn involves the outer layer of skin and a portion of the: fatty layer. dermal layer. muscle fascia. epidermis.

dermal layer.

In general, medevac helicopters should be utilized when: a patient has been in cardiac arrest for more than 15 minutes and has not responded to CPR and defibrillation. ground transport to the hospital exceeds 30 to 45 minutes, even if the patient's present condition is stable. ground transport would leave your service area without an ambulance for greater than 30 minutes. a patient has a time-dependent injury or illness, and traffic conditions would cause a significant delay in definitive care.

a patient has a time-dependent injury or illness, and traffic conditions would cause a significant delay in definitive care.

Situations in which you should use the rapid extrication technique include all of the following, except: a patient who needs immediate care that requires a supine position. a patient whose condition requires immediate transport to the hospital. a patient who blocks access to another seriously injured patient. a patient who can be properly assessed while still in the vehicle.

a patient who can be properly assessed while still in the vehicle.

Compression injuries to the abdomen that occur during a motor vehicle crash are typically the result of: failure to wear seat belts. a poorly placed lap belt. rapid vehicle deceleration. airbag deployment.

a poorly placed lap belt.

Characteristics of a safe ambulance operator include: a positive attitude about the ability to tolerate other drivers. the ability to operate an ambulance at a high rate of speed. an offensive attitude about driving during an emergency call. realizing that lights and siren will be effective traffic tools.

a positive attitude about the ability to tolerate other drivers.

Botulinum is: an acute viral infection. rarely associated with death. a potent bacterial neurotoxin. a disease of the leukocytes.

a potent bacterial neurotoxin.

Continual reassessment of the scene at a suspected terrorist or weapon of mass destruction incident is most important because: bystanders might destroy the evidence. terrorists are often at the scene after an attack. weather conditions might change quickly. a secondary explosive device might detonate.

a secondary explosive device might detonate.

The term "behavioral crisis" is MOST accurately defined as: a situation in which a patient of any age exhibits agitated, violent, or uncooperative behavior. a medical illness with psychological symptoms that may lead to limited motor functioning. a period of severe depression that lasts longer than 2 weeks and cannot be controlled with medications. a sudden, violent outburst of an otherwise mentally stable person toward a family member.

a situation in which a patient of any age exhibits agitated, violent, or uncooperative behavior.

Stimulation of the parasympathetic nervous system would result in: a slower heart rate. tachycardia. vasoconstriction. a strong pulse.

a slower heart rate.

A transient ischemic attack (TIA) occurs when: a small clot in a cerebral artery causes temporary symptoms. signs and symptoms resolve spontaneously within 48 hours. medications are given to dissolve a cerebral blood clot. a small cerebral artery ruptures and causes minimal damage.

a small clot in a cerebral artery causes temporary symptoms.

An 84-year-old male fell a week ago and has been bedridden since then. Today, he presents with an altered mental status. His skin is pale and cold, and his respirations are rapid and shallow. The EMT should suspect: hypovolemic shock. a subdural hematoma. acute hyperglycemia. a systemic infection.

a systemic infection.

If direct pressure fails to immediately stop severe bleeding from an extremity, you should apply: a tourniquet proximal to the injury. digital pressure to a proximal artery. additional sterile dressings. a splint and elevate the extremity.

a tourniquet proximal to the injury.

CPR should be initiated when: the carotid pulse is very weak. rigor mortis is obvious. a valid living will is unavailable. signs of putrefaction are present.

a valid living will is unavailable.

A raised, swollen, well-defined area on the skin that is the result of an insect bite or sting is called: a pustule. purpura. urticaria. a wheal.

a wheal.

Movement or motion away from the body's midline is called: adduction. extension. abduction. flexion.

abduction.

Clinical signs of compensated shock include all of the following, EXCEPT: cool and clammy skin. rapid, shallow breathing. restlessness or anxiety. absent peripheral pulses.

absent peripheral pulses.

An infant with severe dehydration would be expected to present with: absent urine output. excessive tearing. bulging fontanelles. moist oral mucosa.

absent urine output.

The process by which medications travel through body tissues until they reach the bloodstream is called: digestion. adsorption. absorption. suspension.

absorption.

A sign of respiratory distress seen in the neck is: muscular atrophy. muscular definition. muscular twitches. accessory muscle use.

accessory muscle use.

The primary prehospital treatment for most medical emergencies: addresses the patient's symptoms more than the actual disease process. involves transport only until treatment can be performed at the hospital. focuses on definitive care because a diagnosis can usually be made. typically does not require the EMT to contact medical control.

addresses the patient's symptoms more than the actual disease process.

When administering supplemental oxygen to a hypoxemic patient with a chronic lung disease, you should: avoid positive-pressure ventilation because the majority of patients with chronic lung disease are at increased risk for lung trauma. begin with a low oxygen flow rate, even if the patient is unresponsive, because high-flow oxygen may depress his or her breathing. adjust the flow rate accordingly until you see symptom improvement, but be prepared to assist his or her ventilations. recall that most patients with chronic lung diseases are stimulated to breathe by increased carbon dioxide levels.

adjust the flow rate accordingly until you see symptom improvement, but be prepared to assist his or her ventilations.

A 26-year-old female presents with heavy vaginal bleeding. She is conscious, but restless. Her blood pressure is 84/54 mm Hg, her pulse is 120 beats/min and weak, and her respirations are 22 breaths/min with adequate depth. She tells you that she inserted a tampon about 2 hours ago. You should: assist her ventilations with a bag-valve mask, place one sterile dressing into her vagina, perform a rapid secondary assessment, and transport. administer high-flow oxygen, ask her to remove the tampon, perform a detailed secondary assessment, and transport promptly. administer high-flow oxygen, perform a detailed assessment of her vaginal area for signs of trauma, place her on her side, and transport. administer high-flow oxygen, place a sterile pad over her vagina, keep her warm, elevate her lower extremities, and transport without delay.

administer high-flow oxygen, place a sterile pad over her vagina, keep her warm, elevate her lower extremities, and transport without delay.

A 39-year-old male was struck in the head by a baseball during a game. He is confused and has slurred speech. He has a large hematoma in the center of his forehead and cannot remember the events preceding the injury. After manually stabilizing his head and assessing his airway, you should: palpate his radial pulses. administer high-flow oxygen. apply ice to the hematoma. perform a neurologic exam.

administer high-flow oxygen.

A 70-year-old man complains of a sudden onset of difficulty breathing. He has dried blood on his lips and is very anxious. His left leg is red, swollen, and painful. The EMT should: suspect severe pneumonia. position the patient supine. apply a cold pack to his leg. administer high-flow oxygen.

administer high-flow oxygen.

After being stung on the leg by a jellyfish, a man complains of severe pain to his leg, dizziness, and difficulty breathing. He has a red rash covering his trunk, and his blood pressure is 90/50 mm Hg. The EMT should: administer oxygen and epinephrine and prepare for rapid transport. apply warmth to the sting area and cover it with a dry sterile dressing. begin transport and immerse his leg in hot water to help reduce pain. remove the stingers from his leg by scraping them with a stiff object.

administer oxygen and epinephrine and prepare for rapid transport.

During your assessment of a 22-year-old male who was assaulted, you note widespread contusions and abrasions to his face, chest, and abdomen. His pulse is rapid and weak, and his skin is cool and clammy. You should: place him in a sitting position and give him oxygen. administer oxygen and prepare for rapid transport. conclude that he is experiencing intracranial bleeding. perform a focused physical exam of his abdomen.

administer oxygen and prepare for rapid transport.

A 19-year-old female was stung multiple times on the legs by fire ants. She states that she is allergic to fire ants, but does not carry her own epinephrine. The patient is conscious and alert and complains of pain to the area of the bites. Her blood pressure is 122/70 mm Hg, her pulse is 100 beats/min and strong, and her respirations are 18 breaths/min and unlabored. You should: position her legs well above the level of her heart. administer oxygen and transport her to the hospital. request a paramedic unit to administer epinephrine. advise her to see her physician as soon as possible.

administer oxygen and transport her to the hospital.

A 73-year-old man presents with a generalized rash, which he thinks may have been caused by an antibiotic that he recently began taking. He has a history of coronary artery disease, hypertension, and emphysema. He is conscious and alert, his blood pressure is 144/94 mm Hg, and his pulse is 64 beats/min and regular. You auscultate his breath sounds and hear scattered wheezing, although he is not experiencing respiratory distress. You should: administer oxygen if needed, transport the patient, and monitor him for signs of deterioration. ask him if he has epinephrine and request approval from medical control to administer it to the patient. avoid the use of epinephrine because of his cardiac history, even if his symptoms become severe. begin transport and request to administer epinephrine if his systolic blood pressure falls below 110 mm Hg.

administer oxygen if needed, transport the patient, and monitor him for signs of deterioration.

A 51-year-old female presents with a sudden onset of difficulty breathing. She is conscious and alert and able to speak in complete sentences. Her respirations are 22 breaths/min and regular. You should: administer oxygen via a nonrebreathing mask. perform a secondary assessment and then begin treatment. assist her ventilations with a bag-mask device. insert a nasal airway in case her mental status decreases.

administer oxygen via a nonrebreathing mask.

Your primary assessment of an elderly woman reveals that she is conscious and alert, but is experiencing difficulty breathing. She has a history of emphysema, hypertension, and congestive heart failure. As you assess the patient's circulatory status, you should direct your partner to: perform a head-to-toe secondary assessment. administer oxygen with the appropriate device. assess her oxygen saturation and blood pressure. retrieve the stretcher and prepare for transport.

administer oxygen with the appropriate device.

A 66-year-old female with a history of hypertension and diabetes presents with substernal chest pressure of 2 hours' duration. Her blood pressure is 140/90 mm Hg, her pulse is 100 beats/min and irregular, her respirations are 22 breaths/min, and her oxygen saturation is 92%. The patient does not have prescribed nitroglycerin, but her husband does. You should: give her one nitroglycerin and reassess her systolic blood pressure. give her high-flow oxygen, attach the AED, and transport at once. obtain a SAMPLE history and contact medical control for advice. administer oxygen, give her 324 mg of aspirin, and assess her further.

administer oxygen, give her 324 mg of aspirin, and assess her further.

Medical control gives you an order that seems inappropriate for the patient's condition. After confirming that you heard the physician correctly, you should: carry out the order and then carefully document it on the run form. advise the physician that the order is unclear and ask for clarification. state that you will not carry out the order because it is inappropriate. obtain consent from the patient and then carry out the order as usual.

advise the physician that the order is unclear and ask for clarification.

A team of EMTs is caring for a critically injured patient. The team leader advises the EMT that transport will not begin until the patient's closed forearm fracture is splinted. Utilizing the crew resource management model, the EMT should: advise the team leader that immediate transport is more important than splinting. disregard the team leader's request and contact medical control for guidance. ensure that the entire team is aware that transport will be delayed for splinting. repeat the request back to the team leader and then splint the patient's arm.

advise the team leader that immediate transport is more important than splinting.

When assessing an 80-year-old patient in shock, it is important to remember that: the patient's cardiac output is able to increase by nearly 200% in response to the decrease in perfusion. age-related changes in the cardiovascular system might make the patient less able to compensate for decreased perfusion. it is common to see a more significant increase in heart rate than what would be expected in younger adults. in older adults, it is especially common to observe a significant decrease in heart rate in response to shock.

age-related changes in the cardiovascular system might make the patient less able to compensate for decreased perfusion.

Older patients with abdominal problems may not exhibit the same pain response as younger patients because of: chronic dementia, which inhibits communication. age-related deterioration of their sensory systems. progressive deterioration of abdominal organ function. interactions of the numerous medications they take.

age-related deterioration of their sensory systems.

Interoperability, an important feature of the NIMS, refers to the ability of: the federal government to intervene during any large- or small-scale incident involving terrorism. agencies of different types or from different jurisdictions to communicate with each other. EMS systems and fire departments in the same jurisdiction to effectively work as a team. county and state law enforcement agencies to acquire information and pass it along to EMS personnel.

agencies of different types or from different jurisdictions to communicate with each other.

Subcutaneous emphysema is an indication that: blood is slowly accumulating within the tissue of the lung. at least half of one lung has completely collapsed. air is escaping into the chest wall from a damaged lung. your patient is experiencing a pericardial tamponade.

air is escaping into the chest wall from a damaged lung.

An elderly patient has fallen and hit her head. You assess her level of consciousness as unresponsive using the AVPU scale. Your initial care should focus on: gathering medical history data. airway, breathing, and circulation. providing immediate transport. obtaining baseline vital signs.

airway, breathing, and circulation.

According to the Emergency Medical Treatment and Active Labor Act (EMTALA): a healthcare facility has the right to refuse assessment and treatment to a patient, but only if his or her condition is not deemed critical. all healthcare facilities must provide a medical assessment and required treatment, regardless of the patient's ability to pay. a patient maintains the legal right to recant his or her consent to emergency treatment, even after signing in to the emergency department. all healthcare facilities are legally obligated to provide assessment and care only if the patient is critically ill or injured.

all healthcare facilities must provide a medical assessment and required treatment, regardless of the patient's ability to pay.

The foreign substance responsible for causing an allergic reaction is called a(n): allergen. antibody. histamine. leukotriene.

allergen.

Braxton-Hicks contractions are characterized by: regular contractions of progressively increasing intensity. alleviation of pain with movement or changing positions. pink or red bloody show in conjunction with the contractions. a rupture of the amniotic sac just before the contractions begin.

alleviation of pain with movement or changing positions.

During your assessment of a 20-year-old man with a severe headache and nausea, you ask him when his headache began, but he does not answer your question immediately. You should: ask him if he frequently experiences severe headaches and nausea. allow him time to think about the question and respond to it. tell him that you cannot help him unless he answers your questions. repeat your question because he probably did not hear you.

allow him time to think about the question and respond to it.

When decontaminating the back of your ambulance after a call, you should: use a bleach and water solution at a 1:2 dilution ratio to thoroughly wipe all surfaces. clean all surfaces and patient contact areas with a mixture of alcohol and water. spray the contaminated areas and then immediately wipe them dry with a towel. allow surfaces to air dry unless otherwise indicated in the product directions.

allow surfaces to air dry unless otherwise indicated in the product directions.

Pain that radiates to the right lower quadrant from the umbilical area, nausea and vomiting, and lack of appetite are MOST indicative of: pancreatitis. appendicitis. cholecystitis. gastroenteritis.

appendicitis.

Cross-contamination occurs when: an EMT has direct contact with a chemical agent at a terrorist incident. an EMT provides care to a victim after the victim has been decontaminated. two EMTs are exposed to the same agent after being decontaminated. an EMT is exposed to a victim who has not yet been decontaminated.

an EMT is exposed to a victim who has not yet been decontaminated.

An area of swelling or enlargement in a weakened arterial wall is called: a thrombus. an aneurysm. an embolism. atherosclerosis.

an aneurysm.

A 73-year-old female experienced a syncopal episode while watching TV. She is now conscious but diaphoretic, tachycardic, and hypotensive. Your assessment reveals abdominal tenderness and a pulsating mass to the left of her umbilicus. You should suspect: a strangulated bowel. acute appendicitis. myocardial infarction. an aortic aneurysm.

an aortic aneurysm.

An open pneumothorax is: an open chest wound through which air moves during breathing. a fractured rib that perforates the tissue of the lung surface. extreme pleural pressure that causes the lung to rupture. the entry of air into the pleural space from a perforated lung.

an open chest wound through which air moves during breathing.

A weapon of mass destruction is most accurately defined as: a device or agent used to destroy a specific area or region within a given geographic location. any device used for the express purpose of creating carnage to make a particular point. any agent used to bring about mass death, casualties, or massive infrastructural damage. a nuclear or chemical weapon that can be launched from one country to another country.

any agent used to bring about mass death, casualties, or massive infrastructural damage.

The ___________ of the heart is the inferior portion of the ventricles. base dorsum septum apex

apex

A 62-year-old man with a history of congestive heart failure presents with severe respiratory distress and with an oxygen saturation of 82%. When you auscultate his lungs, you hear widespread rales. He is conscious and alert, is able to follow simple commands, and can only speak in two- to three-word sentences at a time. You should: place him in a supine position and assist his ventilations with a bag-valve mask and high-flow oxygen. place him in a position of comfort, deliver oxygen via nasal cannula, and closely monitor his breathing. apply a CPAP device, monitor his blood pressure, and observe him for signs of improvement or deterioration. force fluid from his alveoli by hyperventilating him with a bag-valve mask at a rate of at least 20 breaths/min.

apply a CPAP device, monitor his blood pressure, and observe him for signs of improvement or deterioration.

An unrestrained patient is sitting in his car after an automobile crash. He is conscious and alert, has no visible trauma, and is complaining of neck and back pain. Before removing him from his car, you should: perform a detailed head-to-toe assessment and apply a cervical collar. apply a cervical collar and immobilize him with a vest-style device. slide a scoop stretcher under his buttocks and rotate him laterally. maintain manual stabilization of his head and grasp him by the clothes.

apply a cervical collar and immobilize him with a vest-style device.

When performing the rapid extrication technique to remove a patient from his or her vehicle, you should: apply a vest-style extrication device prior to moving the patient. apply a cervical collar and immobilize the patient on a short backboard. apply a cervical collar and remove the patient on a long backboard. grasp the patient by the clothing and drag him or her from the car.

apply a cervical collar and remove the patient on a long backboard.

A construction worker's arm was severed just above the elbow when a steel girder fell on it. The stump is covered with a blood-soaked towel. The patient's skin is cool, clammy, and pale. The EMT should: wrap the severed arm in a sterile dressing. administer high-flow oxygen to the patient. apply a tourniquet just below the shoulder. remove the towel and inspect the wound.

apply a tourniquet just below the shoulder.

A 20-year-old male has a large laceration to his wrist. He is holding a blood-soaked towel over the wound, but it continues to bleed rapidly. You should: apply pressure to the brachial artery. apply a tourniquet proximal to the wrist. wrap the towel with pressure bandages. administer high-flow supplemental oxygen.

apply a tourniquet proximal to the wrist.

You are dispatched to a convenience store, where the clerk sustained a laceration to the side of his neck during a robbery attempt. During your assessment, you note bright red blood spurting from the laceration. You should: apply direct pressure below the lacerated vessel. circumferentially wrap a dressing around his neck. apply pressure to the closest arterial pressure point. apply direct pressure above and below the wound.

apply direct pressure above and below the wound.

During your assessment of a 29-year-old female with significant deformity to her left elbow, you are unable to palpate a radial pulse. Your transport time to the hospital is approximately 40 minutes. You should: splint the elbow in the position of deformity and transport immediately. apply gentle manual traction in line with the limb and reassess for a pulse. make two or three attempts to restore distal circulation by manipulating the elbow. carefully straighten the injured arm and secure it with padded board splints.

apply gentle manual traction in line with the limb and reassess for a pulse.

You are assessing a young male who was stung on the leg by a scorpion. He is conscious and alert, his breathing is regular and unlabored, and his blood pressure is 122/64 mm Hg. Assessment of his leg reveals a wheal surrounded by an area of redness. He states that he had a "bad reaction" the last time he was stung by a scorpion, and carries his own epinephrine auto-injector. You should: apply high-flow oxygen, apply a chemical cold pack directly to the injection site, and transport at once. apply high-flow oxygen, obtain approval from medical control to assist him with his epinephrine, and transport. assess his ABCs and vital signs in 15 minutes and allow him to drive himself to the hospital if he remains stable. apply oxygen as needed, clean the area with soap and water or a mild antiseptic, and transport him to the hospital.

apply oxygen as needed, clean the area with soap and water or a mild antiseptic, and transport him to the hospital.

A 76-year-old male experienced sudden pain to his left thigh when he was standing in line at the grocery store. Your assessment reveals ecchymosis and deformity to the distal aspect of his left femur, just above the knee. Distal circulation and sensory and motor functions are intact. You should: bind the legs together and elevate them six feet to eight feet. flex the knee slightly and apply a formable splint. apply a traction splint to realign the deformity. apply padded board splints to both sides of the leg.

apply padded board splints to both sides of the leg.

A 54-year-old male experienced an avulsion to his penis when his foreskin got caught in the zipper of his pants. He was able to unzip his pants and remove the foreskin prior to your arrival. Your assessment reveals that he is in severe pain and that the avulsion is bleeding moderately. The most appropriate treatment for this patient includes: applying direct pressure with a dry, sterile dressing. covering the avulsion with moist, sterile dressings. requesting a paramedic to administer pain medication. administering 100% oxygen via a nonrebreathing mask.

applying direct pressure with a dry, sterile dressing.

External bleeding from an extremity can usually be controlled initially by: applying chemical ice packs. elevating the extremity. applying a tourniquet. applying direct pressure.

applying direct pressure.

During your monthly internal quality improvement (QI) meeting, you review several patient care reports (PCRs) with the staff of your EMS system. You identify the patient's name, age, and sex, and then discuss the treatment that was provided by the EMTs in the field. By taking this approach to the QI process, you: violated the patient's privacy because you should have discussed the information only with the EMTs involved. are in violation of HIPAA because you did not remove the PHI from the PCR beforehand. adequately safeguarded the patient's PHI because the cases were discussed internally. acted appropriately but must have each EMT sign a waiver stating that he or she will not discuss the cases with others.

are in violation of HIPAA because you did not remove the PHI from the PCR beforehand.

The kidneys and pancreas are called retroperitoneal organs because they: are protected by the anterior rib cage. are located behind the abdominal cavity. lie just anterior to the costovertebral angle. sit in front of the liver, spleen, and stomach.

are located behind the abdominal cavity.

Components of the Cincinnati Prehospital Stroke Scale include: arm drift, memory, and grip strength. arm drift, speech, and facial droop. speech, pupil reaction, and memory. facial droop, speech, and pupil size.

arm drift, speech, and facial droop.

The smaller vessels that carry blood away from the heart and connect the arteries to the capillaries are called the: capillary arteries. venules. vena cavae. arterioles.

arterioles.

You are assessing a 75-year-old woman with mild shortness of breath. As you are asking her questions about her chief complaint and medical history, you progressively move closer and closer to her. In doing this, it is important to remember that: it is necessary to enter an older person's intimate space because the elderly are typically hearing impaired. placing yourself in the patient's personal space is relaying to her that you can be trusted. as you physically get closer to the patient, a greater and greater sense of trust must be established. a patient's personal space should not be violated, regardless of any barriers that might hamper communication.

as you physically get closer to the patient, a greater and greater sense of trust must be established.

Despite your numerous, sincere efforts to convince a 40-year-old man to consent to EMS treatment and transport, he refuses. After explaining the potential consequences of his refusal and determining that the patient has decision-making capacity, you ask him to sign an EMS refusal form, but he refuses to do that as well. You should: sign the refusal form, include the date and time, and have your partner witness it with his or her signature. document the patient's refusal, but leave the refusal form blank because only the patient can legally sign it. ask a family member, law enforcement officer, or bystander to sign the form verifying that the patient refused to sign. advise the patient that unless he signs the refusal form, he cannot legally refuse EMS treatment or transport.

ask a family member, law enforcement officer, or bystander to sign the form verifying that the patient refused to sign.

You respond to the residence of a 55-year-old woman with a possible allergic reaction to peanuts that she ate approximately 30 minutes ago. The patient is conscious and alert, but has diffuse urticaria and the feeling that she has a lump in her throat. As your partner applies oxygen to the patient, you should: ask her if she has prescribed epinephrine. obtain a complete set of baseline vital signs. ascertain if she has a family history of allergies. ask her when her last allergic reaction occurred.

ask her if she has prescribed epinephrine.

A 4-year-old boy had an apparent seizure. He is conscious and calm and is sitting on his mother's lap. His father is sitting in a nearby chair. The child's mother suddenly begins crying uncontrollably, which causes the child to start crying. You should: give the child a favorite toy or blanket to hold onto and perform your assessment to the best of your ability. reassure the child's mother that seizures in children are very common and that there is nothing to worry about. ask the father to hold the child so you can assess him while your partner tries to calm the mother. attempt to calm the child's mother, but avoid separating her from her child because this will increase her anxiety.

ask the father to hold the child so you can assess him while your

Your unit has been dispatched to stand by at the scene of a structure fire. There are no injuries of which you are aware. Upon arriving at the scene, you should: set up a staging area where firefighters can be treated if necessary. contact medical control and apprise him or her of the situation. ask the incident commander where the ambulance should be staged. park your ambulance behind the incident commander's vehicle.

ask the incident commander where the ambulance should be staged.

When assessing for arm drift of a patient with a suspected stroke, you should: ask the patient to close his or her eyes during the assessment. ask the patient to hold his or her arms up with the palms down. observe movement of the arms for approximately 2 minutes. expect to see one arm slowly drift down to the patient's side.

ask the patient to close his or her eyes during the assessment.

When interacting with an intellectually disabled patient, the best approach is to: position yourself slightly above the patient's level to reduce his or her anxiety. approach the patient as a team to reassure him or her that you are there to help. ask your team members to wait until you can establish a rapport with the patient. speak primarily with the patient's family to establish the degree of disability.

ask your team members to wait until you can establish a rapport with the patient.

General principles for approaching a potential HazMat incident include: parking the ambulance in a location that is upwind and downhill. taking standard precautions before entering any HazMat scene. asking for wind direction from dispatch and entering downwind. maintaining a safe distance and viewing the scene with binoculars.

asking for wind direction from dispatch and entering downwind.

After your partner assumes manual in-line stabilization of the patient's head, you should: assess distal neurovascular status in the extremities. thoroughly palpate the patient's head for deformities. apply an appropriately sized rigid cervical collar. use four people to log roll the patient onto a backboard.

assess distal neurovascular status in the extremities.

A 49-year-old male presents with an acute onset of crushing chest pain and diaphoresis. You should: administer up to three doses of nitroglycerin. obtain vital signs and a SAMPLE history. assess the adequacy of his respirations. administer up to 324 mg of baby aspirin.

assess the adequacy of his respirations.

After ensuring his or her own safety, the EMT's next priority when caring for a patient with a behavioral emergency is to: assess the patient's response to his or her environment. transport the patient directly to a specialized psychiatric facility. determine the underlying cause of the problem and offer advice. diagnose the patient's problem and provide definitive treatment.

assess the patient's response to his or her environment.

A 38-year-old woman was bitten by fire ants while at the park. Your primary assessment reveals that she is semiconscious; has labored breathing; and has a rapid, thready pulse. She has a red rash on her entire body, and her face is swollen. You should: perform a rapid secondary assessment. assist her ventilations with 100% oxygen. locate the area where the fire ants bit her. place her supine with her legs elevated.

assist her ventilations with 100% oxygen.

During your assessment of a young female with nontraumatic vaginal bleeding, you note that her level of consciousness is decreased, her respirations are rapid and shallow, her skin is cool and moist, and her pulse is rapid and weak. You should: assist her ventilations with a bag-mask device. assess her blood pressure and elevate her legs. perform a rapid secondary assessment. perform a visual assessment of her vaginal area.

assist her ventilations with a bag-mask device.

After eating at a local restaurant, a 20-year-old male complains of blurred vision, difficulty speaking, and difficulty breathing. He is conscious; however, his respirations are profoundly labored and producing minimal tidal volume. You should: assist his ventilations with high-flow oxygen. apply oxygen via a nonrebreathing mask. position him supine and elevate his legs. request a paramedic to administer atropine.

assist his ventilations with high-flow oxygen.

Law enforcement has summoned you to a nightclub, where a 22-year-old female was found unconscious in an adjacent alley. Your primary assessment reveals that her respirations are rapid and shallow and her pulse is rapid and weak. She is wearing a medical alert bracelet that identifies her as an epileptic. There is an empty bottle of vodka next to the patient. You should: apply oxygen via nonrebreathing mask and transport her for a blood-alcohol test. assist ventilations, perform a rapid exam, and prepare for immediate transport. place a bite block in her mouth in case she has a seizure and transport at once. apply oxygen via a nonrebreathing mask, place her on her left side, and transport.

assist ventilations, perform a rapid exam, and prepare for immediate transport.

A construction worker fell approximately 30 feet. He is semiconscious with rapid, shallow respirations. Further assessment reveals deformity to the thoracic region of his spine. His blood pressure is 70/50 mm Hg, his pulse is 66 beats/min and weak, and his skin is warm and dry. In addition to spinal immobilization and rapid transport, the MOST appropriate treatment for this patient includes: assisted ventilation, preventing hyperthermia, and elevating his lower extremities. oxygen via nonrebreathing mask, thermal management, and elevation of his legs. assisted ventilation, thermal management, and elevation of the lower extremities. oxygen via nonrebreathing mask, blankets for warmth, and elevation of his head.

assisted ventilation, thermal management, and elevation of the lower

A 20-year-old male was accidentally shot in the right upper abdominal quadrant with an arrow during an archery contest. Prior to your arrival, the patient removed the arrow. Your assessment reveals that he is conscious and alert with stable vital signs. The entrance wound is bleeding minimally and appears to be superficial. You should: assume that the arrow injured an internal organ. clean the wound and apply a dry, sterile dressing. transport only if signs of shock begin to develop. carefully probe the wound to determine its depth.

assume that the arrow injured an internal organ.

Minimum staffing in the patient compartment of a basic life support (BLS) ambulance includes: at least one EMT. at least two EMTs. an EMT and a paramedic. an EMT and an AEMT.

at least one EMT.

Multiple people in a small town began experiencing abdominal cramps, excessive salivation and urination, and muscle twitching shortly after a small crop duster plane made several passes over the community. As you are assessing the patients, you further determine that most of them are bradycardic and have miosis. In addition to high-flow oxygen, the most appropriate treatment for these patients includes: activated charcoal and glucose. epinephrine and hyperbaric oxygen. amyl nitrate and naloxone. atropine and pralidoxime chloride.

atropine and pralidoxime chloride.

Because of the complexity of the older patient and the vagueness of his or her complaint, you should: perform a rapid assessment on all geriatric patients you treat. attempt to differentiate between chronic and acute problems. limit your physical examination to the area of pain or injury. rely exclusively on family members for the medical history.

attempt to differentiate between chronic and acute problems.

You respond to a college campus for a young male who is acting strangely. After law enforcement has secured the scene, you enter the patient's dorm room and find him sitting on the edge of the bed; he appears agitated. As you approach him, you note that he has dried blood around both nostrils. He is breathing adequately, his pulse is rapid and irregular, and his blood pressure is 200/110 mm Hg. Treatment for this patient includes: requesting a paramedic to administer naloxone (Narcan). assisting his ventilations with a bag-mask device. asking law enforcement to place handcuffs on the patient. attempting to calm him and giving him oxygen if tolerated.

attempting to calm him and giving him oxygen if tolerated.

A young female experienced a laceration to her left eyeball from flying glass when her boyfriend broke a soda bottle against a wall. There is moderate bleeding, and the patient states that she cannot see out of the injured eye. You should: carefully examine her eye and remove any foreign objects if needed. avoid applying pressure to the globe when you are covering the eye. ask her to move the injured eye to assess the integrity of the optic nerve. apply firm direct pressure to the injured eye and cover the opposite eye.

avoid applying pressure to the globe when you are covering the eye.

A man jumped from the roof of his house and landed on his feet. He complains of pain to his heels, knees, and lower back. This mechanism of injury is an example of: hyperflexion. axial loading. distraction. hyperextension.

axial loading.

Initial treatment to dislodge a severe foreign body airway obstruction in a responsive infant involves: abdominal thrusts. bag-mask ventilation. blind finger sweeps. back slaps.

back slaps.

Any radio hardware containing a transmitter and a receiver that is located in a fixed location is called a: base station. mobile radio. repeater. multiplex.

base station.

The most appropriate carrying device to use when moving a patient across rough or uneven terrain is the: wheeled stretcher. scoop stretcher. stair chair. basket stretcher.

basket stretcher.

An adult patient who is NOT experiencing difficulty breathing will: have a respiratory rate that is between 20 and 24 breaths/min. assume a position that will facilitate effective and easy breathing. exhibit an indentation above the clavicles and in between the ribs. be able to speak in complete sentences without unusual pauses.

be able to speak in complete sentences without unusual pauses.

As you enter the residence of a patient who has possibly overdosed, you should: be alert for personal hazards. look for drug paraphernalia. observe the scene for drug bottles. quickly gain access to the patient.

be alert for personal hazards.

A football player was struck by another player in the right flank area just below the posterior rib cage. He complains of severe pain and point tenderness to the area. Your assessment reveals that there is a small amount of blood in his underwear. You should be most suspicious for: external genitalia injury. a lacerated liver or spleen. a ruptured urinary bladder. blunt injury to the kidney.

blunt injury to the kidney.

An 8-year-old female with a history of asthma continues to experience severe respiratory distress despite being given multiple doses of her prescribed albuterol by her mother. She is conscious but clearly restless. Her heart rate is 130 beats/min, and her respiratory rate is 30 breaths/min. She is receiving high-flow oxygen via a nonrebreathing mask. You should: continue high-flow oxygen therapy, contact medical control, and request permission to administer more albuterol. be prepared to assist her ventilations, transport at once, and request an ALS intercept en route to the hospital. begin chest compressions if she becomes unresponsive and her heart rate falls below 80 beats/min. begin immediate ventilation assistance and ensure that you squeeze the bag forcefully to open her bronchioles.

be prepared to assist her ventilations, transport at once, and request an ALS intercept en route to the hospital.

A 38-year-old male was electrocuted while attempting to wire a house. Your assessment reveals that he is unresponsive, pulseless, and apneic. A coworker has shut off the power to the house. You should: begin CPR and transport at once. fully immobilize his spinal column. begin CPR and apply the AED. assess for entry and exit wounds.

begin CPR and apply the AED.

A 60-year-old man is found to be unresponsive, pulseless, and apneic. You should: determine if he has a valid living will. withhold CPR until he is defibrillated. start CPR and transport immediately. begin CPR until an AED is available.

begin CPR until an AED is available.

Following proper decontamination, a 30-year-old male is brought to you. He is semiconscious and has rapid, shallow respirations. A quick visual assessment reveals no obvious bleeding. You should: administer high-flow oxygen via a nonrebreathing mask. begin some form of positive-pressure ventilation. ask a firefighter what the patient was exposed to. perform a rapid assessment to locate critical injuries.

begin some form of positive-pressure ventilation.

You respond to the home of a 59-year-old man who is unconscious with slow, shallow breathing and a weak pulse. The family states that the patient has terminal brain cancer and does not wish to be resuscitated. They further state that there is a DNR order for this patient, but they are unable to locate it. You should: begin treatment and contact medical control as needed. transport the patient without providing any treatment. decide on further action once the DNR order is produced. honor the patient's wishes and withhold all treatment.

begin treatment and contact medical control as needed.

General guidelines for managing a patient with a behavioral emergency include: firmly identifying yourself as an EMS provider. placing the patient between yourself and an exit. allowing the patient to be alone if he or she wishes. being prepared to spend extra time with the patient.

being prepared to spend extra time with the patient.

By the 20th week of pregnancy, the uterus is typically at or above the level of the mother's: belly button. pubic bone. xiphoid process. superior diaphragm.

belly button.

A 21-year-old male was thrown over the handlebars of his motorcycle when he rear-ended a car that was stopped at a red light. He was wearing a helmet, which he removed prior to your arrival. He is conscious but restless and has closed deformities to both of his femurs. His skin is pale, his heart rate is rapid and weak, and his respirations are rapid and shallow. You should: splint each of his deformed femurs with air splints, elevate his lower extremities, and transport. splint each of his deformed femurs with long board splints and transport without delay. apply traction splints to both of his legs, keep him warm, and transport without delay. bind his legs together on the backboard, keep him warm, and transport without delay.

bind his legs together on the backboard, keep him warm, and transport without delay.

Carbon monoxide blocks the ability of the blood to oxygenate the body because it: binds with the hemoglobin in red blood cells. fills the alveoli in the lungs with thick secretions. destroys the number of circulating red blood cells. causes the body to expel too much carbon dioxide.

binds with the hemoglobin in red blood cells.

Death caused by shaken baby syndrome is usually the result of: bleeding in the brain. fracture of the cervical spine. intra-abdominal hemorrhage. multiple open fractures.

bleeding in the brain.

A medication with antagonistic properties is one that: enhances the effects of another medication when given in a higher dose. stimulates receptor sites and allows other chemicals to attach to them. blocks receptor sites and prevents other chemicals from attaching to them. produces a cumulative effect when mixed with the same type of medication.

blocks receptor sites and prevents other chemicals from attaching to them.

Alkalosis is a condition that occurs when: blood acidity is reduced by excessive breathing. slow, shallow breathing eliminates too much carbon dioxide. the level of carbon dioxide in the blood increases. dangerous acids accumulate in the bloodstream.

blood acidity is reduced by excessive breathing.

The term "hyphema" is defined as: blood in the anterior chamber of the eye. an acute rupture of the globe of the eye. inflammation of the iris, cornea, and lens. compression of one or both optic nerves.

blood in the anterior chamber of the eye.

An infant's blood pressure typically increases with age because: as the infant gets older, his or her blood vessels dilate. the infant's total blood volume decreases with age. blood pressure directly corresponds to body weight. his or her normal heart rate usually increases with age.

blood pressure directly corresponds to body weight.

When assessing a patient with signs and symptoms of shock, it is important to remember that: multiple fractures are the most common cause of hypovolemic shock. blood pressure may be the last measurable factor to change in shock. the patient's respirations are deep during the early stages of shock. irreversible shock often responds well to a prompt blood transfusion.

blood pressure may be the last measurable factor to change in shock.

Contraction of the right ventricle causes: closure of the mitral and aortic valves. blood to flow into the pulmonary circulation. ejection of blood into the systemic circulation. a return of blood from the pulmonary veins.

blood to flow into the pulmonary circulation.

An organ or tissue might better resist damage from hypoperfusion if the: heart rate is maintained at more than 100 beats/min. body's demand for oxygen is markedly increased. systolic arterial blood pressure is at least 60 mm Hg. body's temperature is considerably less than 98.6°F (37.0°C).

body's temperature is considerably less than 98.6°F (37.0°C).

The most basic functions of the body, such as breathing, blood pressure, and swallowing, are controlled by the: cerebrum. cerebral cortex. brain stem. cerebellum.

brain stem.

A construction worker complains of intense pain after a bag of dry powder was spilled on his arm. The EMT should: cover the exposed area with a sterile dressing and elevate his arm. brush the chemical from his arm and cover it with a sterile dressing. immediately flush the skin with clean water for 15 to 20 minutes. brush the chemical from his arm and then flush the skin with water.

brush the chemical from his arm and then flush the skin with water.

In contrast to Lyme disease, Rocky Mountain spotted fever: might be confused with rheumatoid arthritis. can cause paralysis and cardiorespiratory collapse. causes painful joint swelling after a few days or weeks. presents with flu-like symptoms and a bull's-eye rash.

can cause paralysis and cardiorespiratory collapse.

In contrast to hypoglycemia, hyperglycemia: is rapidly reversible if oral glucose is given. is a rapidly developing metabolic disturbance. commonly results in excess water retention. can only be corrected in the hospital setting.

can only be corrected in the hospital setting.

The waste products of aerobic metabolism include: uric acid and nitrogen. carbon dioxide and water. ATP and glucose. glucose and lactic acid.

carbon dioxide and water.

When assessing an elderly male who complains of nausea and generalized weakness, you find that he takes atorvastatin (Lipitor) and amlodipine (Norvasc). These medications suggest a history of: reactive airway disease. cardiovascular disease. non-insulin-dependent diabetes. bacterial infection.

cardiovascular disease.

A 50-year-old male was splashed in the eyes with radiator fluid when he was working on his car. During your assessment, he tells you that he wears soft contact lenses. You should: leave the contact lenses in place and flush his eyes with sterile water. remove the contact lenses and cover his eyes with a dry, sterile dressing. leave the contact lenses in place and cover both eyes with a dry dressing. carefully remove the contact lenses and then irrigate his eyes with saline.

carefully remove the contact lenses and then irrigate his eyes with saline.

General treatment for a woman with vaginal bleeding and shock following sexual assault includes all of the following, EXCEPT: treating external lacerations with sterile compresses. refraining from placing any dressings into the vagina. carefully removing any foreign bodies from the vagina. supplemental oxygen and keeping the patient supine.

carefully removing any foreign bodies from the vagina.

The five sections of the spinal column, in descending order, are the: thoracic, cervical, lumbar, coccygeal, and sacral. coccygeal, sacral, lumbar, thoracic, and cervical. cervical, thoracic, lumbar, sacral, and coccygeal. cervical, coccygeal, thoracic, sacral, and lumbar.

cervical, thoracic, lumbar, sacral, and coccygeal.

The respiratory distress that accompanies emphysema is caused by: massive constriction of the bronchioles. chronic stretching of the alveolar walls. repeated exposure to cigarette smoke. acute fluid accumulation in the alveoli.

chronic stretching of the alveolar walls.

Perfusion is most accurately defined as the: circulation of blood within an organ in adequate amounts to meet the body's metabolic needs. effective transfer of oxygen from the venules across the systemic capillary membrane walls. effective removal of carbon dioxide and other metabolic waste products from the body's cells. ability of the systemic arteries to constrict as needed to maintain an adequate blood pressure.

circulation of blood within an organ in adequate amounts to meet the body's metabolic needs.

If it is not possible to adequately clean your ambulance at the hospital following a call, you should: wait until the end of your shift and then disinfect the entire patient compartment. quickly wipe down all high-contact surfaces with an antibacterial solution. thoroughly wash the back of the ambulance at a local car wash or similar facility. clean the ambulance at your station in a designated area that is well ventilated.

clean the ambulance at your station in a designated area that is well ventilated.

For a do not resuscitate (DNR) order to be valid, it must: be dated within the previous 24 months. be updated a minimum of every 6 months. be signed by the local justice of the peace. clearly state the patient's medical problem.

clearly state the patient's medical problem.

A 39-year-old female experienced a severe closed head injury. She is unresponsive with her eyes slightly open; her pupils are bilaterally dilated and slow to react. In addition to managing problems with airway, breathing, and circulation, you should: close her eyes and cover them with a moist dressing. secure her eyes open so you can reassess her pupils. inspect her eyes and gently remove impaled objects. irrigate her eyes with water to prevent mucosal drying.

close her eyes and cover them with a moist dressing.

You are transporting a patient with blunt abdominal trauma. The patient is unstable and is experiencing obvious signs and symptoms of shock. Your estimated time of arrival at the hospital is less than 10 minutes. After treating the patient appropriately, you should: closely monitor him and reassess him frequently. begin documenting the call on the patient care form. forgo the hospital radio report because of his condition. perform a comprehensive secondary assessment.

closely monitor him and reassess him frequently.

Signs and symptoms of a tension pneumothorax include all of the following, except: unilaterally absent breath sounds. profound cyanosis. altered mental status. collapsed jugular veins.

collapsed jugular veins.

For patient handoff, it is important for EMTs and hospital staff to use: shared training. common goals. common language. metric-sized tools.

common language.

A 30-year-old male experienced a crushing injury when his forearm was trapped between the back of a truck and a loading dock. Upon your arrival, the man's arm has been freed. Your assessment reveals that his arm is obviously deformed and swollen and is cold and pale. Further assessment reveals an absent radial pulse. You should be most concerned that this patient has: a severe closed fracture. damage to the radial nerve. internal hemorrhage. compartment syndrome.

compartment syndrome.

Rapid deceleration of the head, such as when it impacts the windshield, causes: compression injuries or bruising to the anterior portion of the brain and stretching or tearing to the posterior portion of the brain. stretching or tearing of the anterior aspect of the brain and compression injuries or bruising to the posterior aspect of the brain. compression injuries and contusions to the anterior, posterior, and lateral aspects of the brain. primary impact to the posterior aspect of the brain, resulting in compression injuries, bruising, or torn blood vessels.

compression injuries or bruising to the anterior portion of the brain and stretching or tearing to the posterior portion of the brain.

When a warm hand is immersed in water that is 70°F (21°C), heat is transferred from the hand to the water through a process called: radiation. convection. conduction. evaporation.

conduction.

You are treating a middle-aged man with chest discomfort. He has a history of three previous heart attacks and takes nitroglycerin as needed for chest pain. You have standing orders to administer aspirin to patients with suspected cardiac-related chest pain or discomfort. While your partner is preparing to give oxygen to the patient, you should: confirm that the patient is not allergic to aspirin, give him the appropriate dose of aspirin, and document the time and dose given. assist the patient in taking one of his prescribed nitroglycerins, assess his vital signs, and give him aspirin if he is still experiencing chest discomfort. contact medical control, apprise him or her of the patient's chief complaint and vital signs, and request permission to give him aspirin. ensure that the patient's systolic blood pressure is at least 100 mm Hg because aspirin dilates the blood vessels and can cause a drop in blood pressure.

confirm that the patient is not allergic to aspirin, give him the appropriate dose of aspirin, and document the time and dose given.

You are transporting a 49-year-old male with "tearing" abdominal pain. You are approximately 30 miles away from the closest hospital. During your reassessment, you determine that the patient's condition has deteriorated significantly. You should: continue transporting and alert the receiving hospital. immediately perform a rapid physical examination. assist his ventilations with a bag-valve mask. consider requesting a rendezvous with an ALS unit.

consider requesting a rendezvous with an ALS unit.

Ethnocentrism is defined as: subconsciously forcing your cultural values onto a patient because you believe that your own values are more acceptable. understanding that people from different cultural backgrounds respond to pain and stress differently. considering your own cultural values as more important when interacting with people of a different culture. suspecting that a person has an ulterior motive based on the tone of his or her voice when answering a question.

considering your own cultural values as more important when interacting with people of a different culture.

During a resuscitation attempt, the team leader asks the EMT to ventilate the patient at a rate of 20 breaths/min, and the EMT replies, "Actually, sir, the correct ventilation rate is 10 breaths/min." This is an example of: closed-loop communication. constructive intervention. situational awareness. quality assurance monitoring.

constructive intervention.

Esophageal varices MOST commonly occur in patients who: have uncontrolled diabetes. consume a lot of alcohol. have a history of esophagitis. have weak immune systems.

consume a lot of alcohol.

A 19-year-old male complains of "not feeling right." His insulin and a syringe are on a nearby table. The patient says he thinks he took his insulin and cannot remember whether he ate. He is also unable to tell you the time or what day it is. The glucometer reads "error" after several attempts to assess his blood glucose level. You should: transport only with close, continuous monitoring en route. request a paramedic ambulance to administer IV glucose. assist him with his insulin injection and reassess him. contact medical control and administer oral glucose.

contact medical control and administer oral glucose.

While transporting a woman with diabetes, you inadvertently give her oral glucose even though her blood glucose level was high. You reassess the patient and note that her condition did not change; she remained stable. You should: notify law enforcement so they can file an incident report. contact medical control and notify them of the error. document the error and report it to your supervisor. exclude this intervention from the PCR because it did not harm the patient.

contact medical control and notify them of the error.

You are dispatched to a movie theater for a 39-year-old female with signs and symptoms of a severe allergic reaction. As you are assessing her, she pulls an epinephrine auto-injector out of her purse and hands it to you. After confirming the drug's name and expiration date, you should: contact medical control. request an ALS ambulance to administer the drug. ask her if she takes other medications. administer the drug.

contact medical control.

A 3-year-old female ingested several leaves from a plant in the living room. The child's mother is not sure what type of plant it is, stating that she bought it simply because it was pretty. After completing your primary assessment of the child, you should: administer 25 g of activated charcoal. induce vomiting with syrup of ipecac. contact the regional poison control center. immediately transport the child to the hospital.

contact the regional poison control center.

Your presence is requested by law enforcement to assess a 33-year-old female who was sexually assaulted. The patient is conscious and obviously upset. As you are talking to her, you note an impressive amount of blood on her clothes in the groin area. Her blood pressure is 98/58 mm Hg, her pulse is 130 beats/min, and her respirations are 24 breaths/min. You should: allow her to change her clothes and take a shower before you transport. visualize the vaginal area and pack the vagina with sterile dressings. arrange for a rape crisis center representative to speak with the patient. control any external bleeding, administer oxygen, and transport at once.

control any external bleeding, administer oxygen, and transport at once.

A 54-year-old male accidentally shot himself in the leg while cleaning his gun. Your assessment reveals a small entrance wound to the medial aspect of his right leg. The exit wound is on the opposite side of the leg and is actively bleeding. The patient complains of numbness and tingling in his right foot. You should: control the bleeding and cover the wound with a sterile dressing. assess distal pulses as well as sensory and motor functions. manually stabilize the leg above and below the site of injury. gently manipulate the injured leg until the numbness dissipates.

control the bleeding and cover the wound with a sterile dressing.

A construction worker fell approximately 30 feet and landed in a pile of steel rods. Your assessment reveals that he is pulseless and apneic and has a 10-foot steel rod impaled in his left leg. You should: remove the steel rod, control the bleeding, apply an automated external defibrillator, begin CPR, and transport to a trauma center. control the bleeding, carefully remove the steel rod, begin CPR, and transport as soon as possible. control the bleeding, begin CPR, stabilize the steel rod, immobilize his spine, and transport immediately. stabilize the steel rod, control the bleeding, begin CPR, and rapidly transport to a trauma center.

control the bleeding, begin CPR, stabilize the steel rod, immobilize his spine, and transport immediately.

A 2-year-old female has experienced a seizure. When you arrive at the scene, the child is conscious, crying, and clinging to her mother. Her skin is hot and moist. The mother tells you that the seizure lasted approximately 5 minutes. She further tells you that her daughter has no history of seizures but has had a recent ear infection. You should: place the child in cold water to attempt to reduce her fever. allow the mother to drive her daughter to the hospital. cool the child with tepid water and transport to the hospital. suspect that the child has meningitis and transport at once.

cool the child with tepid water and transport to the hospital.

Common signs and symptoms of severe hyperglycemia include all of the following, EXCEPT: rapid, thready pulse. warm, dry skin. cool, clammy skin. acetone breath odor.

cool, clammy skin.

Classic signs and symptoms of hypoglycemia include: cold, clammy skin; bradycardia; hunger; and deep, rapid respirations. warm, dry skin; irritability; bradycardia; and rapid respirations. warm, dry skin; hunger; abdominal pain; and deep, slow respirations. cool, clammy skin; weakness; tachycardia; and rapid respirations.

cool, clammy skin; weakness; tachycardia; and rapid respirations.

All of the following snakes are pit vipers, except for the: cottonmouth. copperhead. rattlesnake. coral snake.

coral snake.

Patients with tuberculosis pose the greatest risk for transmitting the disease when they: have a fever. vomit. are bleeding. cough.

cough.

Hemoptysis is defined as: coughing up blood. blood in the pleural space. abnormal blood clotting. vomiting blood.

coughing up blood.

A 40-year-old male was in his woodworking shop when he felt a sudden, sharp pain in his left eye. Your assessment reveals a small splinter of wood embedded in his cornea. You should: scrape the splinter away with moist, sterile gauze. cover his right eye and flush the left eye with saline. cover both of his eyes and transport to the hospital. remove the object with a cotton-tipped applicator.

cover both of his eyes and transport to the hospital.

A man has a large laceration across his lower abdominal wall and a loop of bowel is protruding from the wound. He is conscious and alert and there is minimal bleeding from the wound. You should: cover the exposed bowel with a moist sterile dressing and then secure it in place with a dry bandage. gently irrigate the exposed bowel with sterile saline and then cover the wound with a dry sterile dressing. cover the bowel with a dry sterile dressing, elevate his lower extremities, and cover him with a blanket. make one attempt to replace the bowel back into the abdomen and then cover the wound with a sterile dressing.

cover the exposed bowel with a moist sterile dressing and then secure it in place with a dry bandage.

The firm cartilaginous ring that forms the inferior portion of the larynx is called the: tracheal cartilage. costal cartilage. cricoid cartilage. thyroid cartilage.

cricoid cartilage.

A viral infection that might cause obstruction of the upper airway in a child is called: croup. epiglottitis. asthma. bronchitis.

croup.

A 49-year-old man has been removed from his overturned tanker, which was carrying a hazardous material. The tank ruptured, and he was exposed to the material. When rescue personnel bring him to the decontamination area, they note that he is unconscious and has slow, shallow breathing. They should: request that EMS personnel don standard precautions, enter the warm zone, and begin immediate treatment of the patient. defer the decontamination procedure and bring the patient directly to awaiting EMS personnel for immediate treatment. decontaminate the patient as they would any other patient and then move him to the area where EMTs are waiting. cut away all of the patient's clothing and do a rapid rinse to remove as much of the contaminating matter as they can.

cut away all of the patient's clothing and do a rapid rinse to remove as much of the contaminating matter as they can.

A common cause of shock in an infant is: a cardiac dysrhythmia. excessive tachycardia. cardiovascular disease. dehydration from vomiting and diarrhea.

dehydration from vomiting and diarrhea.

A 49-year-old male presents with confusion, sweating, and visual hallucinations. The patient's wife tells you that he is a heavy drinker and that he might have had a seizure shortly before your arrival. This patient is most likely experiencing: acute hypovolemia. alcohol intoxication. acute schizophrenia. delirium tremens.

delirium tremens.

After an advanced airway device has been inserted during two-rescuer CPR, you should: deliver one rescue breath every 6 seconds. pause compressions to deliver ventilations. increase rescue breathing to a rate of 14 breaths/min. decrease the compression rate to about 80 per minute.

deliver one rescue breath every 6 seconds.

A conscious and alert 29-year-old female with a history of asthma complains of difficulty breathing that began after her morning jog. The temperature outside is 40°F (5°C). On exam, you hear bilateral expiratory wheezing. After providing supplemental oxygen, you should: contact medical control and administer an antihistamine. determine if she has been prescribed a beta-agonist inhaler. call medical control and ask how to proceed with treatment. place her in a recumbent position to facilitate breathing.

determine if she has been prescribed a beta-agonist inhaler.

When communicating with a visually impaired patient, you should: expect the patient to have difficulty understanding. possess an in-depth knowledge of sign language. determine the degree of the patient's impairment. recall that most visually impaired patients are blind.

determine the degree of the patient's impairment.

Signs of excited delirium include: subdued behavior, crying, and suicidal thoughts. diaphoresis, tachycardia, and hallucinations. pallor, hypotension, and constricted pupils. slurred speech, bradycardia, and a high fever.

diaphoresis, tachycardia, and hallucinations.

A 29-year-old male complains of a severe headache and nausea that has gradually worsened over the past 12 hours. He is conscious, alert, and oriented and tells you that his physician diagnosed him with migraine headaches. He further tells you that he has taken numerous different medications, but none of them seems to help. His blood pressure is 132/74 mm Hg, his pulse is 110 beats/min and strong, and his respirations are 20 breaths/min. Treatment should include: placing him in a supine position and transporting with lights and siren to a stroke center. applying warm compresses to the back of his neck and transporting with lights and siren. assisting him with his migraine medication and transporting without lights and siren. dimming the lights in the back of the ambulance and transporting without lights and siren.

dimming the lights in the back of the ambulance and transporting without lights and siren.

Upon arriving at the scene of a motor vehicle crash, you find a single patient still seated in his car. There are no scene hazards. As you approach the vehicle, you note that the patient is semiconscious and has a large laceration to his forehead. You should: direct your partner to apply manual in-line support of the patient's head. apply a vest-style extrication device before attempting to move the patient. apply a cervical collar and quickly remove the patient with a clothes drag. slide a long backboard under his buttocks and lay him sideways on the board.

direct your partner to apply manual in-line support of the patient's head.

A 19-year-old female has just been extricated from her severely damaged car. She is on a long backboard and has been moved to a place of safety. As your partner maintains manual stabilization of her head, you perform a rapid assessment. The patient is unresponsive, has slow and shallow respirations, and has bilateral closed femur deformities. You should: stabilize her legs with long board splints. apply oxygen via a nonrebreathing mask. direct your partner to begin ventilatory assistance. obtain baseline vital signs and transport at once.

direct your partner to begin ventilatory assistance.

During the primary assessment of a semiconscious 70-year-old female, you should: ensure a patent airway and support ventilation as needed. ask family members if the patient has a history of stroke. immediately determine the patient's blood glucose level. insert a nasopharyngeal airway and assist ventilations.

ensure a patent airway and support ventilation as needed.

A 13-year-old child is on a home ventilator. The parents called because the mechanical ventilator is malfunctioning, and the child has increasing respiratory distress. You should: attempt to troubleshoot the mechanical ventilator problem. disconnect the ventilator and apply a tracheostomy collar. place a call to the home health agency treating this patient. reset the ventilator by unplugging it for 30 to 60 seconds.

disconnect the ventilator and apply a tracheostomy collar.

A Colles fracture involves a fracture of the: distal radius. proximal radius. radius and ulna. distal ulna.

distal radius.

Accumulation of blood in the abdominal cavity will most likely cause: distention. diffuse bruising. nausea or vomiting. referred pain.

distention.

You are dispatched to the residence of an Asian family for a child with a high fever. When you assess the child, you note that he has numerous red marks on his back. The child's parents explain that these marks represent coining acknowledge and respect this practice as a cultural belief, but advise the child's parents that it has no healing power. advise the child's parents that this is a harmful practice and is considered a form of child abuse in the United States. document this finding on your patient care report and advise the emergency department staff of what the child's parents told you. advise the emergency department physician that you believe the child was intentionally abused by his parents.

document this finding on your patient care report and advise the emergency department staff of what the child's parents told you.

After delivering your patient to the hospital, you sit down to complete the handwritten PCR. When documenting the patient's last blood pressure reading, you inadvertently write 120/60 instead of 130/70. To correct this mistake, you should: erase the error, initial it, and then write the correct data on a separate addendum. draw a single horizontal line through the error, initial it, and write the correct data next to it. leave the error on the PCR, but inform the staff of the patient's actual blood pressure. cover the error with correction fluid and then write the patient's actual blood pressure over it.

draw a single horizontal line through the error, initial it, and write the correct data next to it.

Common signs and symptoms of an allergic reaction include all of the following, EXCEPT: abdominal cramps. drying of the eyes. flushing of the skin. persistent dry cough.

drying of the eyes.

After the fetus has descended into the pelvis at the end of the third trimester, many mothers experience: mid-back pain. a bloated feeling. an urge to push. easier breathing.

easier breathing.

Breathing is often more difficult in older adults because the: elasticity of the lungs decreases. surface area of the alveoli increases. diaphragm and intercostal muscles enlarge. overall size of the airway decreases.

elasticity of the lungs decreases.

After a baby is born, it is important to: ensure that it is thoroughly dried and warmed. position it so that its head is higher than its body. cool the infant to stimulate effective breathing. immediately clamp and cut the umbilical cord.

ensure that it is thoroughly dried and warmed.

The rescue team is in the process of extricating a 40-year-old male from his truck. The patient's wife, who was uninjured in the crash, is calmly observing the extrication and asks you if her husband will be all right. You should: ensure that she is in a safe area, away from the scene. allow her to talk to her husband during the extrication. allow her to observe the extrication and keep her calm. ask her follow-up questions about the details of the crash.

ensure that she is in a safe area, away from the scene.

The leaf-shaped flap of cartilage that prevents food and liquid from entering the trachea during swallowing is called the: epiglottis. uvula. vallecula. pharynx.

epiglottis.

A 40-year-old patient without a history of seizures experiences a generalized (tonic-clonic) seizure. The LEAST likely cause of this seizure is: epilepsy. a serious infection. intracranial bleeding. a brain tumor.

epilepsy.

A 30-year-old male presents with acute shortness of breath, widespread hives, and facial swelling. He denies any past medical history and takes no medications. During your assessment, you hear wheezing over all lung fields. His blood pressure is 90/50 mm Hg, and his heart rate is 110 beats/min. In addition to giving him high-flow oxygen, the MOST important treatment for this patient is: an antihistamine. albuterol. a beta-antagonist. epinephrine.

epinephrine.

A(n) __________ fracture occurs in the growth section of a child's bone and might lead to bone growth abnormalities. metaphyseal greenstick epiphyseal diaphyseal

epiphyseal

The most important immediate treatment for patients with a head injury, regardless of severity, is to: transport to a trauma center. administer high-flow oxygen. establish an adequate airway. immobilize the entire spine.

establish an adequate airway.

As the first-arriving senior EMT at the scene of an incident, you should perform a scene size-up and then: quickly identify the walking wounded. call for additional resources. begin the triage process. establish command.

establish command.

When caring for a morbidly obese patient, you should: ask the patient if he or she knows what led to his or her obesity problem. establish his or her chief complaint and then communicate your plan to help. call for ALS backup because obese patients are at high risk for heart attack. leave him or her in a supine position, as this often facilitates breathing.

establish his or her chief complaint and then communicate your plan to help.

In an interdependent group, when one person fails: pay is withheld. that person is fired. everyone fails. management will be changed.

everyone fails.

EMTs are dispatched to a residence for an 80-year-old woman who is ill. The patient's daughter states that her mother almost fainted after going to the bathroom and that her pulse was very slow. The patient's pulse rate is 80 beats/min and irregular, and she is conscious and alert. The EMTs should suspect that the patient: has an abdominal aortic aneurysm. experienced a vasovagal response. took too much of her medication. has a gastrointestinal hemorrhage.

experienced a vasovagal response.

Anaphylaxis is MOST accurately defined as a(n): moderate allergic reaction that primarily affects the vasculature. extreme allergic reaction that may affect multiple body systems. severe allergic reaction that typically resolves without treatment. allergic reaction that causes bronchodilation and vasoconstriction.

extreme allergic reaction that may affect multiple body systems.

Each ovary produces an ovum in alternating months and releases it into the: cervical os. vagina. uterus. fallopian tube.

fallopian tube.

A blood pressure cuff that is too small for a patient's arm will give a: falsely low systolic but high diastolic reading. falsely high systolic and diastolic reading. falsely low systolic and diastolic reading. falsely high systolic but low diastolic reading.

falsely high systolic and diastolic reading.

Blood that is ejected from the right ventricle: flows into the pulmonary arteries. has a high concentration of oxygen. enters the systemic circulation. was received directly from the aorta.

flows into the pulmonary arteries.

A 27-year-old male was stabbed in the chest during a disagreement at a poker game. As you approach him, you see that a knife is impaled in his chest. Before you make physical contact with the patient, it is MOST important to: form a general impression. follow standard precautions. ask bystanders what happened. call for an ALS ambulance.

follow standard precautions.

A 5-year-old boy has fallen and has a severe deformity of the forearm near the wrist. He has possibly sustained a fracture of the forearm. Correct! distal proximal dorsal superior

forearm.

To be a great EMT, strive for: management work. the chance to replace an EMR. foundational knowledge. retraining.

foundational knowledge.

Good general communication techniques with the elderly include: using medical terms to ensure patient understanding. having at least two EMTs talk to the patient at a time. frequently asking the patient if he or she understands. explaining procedures while you are performing them.

frequently asking the patient if he or she understands.

A 52-year-old unrestrained female struck the steering wheel with her face when her truck collided with another vehicle. She has obvious swelling to her face and several dislodged teeth. A visual exam of her mouth reveals minimal bleeding. She is conscious and alert with a blood pressure of 130/80 mm Hg, a pulse of 110 beats/min, and respirations of 22 breaths/min with adequate tidal volume. You should: fully immobilize her spine, attempt to locate the dislodged teeth, suction as needed, and transport. assist ventilations with a BVM device, immobilize her spine, suction her oropharynx for 30 seconds, and transport. fully immobilize her spine, irrigate her empty tooth sockets, attempt to locate the dislodged teeth, and transport. apply oxygen via a nonrebreathing mask, suction her airway as needed, disregard the dislodged teeth, and transport.

fully immobilize her spine, attempt to locate the dislodged teeth, suction as needed, and transport.

A physiologic disorder that impairs bodily function when the body seems to be structurally normal is called a: seizure drug and alcohol abuse functional disorder traumatic brain injury

functional disorder

Complications associated with chest compressions include all of the following, EXCEPT: a fractured sternum. rib fractures. liver laceration. gastric distention.

gastric distention.

Initial signs and symptoms associated with viral hemorrhagic fevers include: acute onset of double vision. joint pain and unequal pupils. headache and sore throat. severe abdominal muscle spasms.

headache and sore throat.

Placards and labels on a storage container are intended to: give a general idea of the hazard inside that particular container. advise responders of the appropriate neutralizing measures. broadly classify chemicals as being explosive or nonexplosive. provide specific information about the chemical being carried.

give a general idea of the hazard inside that particular container.

Whenever possible, a female sexual assault victim should be: encouraged to take a shower and change her clothes. asked to provide a brief description of the perpetrator. thoroughly assessed, even if no signs of injury exist. given the option of being treated by a female EMT.

given the option of being treated by a female EMT.

The official transfer of patient care does not occur until the EMT: notifies the admitting clerk of the patient's arrival at the hospital. informs dispatch of the patient's arrival at the emergency department. gives a radio report to the receiving medical facility. gives an oral report to the emergency room physician or nurse.

gives an oral report to the emergency room physician or nurse.

Delivering a patient to the hospital involves all of the following activities, except: completing a detailed written report. informing the dispatcher of your arrival. restocking any disposable items you used. giving a verbal report to the triage clerk.

giving a verbal report to the triage clerk.

You are standing by at the scene of a hostage situation when the incident commander advises you that one of his personnel has been shot. The patient is lying supine in an open area and is not moving. As the SWAT team escorts you to the patient, you should: perform a rapid assessment and move him to a place of safety. limit your primary assessment to airway and breathing only. treat only critical injuries before moving him to a safe place. grab him by the clothes and immediately move him to safety.

grab him by the clothes and immediately move him to safety.

If hydroplaning of the ambulance occurs, the driver should: gradually slow down without jamming on the brakes. slowly pump the brakes until he or she regains vehicle control. quickly jerk the steering wheel. slowly move the steering wheel back and forth.

gradually slow down without jamming on the brakes.

You are assessing a 59-year-old male and note that his pupils are unequal. He is conscious and alert. When obtaining his medical history, it is most pertinent to ask him if he: is allergic to any medications. has a history of eye surgeries. regularly sees a family physician. noticed the change during a meal.

has a history of eye surgeries.

In general, oral glucose should be given to any patient who: has documented hypoglycemia and an absent gag reflex. is unresponsive, even in the absence of a history of diabetes. has an altered mental status and a history of diabetes. has a blood glucose level that is less than 100 mg/dL.

has an altered mental status and a history of diabetes.

A patient who is possibly experiencing a stroke is NOT eligible for thrombolytic (fibrinolytic) therapy if he or she: is older than 60 years of age. has bleeding within the brain. has had a prior heart attack. has a GCS score that is less than 8.

has bleeding within the brain.

A supervisor who has more than seven people reporting to him or her: has exceeded an effective span of control and should divide tasks and delegate the supervision of some tasks to another person. should assign a specific task to each person reporting to him or her and regularly follow up to ensure that the tasks were carried out. is more beneficial to the overall effort than a supervisor with fewer personnel because his or her team can accomplish more tasks. should regularly report to the incident commander (IC) to inform him or her of the functions that his or her team is performing.

has exceeded an effective span of control and should divide tasks and delegate the supervision of some tasks to another person.

EMS personnel would most likely be called to the residence of a patient receiving home health care when the home care provider: must confirm that a specific intervention is required. needs simple assistance in providing patient care. has recognized a change in the patient's health status. has a question that is specific to the patient's condition.

has recognized a change in the patient's health status.

An elderly patient might understate or minimize the symptoms of his or her illness because: of decreased perception of pain. the nervous system has deteriorated. of conditions such as dementia. he or she fears hospitalization.

he or she fears hospitalization.

You suspect that a 75-year-old man has internal injuries after he fell and struck his ribs and abdomen on the corner of a table. When assessing and treating an injured patient of this age, you must recall that: blood pressure is usually adequately maintained because the blood vessels of older people can contract easily. functional blood volume in patients of this age steadily increases due to increased production of red blood cells. his ability to physiologically compensate for his injury might be impaired due to an inability to increase cardiac output. it is not uncommon to observe heart rates in excess of 150 beats/min in elderly patients with internal injuries.

his ability to physiologically compensate for his injury might be impaired due to an inability to increase cardiac output.

Urticaria is the medical term for: hives. burning. swelling. a wheal.

hives.

Common signs and symptoms of heat exhaustion include all of the following, except: nausea. dry tongue and thirst. hot, dry skin. tachycardia.

hot, dry skin.

The ability of a person's cardiovascular system to compensate for blood loss is most related to: the part of the body injured. his or her baseline blood pressure. how fast his or her heart beats. how rapidly he or she bleeds.

how rapidly he or she bleeds.

Naloxone (Narcan) would reverse the effects of: diazepam (Valium). temazepam (Restoril). phenobarbital (Luminal). hydromorphone (Dilaudid).

hydromorphone (Dilaudid).

Causes of infant death that might be mistaken for SIDS include all of the following, except: child abuse. meningitis. hyperglycemia. severe infection.

hyperglycemia.

Common side effects of nitroglycerin include all of the following, EXCEPT: hypotension. bradycardia. severe headache. hypertension.

hypertension.

The MOST significant risk factor for a hemorrhagic stroke is: diabetes mellitus. hypertension. severe stress. heavy exertion.

hypertension.

You respond to a residence for a 40-year-old female who was assaulted by her husband; the scene has been secured by law enforcement. Upon your arrival, you find the patient lying supine on the floor in the kitchen. She is semiconscious with severely labored breathing. Further assessment reveals a large bruise to the left anterior chest, jugular venous distention, and unilaterally absent breath sounds. As your partner is supporting her ventilations, you should: obtain a set of baseline vital signs. immediately request ALS support. perform a focused secondary exam. insert an oropharyngeal airway.

immediately request ALS support.

After the AED has delivered a shock, the EMT should: assess for a carotid pulse. transport the patient at once. re-analyze the cardiac rhythm. immediately resume CPR.

immediately resume CPR.

A 33-year-old restrained driver of a motor vehicle crash is awake and alert, complaining only of neck pain and left leg pain. The vehicle is stable, and no hazards are present. When removing this patient from his vehicle, you should: immobilize him with a vest-style device. maintain slight traction to his neck area. apply a full leg splint prior to extrication. use the rapid extrication technique.

immobilize him with a vest-style device.

A 45-year-old male was working on his roof when he fell approximately 12 feet, landing on his feet. He is conscious and alert and complains of an ache in his lower back. He is breathing adequately and has stable vital signs. You should: obtain a Glasgow Coma Score value and give him oxygen. allow him to refuse transport if his vital signs remain stable. immobilize his spine and perform a focused secondary exam. perform a rapid head-to-toe exam and immobilize his spine.

immobilize his spine and perform a focused secondary exam.

A 17-year-old football player collided with another player and has pain to his left clavicle. He is holding his arm against his chest and refuses to move it. Your assessment reveals obvious deformity to the midshaft clavicle. After assessing distal pulse, sensory, and motor functions, you should: immobilize the injury with a sling and swathe. place a pillow under his arm and apply a sling. straighten his arm and apply a board splint. perform a rapid secondary assessment.

immobilize the injury with a sling and swathe.

During your secondary assessment of a 19-year-old female with multiple traumas, you note bilateral humeral deformities and a deformity to the left midshaft femur. Her skin is diaphoretic, and her pulse is rapid and weak. Your partner has appropriately managed her airway and is maintaining manual stabilization of her head. The most appropriate treatment for this patient includes: splinting her femur fracture with padded board splints. applying a traction splint to immobilize her femur. carefully splinting each of her deformed extremities. immobilizing her to a backboard and rapidly transporting.

immobilizing her to a backboard and rapidly transporting.

Asthma is caused by a response of the: respiratory system. immune system. cardiovascular system. endocrine system.

immune system.

Cardiogenic shock may result from all of the following, EXCEPT: increased afterload. heart attack. increased preload. poor contractility.

increased preload.

In contrast to typical wheeled ambulance stretchers, features of a bariatric stretcher include: weight capacity of up to 650 pounds. two safety rails on both sides of the stretcher. a collapsible undercarriage. increased stability from a wider wheelbase.

increased stability from a wider wheelbase.

Factors that contribute to a decline in the vital capacity of an elderly patient include all of the following, except: a loss of respiratory muscle mass. increased surface area available for air exchange. decreased residual volume. increased stiffness of the thoracic cage.

increased surface area available for air exchange.

A hypnotic drug is one that: induces sleep. prevents amnesia. increases the pulse. increases the senses.

induces sleep.

Deoxygenated blood from the abdomen, pelvis, and lower extremities is returned to the right atrium via the: inferior vena cava. superior vena cava. coronary sinus vein. common iliac vein.

inferior vena cava.

The topographic term used to describe the parts of the body that are nearer to the feet is: inferior. dorsal. superior. internal.

inferior.

A 2-month-old infant was found unresponsive in his crib by his mother. When you arrive, you determine that the infant is apneic and pulseless. His skin is pale and cold and his arms are stiff. You should: begin high-quality CPR and request an ALS ambulance. begin high-quality CPR and transport immediately. request the medical examiner to perform an autopsy. inform the child's mother that her son is deceased.

inform the child's mother that her son is deceased.

The diaphragm and intercostal muscles contract during: inhalation. exhalation. ventilation. respiration.

inhalation.

A 78-year-old female presents with an acute change in her behavior. The patient's son tells you that his mother has type 2 diabetes and was diagnosed with Alzheimer's disease 6 months ago. The patient's speech is slurred and she is not alert to her surroundings. You should: allow the patient to refuse transport if she wishes. inquire about the possibility of head trauma. conclude that the patient's blood sugar is high. transport the patient to a psychiatric facility.

inquire about the possibility of head trauma.

A 25-year-old man overdosed on heroin and is unresponsive. His breathing is slow and shallow and he is bradycardic. He has track marks on both arms. The EMT should: administer naloxone via the intranasal route. insert a nasal airway and ventilate with a bag-mask device. position the patient on his side and transport without delay. administer naloxone via the intramuscular route.

insert a nasal airway and ventilate with a bag-mask device.

You are dispatched to an apartment complex where a 21-year-old female has apparently overdosed on several narcotic medications. She is semiconscious and has slow, shallow respirations. You should: place her in the recovery position and monitor for vomiting. apply oxygen via a nonrebreathing mask and transport at once. insert a nasopharyngeal airway and begin assisted ventilation. insert an oropharyngeal airway and perform oral suctioning.

insert a nasopharyngeal airway and begin assisted ventilation.

The most critical treatment for a tension pneumothorax involves: surgically removing the portion of the lung that is damaged. placing a bulky dressing over the affected side of the chest. inserting a needle through the rib cage into the pleural space. assisting the patient's breathing with increased tidal volume.

inserting a needle through the rib cage into the pleural space.

During your primary assessment of a semiconscious 30-year-old female with closed head trauma, you note that she has slow, irregular breathing and a slow, bounding pulse. As your partner maintains manual in-line stabilization of her head, you should: perform a focused secondary assessment of the patient's head and neck. immediately place her on a long backboard and prepare for rapid transport. instruct him to assist her ventilations while you perform a rapid assessment. apply 100% oxygen via a nonrebreathing mask and obtain baseline vital signs.

instruct him to assist her ventilations while you perform a rapid assessment.

Diabetic ketoacidosis occurs when: blood glucose levels rapidly fall. the pancreas produces excess insulin. the cells rapidly metabolize glucose. insulin is not available in the body.

insulin is not available in the body.

The most common and usually the most serious ambulance crashes occur at: stop lights. railroad crossings. stop signs. intersections.

intersections.

During your assessment of a patient who experienced blunt trauma to the abdomen, you notice bruising around the umbilicus. This is a sign of: rupture of a hollow organ. intra-abdominal bleeding. a severe liver laceration. a ruptured spleen.

intra-abdominal bleeding.

Bleeding within the brain tissue itself is called a(n): subarachnoid hemorrhage. subdural hematoma. epidural hematoma. intracerebral hematoma.

intracerebral hematoma.

In contrast to simple access, complex access: is a skill commonly taught to EMTs. does not involve the breaking of glass. involves forcible entry into a vehicle. often involves simply unlocking a door.

involves forcible entry into a vehicle.

A laceration: is a jagged cut caused by a sharp object or blunt force trauma. rarely penetrates through the subcutaneous tissue to the muscle. is a sharp, smooth cut that is made by a surgical scalpel. is an injury that separates various layers of soft tissue.

is a jagged cut caused by a sharp object or blunt force trauma.

In contrast to inhalation, exhalation: is an active process caused by decreased intrathoracic pressure. is a passive process caused by increased intrathoracic pressure. requires muscular effort to effectively expel air from the lungs. occurs when the diaphragm lowers and expels air from the lungs.

is a passive process caused by increased intrathoracic pressure.

Bleeding from the nose following head trauma: should be controlled by packing the nostril with gauze. should be assumed to be caused by a fractured septum. is usually due to hypertension caused by the head injury. is a sign of a skull fracture and should not be stopped.

is a sign of a skull fracture and should not be stopped.

The scene size-up at a motor vehicle crash or other incident: should be performed by the most experienced EMT. determines who is allowed to safely enter the hot zone. is an ongoing process until the incident is terminated. is a quick visual assessment of the scene prior to entry.

is an ongoing process until the incident is terminated.

In contrast to type 1 diabetes, type 2 diabetes: is commonly diagnosed in children and young adults. is caused by resistance to insulin at the cellular level. occurs when antibodies attack insulin-producing cells. is caused by a complete lack of insulin in the body.

is caused by resistance to insulin at the cellular level.

A diabetic patient has polydipsia. This means that she: urinates frequently. is excessively thirsty. is unable to swallow. has low blood sugar.

is excessively thirsty.

In contrast to the brown recluse spider, the black widow spider: is large and has a red-orange hourglass mark on its abdomen. has a bite that is typically painless until a blister develops. has a bite that usually produces local pain but no systemic signs or symptoms. is very small and has a violin-shaped marking on its back.

is large and has a red-orange hourglass mark on its abdomen.

The secondary assessment of a medical patient: should be performed at the scene, especially if the patient is critically ill. is not practical if the patient is critically ill or your transport time is short. is typically limited to a focused exam for patients who are unconscious. should routinely include a comprehensive examination from head to toe.

is not practical if the patient is critically ill or your transport time is short.

Chronic renal failure is a condition that: occurs from conditions such as dehydration. is often caused by hypertension or diabetes. can be reversed with prompt treatment. causes dehydration from excessive urination.

is often caused by hypertension or diabetes.

A 28-year-old female patient is found to be responsive to verbal stimuli only. Her roommate states that she was recently diagnosed with type 1 diabetes and has had difficulty controlling her blood sugar level. She further tells you that the patient has been urinating excessively and has progressively worsened over the last 24 to 36 hours. On the basis of this patient's clinical presentation, you should suspect that she: has a low blood glucose level. has overdosed on her insulin. has a urinary tract infection. is significantly hyperglycemic.

is significantly hyperglycemic.

A patient should be placed in the recovery position when he or she: has experienced trauma but is breathing effectively. has a pulse but is unresponsive and breathing shallowly. is semiconscious, injured, and breathing adequately. is unresponsive, uninjured, and breathing adequately.

is unresponsive, uninjured, and breathing adequately.

A nuchal cord is defined as an umbilical cord that: has separated from the placenta. is wrapped around the baby's neck. is lacerated due to a traumatic delivery. has abnormally developed blood vessels.

is wrapped around the baby's neck.

Aggressive ambulance driving might have a negative effect on other motorists because: they move to the right or drive as close to the curb as possible. they often freeze when they see the lights in the rearview mirror. they might become enraged and attempt to run you off the road. it might not allow for their reaction time to respond to your vehicle.

it might not allow for their reaction time to respond to your vehicle.

For sweating to be an effective cooling mechanism: the body must produce at least 1 L per hour. several layers of clothing must be worn. the relative humidity must be above 90%. it must evaporate from the body.

it must evaporate from the body.

Hypotension in a child with blunt or penetrating trauma is particularly significant because: it often indicates the loss of half of his or her blood volume. it typically develops earlier in children than it does in adults. the most likely cause of the hypotension is respiratory failure. most children with hypotension die in the prehospital setting.

it often indicates the loss of half of his or her blood volume.

When a light is shone into the pupil: the opposite pupil should dilate. it should become smaller in size. it should become larger in size. both pupils should dilate together.

it should become smaller in size.

Early signs and symptoms of viral hepatitis include all of the following, EXCEPT: vomiting, fever, and fatigue. jaundice and abdominal pain. loss of appetite and a cough. pain in the muscles and joints.

jaundice and abdominal pain.

When carrying a patient on a backboard up or down stairs: carry the patient headfirst. keep your palms facing down. keep the head end elevated. keep the foot end elevated.

keep the head end elevated.

When documenting a call in which a female was sexually assaulted, you should: keep the report concise and record only what the patient stated in her own words. record your opinion only if you have reasonable proof to justify the statement. translate the patient's words or statements using proper medical terminology. only use quotation marks when recording any statements made by witnesses.

keep the report concise and record only what the patient stated in her own words.

Common duties and responsibilities of EMS personnel at the scene of a motor vehicle crash include all of the following, except: keeping bystanders at a safe distance. preparing all patients for transportation. ongoing assessment of critical patients. assigning all patients a triage category.

keeping bystanders at a safe distance.

As you and your partner report for duty, you check your ambulance and begin talking about the possibility of a terrorist attack. The most effective and appropriate way to determine the likelihood of this happening is to: ask your immediate supervisor if he or she has been watching the local news. check with local businesses to see if they have received any terrorist threats. know the current threat level issued by the Department of Homeland Security. ascertain the current situation overseas regarding the number of casualties.

know the current threat level issued by the Department of Homeland Security.

You arrive at the scene of a major motor vehicle crash. The patient, a 50-year-old female, was removed from her vehicle prior to your arrival. Bystanders who removed her state that she was not wearing a seatbelt. The patient is unresponsive, tachycardic, and diaphoretic. Your assessment reveals bilaterally clear and equal breath sounds, a midline trachea, and collapsed jugular veins. You should be most suspicious that this patient has experienced a: massive hemothorax. laceration of the aorta. tension pneumothorax.

laceration of the aorta.

A hematoma develops when: small amounts of blood leak into the epidermis. severe swelling compromises arterial circulation. large blood vessels beneath the skin are damaged. cells and small vessels in the dermis are damaged.

large blood vessels beneath the skin are damaged.

You are dispatched to a residence for a young female who was kicked in the abdomen. While en route to the scene, you should ask the dispatcher if: there are other patients involved. the patient is conscious. the severity of the injury is known. law enforcement is at the scene.

law enforcement is at the scene.

Injury to a hollow abdominal organ would MOST likely result in: leakage of contents into the abdominal cavity. pain secondary to blood in the peritoneum. profound shock due to severe internal bleeding. impairment in the blood's clotting abilities.

leakage of contents into the abdominal cavity.

A high school football player was injured during a tackle and complains of neck and upper back pain. He is conscious and alert and is breathing without difficulty. The EMT should: leave his helmet in place, but remove his shoulder pads. leave his helmet and shoulder pads in place. remove his helmet and shoulder pads. remove his helmet, but leave his shoulder pads in place.

leave his helmet and shoulder pads in place.

Most patients with abdominal pain prefer to: lie on their side with their knees drawn into the abdomen. sit fully upright because it helps relax the abdominal muscles. lie in a supine position with their knees in a flexed position. sit in a semi-Fowler position with their knees slightly bent.

lie on their side with their knees drawn into the abdomen.

The proper technique for using the power grip is to: rotate your palms down. lift with your palms up. hold the handle with your fingers. position your hands about 6″ apart.

lift with your palms up.

Bones are connected to other bones by bands of tough fibrous tissues called: bursa. tendons. cartilage. ligaments.

ligaments.

Law enforcement personnel request your assistance for a 30-year-old man who was pulled over for erratic driving. The patient became acutely violent while he was being questioned, which required one of the officers to subdue him with a Taser. When you arrive and assess the patient, you find that he is very agitated and is experiencing apparent hallucinations. His skin is flushed and diaphoretic. You should: suspect that he is acutely hypoglycemic, consider giving him one tube of oral glucose, and transport with lights and siren. limit physical contact with the patient as much as possible and avoid interrupting him if he is attempting to communicate with you. recognize that he is experiencing a complex psychiatric crisis, quickly load him into the ambulance, and transport without delay. quickly rule out any life-threatening conditions and then perform a detailed secondary assessment as he is being restrained.

limit physical contact with the patient as much as possible and avoid interrupting him if he is attempting to communicate with you.

The physical examination of a sexual assault victim should be: limited to a brief survey for life-threatening injuries. performed in the presence of at least two police officers. deferred until the patient can be evaluated by a physician. as detailed as possible so all injuries can be documented.

limited to a brief survey for life-threatening injuries.

An overdose of acetaminophen, the active ingredient in Tylenol, will most likely cause: liver damage and failure. bleeding gastric ulcers. acute kidney failure. central nervous system depression.

liver damage and failure.

A 40-year-old male presents with pain to the right upper quadrant of his abdomen. He is conscious and alert with stable vital signs. During your assessment, you note that his skin and sclera are jaundiced. You should suspect: gallbladder disease. renal insufficiency. acute pancreatitis. liver dysfunction.

liver dysfunction.

Individuals with chronic alcoholism are predisposed to intracranial bleeding and hypoglycemia secondary to abnormalities in the: liver. pancreas. brain. kidneys.

liver.

If the incident command system is already established at the scene of a terrorist attack, the EMT should: contact the EMS medical director and obtain further instructions. remain in the rehabilitation area until he or she is assigned a task. begin triaging patients and start treating the most critical ones. locate the medical staging officer to obtain his or her assignment.

locate the medical staging officer to obtain his or her assignment.

All of the following are considered to be protected health information (PHI), except: patient history. treatment rendered. assessment findings. location of the call.

location of the call.

The hypoxic drive is influenced by: high blood carbon dioxide levels. high blood oxygen levels. low blood oxygen levels. low blood carbon dioxide levels.

low blood oxygen levels.

A person who experiences a calcaneus fracture after jumping and landing on his or her feet would most likely experience an accompanying fracture of the: thoracic spine. symphysis pubis. lumbar spine. coccygeal spine.

lumbar spine.

At the scene of an automobile crash, a utility pole has been broken, and power lines are lying across the car. The patients inside the car are conscious. You should: advise the patients to carefully get out of the car. mark off a danger zone around the downed lines. remove the lines with a nonconductive object. proceed with normal extrication procedures.

mark off a danger zone around the downed lines.

In contrast to bleeding caused by external trauma to the vagina, bleeding caused by conditions such as polyps or cancer: can be controlled in the field. is typically not as severe. may be relatively painless. often presents with acute pain.

may be relatively painless.

The topographic term used to describe the location of body parts that are closer toward the midline of the body is: medial. midaxillary. midclavicular. lateral.

medial.

The EMT's scope of practice within his or her local response area is defined by the: EMS supervisor. state EMS office. local health district. medical director.

medical director.

A patient who presents with a headache, fever, confusion, and red blotches on his or her skin should be suspected of having: MERS-CoV. hepatitis. tuberculosis. meningitis.

meningitis.

Each cell of the body combines nutrients and oxygen and produces energy and waste products through a process called: oxygenation. ventilation. metabolism. respiration.

metabolism.

Placing a pregnant patient in a supine position during the third trimester of pregnancy: results in spontaneous urinary incontinence if the bladder is full. often causes hypotension secondary to cardiac compression. is recommended if the patient has severe abdominal discomfort. might decrease the amount of blood that returns to the heart.

might decrease the amount of blood that returns to the heart.

As an EMT, it is important to remember that the signs and symptoms of cumulative stress: cannot be identified and can cause health problems. are most effectively treated with medications. might not be obvious or present all the time. usually manifest suddenly and without warning.

might not be obvious or present all the time.

When a person is lying supine at the end of exhalation, the diaphragm: might rise as high as the nipple line. descends below the level of the navel. contracts and flattens inferiorly. is less prone to penetrating trauma.

might rise as high as the nipple line.

While transferring a patient to ALS staff, interference should be: optimized for care. minimized overall. transformed by coordination. kept in place.

minimized overall.

After administering 0.3 mg of epinephrine via auto-injector to a 22-year-old female with an allergic reaction, you note improvement in her breathing and resolution of her hives. However, she is still anxious and tachycardic. You should: contact medical control and obtain authorization to administer another 0.3 mg of epinephrine. transport her rapidly, because it is obvious that she is having a severe reaction to the epinephrine. monitor her closely but recall that anxiety and tachycardia are side effects of epinephrine. consider administering 0.15 mg of epinephrine to completely resolve her allergic reaction.

monitor her closely but recall that anxiety and tachycardia are side effects of epinephrine.

Angina pectoris occurs when: a coronary artery is totally occluded by plaque. myocardial oxygen supply exceeds the demand. one or more coronary arteries suddenly spasm. myocardial oxygen demand exceeds supply.

myocardial oxygen demand exceeds supply.

An acute myocardial infarction (AMI) occurs when: coronary artery dilation decreases blood flow to the heart. myocardial tissue dies secondary to an absence of oxygen. the entire left ventricle is damaged and cannot pump blood. the heart muscle progressively weakens and dysfunctions.

myocardial tissue dies secondary to an absence of oxygen.

While auscultating breath sounds of a patient who was stung multiple times by a yellow jacket, you hear bilateral wheezing over all lung fields. This indicates: rapid swelling of the upper airway tissues. a significant amount of fluid in the alveoli. narrowing of the bronchioles in the lungs. enlargement of the bronchioles in the lungs.

narrowing of the bronchioles in the lungs.

When caring for a 65-year-old male with respiratory distress, you place him in a comfortable position but do not apply oxygen. The patient's condition continues to deteriorate: He develops cardiac arrest and dies at the hospital. This scenario is an example of: battery. assault. negligence. abandonment.

negligence.

A patient who is suspected of being hypoxic and is breathing adequately should be given supplemental oxygen with a: nonrebreathing mask. nasal cannula. bag-mask device. mouth-to-mask device.

nonrebreathing mask.

Bacterial vaginosis is a condition that occurs when: numerous bacteria enter the uterus through the cervix and cause severe tissue damage. abnormal bacteria enter the vagina and cause damage without causing any symptoms. normal bacteria in the vagina are replaced by an overgrowth of other bacterial forms. harmful bacteria infect and cause damage to the uterus, cervix, and fallopian tubes.

normal bacteria in the vagina are replaced by an overgrowth of other bacterial forms.

Immediately upon arriving at the scene of an emergency call involving a traumatic injury, you should notify the dispatcher of your arrival and then: determine if additional units are needed. quickly gain access to the patient. carefully assess the mechanism of injury. observe the scene for safety hazards.

observe the scene for safety hazards.

According to Occupational Safety and Health Administration (OSHA) regulations, an employer must: guarantee a 100% risk-free environment prior to employment. offer a workplace environment that reduces the risk of exposure. screen all prospective employees for bloodborne pathogens. provide gloves and other personal protective equipment to full-time paid employees only.

offer a workplace environment that reduces the risk of exposure.

Prescription glasses do not provide adequate eye protection because they: have large, rounded lenses. do not have shatterproof lenses. are not secured with a strap. offer little or no side protection.

offer little or no side protection.

The proper depth of chest compressions on a 9-month-old infant is: one half to two thirds the diameter of the chest. one half the diameter of the chest, or about 1½ inches. one third the diameter of the chest, or about 1½ inches. two thirds the diameter of the chest, or about 2 inches.

one third the diameter of the chest, or about 1½ inches.

A patient with a pneumothorax has unilateral chest expansion. This means that: both of his lungs are expanding when he inhales. his chest and his abdomen are moving in opposite directions. both sides of his chest are moving minimally. only one side of his chest rises when he inhales.

only one side of his chest rises when he inhales.

A man finds his 59-year-old wife unconscious on the couch. He states that she takes medications for type 2 diabetes. He further tells you that his wife has been ill recently and has not eaten for the past 24 hours. Your assessment reveals that the patient is unresponsive. You should: open and maintain her airway and assess breathing. administer oral glucose between her cheek and gum. administer 100% oxygen via a nonrebreathing mask. assess for the presence of a medical identification tag.

open and maintain her airway and assess breathing.

As you approach a young male who was involved in an industrial accident, you note that his eyes are closed and that he is not moving. You can see several large contusions to his arms, a laceration to his forehead with minimal bleeding, and a closed deformity to his right leg. You should: assess his pulse for rate, regularity, and quality. open his airway and assess his breathing status. perform an immediate head-to-toe assessment. apply high-flow oxygen and assess his injuries.

open his airway and assess his breathing status.

A 40-year-old male crashed his motorcycle into a tree. He is semiconscious, has snoring respirations, and has a laceration to the forearm with minimal bleeding. You should: tilt the patient's head back and lift up on his chin. apply a pressure dressing to the patient's arm. open his airway with the jaw-thrust maneuver. apply a cervical collar and suction his airway.

open his airway with the jaw-thrust maneuver.

A 22-year-old male with a history of clinical depression called 9-1-1 and stated that he has attempted to kill himself. Your unit and law enforcement officers arrive at the scene simultaneously. You find the patient lying supine on the living room floor. He is unresponsive and cyanotic. An empty bottle of hydromorphone (Dilaudid) is found on an adjacent table. You should: ask the police to handcuff the patient for safety purposes. wait for the police to examine him before providing care. open the patient's airway and assess his respirations. provide care after determining what Dilaudid is used for.

open the patient's airway and assess his respirations.

At a very large incident, the __________ section is responsible for managing the tactical operations usually handled by the incident commander on routine EMS calls. finance operations logistics planning

operations

Hypotension, hypoventilation, and pinpoint pupils would be expected following an overdose of: ecstasy. oxycodone (Percocet). amphetamine sulfate (Benzedrine). crack cocaine.

oxycodone (Percocet).

In order for efficient pulmonary gas exchange to occur: the percentage of inhaled carbon dioxide must exceed the percentage of inhaled oxygen. oxygen and carbon dioxide must be able to freely diffuse across the alveolar-capillary membrane. the pulmonary capillaries must be completely constricted and the alveoli must be collapsed. there must be low quantities of pulmonary surfactant to allow for full alveolar expansion.

oxygen and carbon dioxide must be able to freely diffuse across the alveolar-capillary membrane.

Airborne substances should be diluted with: oxygen. syrup of ipecac. activated charcoal. an alkaline antidote.

oxygen.

A 44-year-old male sustained a laceration to his left ear during a minor car accident. Your assessment reveals minimal bleeding. Appropriate care for this injury includes: applying a tight pressure dressing. padding between the ear and the scalp. packing the ear with sterile gauze pads. covering the wound with a moist dressing.

padding between the ear and the scalp.

Early signs and symptoms of intra-abdominal bleeding include: significant hypotension. widespread ecchymosis. bruising only. pain and distention.

pain and distention.

Common signs and symptoms of AMI include all of the following, EXCEPT: shortness of breath or dyspnea. sudden unexplained sweating. pain exacerbated by breathing. irregular heartbeat.

pain exacerbated by breathing.

Signs and symptoms of an air embolism include all of the following, except: joint pain. dysphasia. pale skin. dizziness.

pale skin.

Early signs of respiratory distress in the child include: decreased LOC. bradycardia. restlessness. cyanosis.

restlessness.

When assessing a patient with abdominal pain, you should: observe for abdominal guarding, which is characterized by sudden relaxation of the abdominal muscles when palpated. visually assess the painful area of the abdomen, but avoid palpation because this could worsen his or her condition. palpate the abdomen in a clockwise direction, beginning with the quadrant after the one the patient indicates is painful. ask the patient to point to the area of pain or tenderness and assess for rebound tenderness over that specific area.

palpate the abdomen in a clockwise direction, beginning with the quadrant after the one the patient indicates is painful.

A spinal cord injury at the level of C7 would most likely result in: paralysis of all the respiratory muscles. immediate cardiac arrest. paralysis of the intercostal muscles. paralysis of the diaphragm.

paralysis of the intercostal muscles.

You have sealed the open chest wound of a 40-year-old male who was stabbed in the anterior chest. Your reassessment reveals that he is experiencing increasing respiratory distress and tachycardia, and is developing cyanosis. You should: begin rapid transport at once. partially remove the dressing. call for a paramedic ambulance. begin ventilatory assistance.

partially remove the dressing.

Burns to pediatric patients are generally considered more serious than burns to adults because: pediatric patients have more surface area relative to total body mass. most burns in children are the result of child abuse. pediatric patients have a proportionately larger volume of blood. pediatric patients are more prone to hyperthermia.

pediatric patients have more surface area relative to total body mass.

EMTs are dispatched for a patient in cardiac arrest. When they arrive, they find that the patient is pulseless, apneic, and wearing an external defibrillator vest. In this situation, the EMTs should: apply the AED pads over the vests' pads. perform CPR and leave the vest in place. quickly remove the vest and perform CPR. wait 5 minutes prior to using the AED.

perform CPR and leave the vest in place.

To evaluate hazards present at the scene and determine the number of patients, you should: interview bystanders present at the scene. use the information provided by dispatch. perform a 360° walkaround of the scene. request the fire department at all scenes.

perform a 360° walkaround of the scene.

After using the pediatric assessment triangle (PAT) to form your general impression of a sick or injured child, you should: evaluate the child's baseline vital signs. assess the child's heart rate and skin condition. perform a hands-on assessment of the ABCs. obtain a SAMPLE history from the parents.

perform a hands-on assessment of the ABCs.

Once entrance and access to the patient have been provided, you should: begin treating his or her injuries. allow extrication to commence. administer high-flow oxygen. perform a primary assessment.

perform a primary assessment.

A 2-year-old child who has no recent history of illness suddenly appears cyanotic and cannot speak after playing with a small toy. You should: visualize the child's airway. perform a blind finger sweep. give oxygen and transport at once. perform abdominal thrusts.

perform abdominal thrusts.

A 71-year-old male is semiconscious following a sudden, severe headache. There is vomitus on his face, and his respirations are slow and shallow. The EMT must immediately: apply oxygen via a nonrebreathing mask. perform oropharyngeal suctioning. insert a nasopharyngeal airway. begin assisting the patient's ventilations.

perform oropharyngeal suctioning.

During your assessment of a 6-month-old male with vomiting and diarrhea, you note that his capillary refill time is approximately 4 seconds. From this information, you should conclude that the infant's: skin temperature is abnormally cold. respiratory status is adequate. systolic blood pressure is normal. peripheral circulation is decreased.

peripheral circulation is decreased.

The MOST common and significant complication associated with an acute abdomen is: internal bleeding. high fever. peritonitis. severe pain.

peritonitis.

The bones that constitute the fingers and toes are called: carpals. metacarpals. metatarsals phalanges.

phalanges.

A 43-year-old man is experiencing a severe nosebleed. His blood pressure is 190/110 mm Hg and his heart rate is 90 beats/min and bounding. Preferred treatment for this patient includes: packing both nostrils with gauze pads until the bleeding stops. placing a rolled 4 × 4-inch dressing between his lower lip and gum. having the patient pinch his own nostrils and then lie supine. pinching the patient's nostrils and having him lean forward.

pinching the patient's nostrils and having him lean forward.

To avoid exacerbating a patient's injury, it is especially important to use extreme caution when providing positive-pressure ventilation to patients with a: myocardial contusion. pneumothorax. flail chest. cardiac tamponade.

pneumothorax.

Excessive eating caused by cellular "hunger" is called: polydipsia. polyphagia. dyspepsia. dysphasia.

polyphagia.

Patients develop septic shock secondary to: poor vessel function and severe volume loss. an infection that weakens cardiac contractions. weak vessel tone caused by nervous system damage. failure of the blood vessels to adequately dilate.

poor vessel function and severe volume loss.

When arriving at the scene of an overturned tractor-trailer rig, you note that a green cloud is being emitted from the crashed vehicle. The driver is still in the truck; he is conscious but bleeding profusely from the head. After notifying the hazardous materials team, you should: park downhill from the scene. position the ambulance upwind. ask the driver to exit the vehicle. quickly gain access to the patient.

position the ambulance upwind.

Common interventions used to stimulate spontaneous respirations in the newborn include all of the following, except: suctioning of the upper airway. thorough drying with a towel. positive-pressure ventilations. some form of tactile stimulation.

positive-pressure ventilations.

The EMT must assume that any unwitnessed water-related incident is accompanied by: alcohol intoxication. an air embolism. cold-water immersion. possible spinal injury.

possible spinal injury.

A 45-year-old female was the unrestrained passenger of a small car that rear-ended another vehicle at a moderate rate of speed. She is conscious and alert but complains of pain to both of her knees. There is visible damage to the dashboard on the passenger's side of the vehicle. In addition to fractures or dislocations of the knees, you should be most suspicious for: a thoracic spine fracture. fracture of the tibia or fibula. anterior hip dislocation. posterior hip dislocation.

posterior hip dislocation.

In relation to the chest, the back is: posterior. anterior. inferior. ventral.

posterior.

A 50-year-old male presents with altered mental status. His wife tells you that he had a "small stroke" 3 years ago but has otherwise been in good health. The patient is responsive but unable to follow commands. After administering oxygen if needed, you should: prepare for immediate transport. perform a head-to-toe assessment. inquire about his family history. repeat the primary assessment.

prepare for immediate transport.

A 59-year-old male presents with severe vomiting and diarrhea of 3 days' duration. He is confused and diaphoretic, and his radial pulses are absent. His blood pressure is 78/50 mm Hg. After applying supplemental oxygen, you should: perform a head-to-toe exam. obtain a repeat blood pressure in 5 minutes. allow him to drink plain water. prepare for immediate transport.

prepare for immediate transport.

The function of the National Incident Management System (NIMS) is to: educate city and county governments regarding foreign terrorist attacks. facilitate a standard method of incident command for natural disasters. prepare for the potential of a nuclear attack against the United States. prepare for, prevent, respond to, and recover from domestic incidents.

prepare for, prevent, respond to, and recover from domestic incidents.

You have just completed your primary assessment of a 48-year-old man with crushing chest pain and difficulty breathing. The patient has been given 324 mg of aspirin and is receiving supplemental oxygen. As you begin your secondary assessment, you note that his mental status has deteriorated and he is now bradycardic. You should: request an ALS unit to respond to the scene. insert a nasal airway and assist his breathing. prepare the patient for immediate transport. continue with your secondary assessment.

prepare the patient for immediate transport.

A 30-year-old man complains of severe pain to his right tibia after an injury that occurred the day before. The patient's leg is pale, and he is unable to move his foot. The EMT should suspect that: pressure in the fascial compartment is elevated. the nerves behind the knee are compromised. the nerves supplying the foot have been severed. a severe infection has developed in the muscle.

pressure in the fascial compartment is elevated.

Aspirin is beneficial to patients suspected of having a heart attack because it: reduces the associated chest pain. prevents the aggregation of platelets. dissolves the coronary artery clot. causes direct coronary vasodilation.

prevents the aggregation of platelets.

Factors that increase the risk for developing methicillin-resistant Staphylococcus aureus (MRSA) include: prolonged hospitalization, especially in an intensive care unit. failure to be vaccinated against any strain of hepatitis. a history of a respiratory illness within the past 6 to 8 weeks. prior exposure to Mycobacterium tuberculosis.

prolonged hospitalization, especially in an intensive care unit.

Status epilepticus is characterized by: prolonged seizures without a return of consciousness. profound tachycardia and total muscle flaccidity. generalized seizures that last less than 5 minutes. an absence seizure that is not preceded by an aura.

prolonged seizures without a return of consciousness.

Because the depth of an open abdominal wound is often difficult to determine: the EMT must perform a thorough exam. prompt transport to the hospital is essential. vital signs should be monitored frequently. the abdomen must be vigorously palpated.

prompt transport to the hospital is essential.

A 30-year-old woman with a history of alcoholism presents with severe upper abdominal pain and is vomiting large amounts of bright red blood. Her skin is cool, pale, and clammy; her heart rate is 120 beats/min and weak; and her blood pressure is 70/50 mm Hg. Your MOST immediate action should be to: give her high-flow supplemental oxygen. keep her supine and keep her warm. protect her airway from aspiration. rapidly transport her to the hospital.

protect her airway from aspiration.

Law enforcement personnel request your assistance to assess a 31-year-old female who was sexually assaulted. When you arrive at the scene, you find the patient sitting on a curb outside her apartment. She is conscious, alert, and crying. When you ask her what happened, she tells you that she does not want to be treated or transported to the hospital. She further tells you that all she wants to do is clean up. You should: defer any physical assessment so that you do not destroy potential evidence. advise her that she cannot clean herself up because this will destroy evidence. ask her if there is anyone you can contact, such as a friend or relative. provide emotional support and visually assess her for obvious trauma.

provide emotional support and visually assess her for obvious trauma.

An important aspect in the treatment of a patient with severe abdominal pain is to: administer analgesic medications to alleviate pain. encourage the patient to remain in a supine position. provide emotional support en route to the hospital. give 100% oxygen only if signs of shock are present.

provide emotional support en route to the hospital.

After delivery of a pulseless and apneic infant who has a foul odor, skin sloughing, and diffuse blistering, you should: begin full resuscitation and transport. report the case to the medical examiner. provide emotional support to the mother. dry the infant off to stimulate breathing.

provide emotional support to the mother.

A 22-year-old female patient is complaining of dyspnea and numbness and tingling in her hands and feet after an argument with her fiancé. Her respirations are 40 breaths/min. You should: position her on her left side and transport at once. request a paramedic to give her a sedative. have her breathe into a paper or plastic bag. provide reassurance and give oxygen as needed.

provide reassurance and give oxygen as needed.

A 20-year-old male was pulled from cold water by his friends. The length of his submersion is not known and was not witnessed. You perform a primary assessment and determine that the patient is apneic and has a slow, weak pulse. You should: provide rescue breathing, remove his wet clothing, immobilize his entire spine, keep him warm, and transport carefully. suction his airway for 30 seconds, provide rescue breathing, keep him warm, and transport at once. apply high-flow oxygen via a nonrebreathing mask, immobilize his spine, keep him warm, and transport rapidly. ventilate with a bag-valve mask, apply a rigid cervical collar, remove his wet clothing, and transport rapidly.

provide rescue breathing, remove his wet clothing, immobilize his entire spine, keep him warm, and transport carefully.

A man called EMS 12 hours after injuring his chest. Your assessment reveals a flail segment to the right side of the chest. The patient is experiencing respiratory distress, and his oxygen saturation is 78%. His breath sounds are equal bilaterally, and his jugular veins are normal. You should suspect: traumatic asphyxia. tension pneumothorax. pulmonary contusion. massive hemothorax.

pulmonary contusion.

Observations made when forming a general impression of a patient would include all of the following, EXCEPT: pulse strength. appearance. race and gender. level of distress.

pulse strength.

When activated, the sympathetic nervous system produces all of the following effects, except: increase in heart rate. pupillary constriction. shunting of blood to vital organs. dilation of the bronchiole smooth muscle.

pupillary constriction.

You and your partner are standing by at the scene of a residential fire when you hear the incident commander state, "We have located a victim" over the radio. You should: locate the victim and provide initial care while your partner stays with the ambulance. remain with the ambulance and wait for fire personnel to bring the victim to you. notify the hospital that you will be transporting a burn patient to their facility. immediately locate the incident commander and ask where the victim is located.

remain with the ambulance and wait for fire personnel to bring the victim to you.

You are dispatched to an apartment complex to respond to a shooting. Law enforcement personnel are present and have the suspect in custody. You find the patient lying in a narrow space between the couch and coffee table of his small apartment. He is semiconscious and has a large gunshot wound to his chest. You should: obtain permission from law enforcement before moving anything. drag the patient into a larger area so that you can begin treatment. treat the patient where he is so that you do not destroy any evidence. quickly move the coffee table so you can access and treat the patient.

quickly move the coffee table so you can access and treat the patient.

The bones of the forearm are called the: humerus and ulna. tibia and radius. radius and humerus. radius and ulna.

radius and ulna.

A utility worker was trimming branches and was electrocuted when he accidentally cut a high-power line. He fell approximately 20 feet and is lying unresponsive on the ground; the power line is lying across his chest. You should: apply insulated gloves and assume manual control of his cervical spine. rapidly assess the patient after ensuring that the power line is not live. quickly but carefully move the patient away from the power line. manually stabilize his head as your partner assesses for breathing.

rapidly assess the patient after ensuring that the power line is not live.

A 6-year-old female was riding her bicycle and struck a clothesline with her throat. She is breathing but with obvious difficulty. Your assessment reveals a crackling sensation in the soft tissues of her neck and facial cyanosis. In addition to the appropriate airway management, the intervention that will most likely improve her chance of survival is: requesting a paramedic ambulance. carefully monitoring her vital signs. rapidly transporting her to the hospital. quickly immobilizing her spinal column.

rapidly transporting her to the hospital.

An EMT's primary responsibility to the patient who has been poisoned is to: administer the appropriate antidote. recognize that a poisoning occurred. administer 25 g of activated charcoal. contact poison control immediately.

recognize that a poisoning occurred.

A 50-year-old male was stung by a honeybee approximately 15 minutes ago. He presents with respiratory distress, facial swelling, and hypotension. After placing him on oxygen and administering his epinephrine via auto-injector, you note that his breathing has improved. Additionally, his facial swelling is resolving and his blood pressure is stable. Your next action should be to: reassess his breathing and blood pressure in 15 minutes. visualize his airway to assess for oropharyngeal swelling. notify medical control of the patient's response to your treatment. record the time and dose of the injection and transport promptly.

record the time and dose of the injection and transport promptly.

Early bruising following abdominal trauma often manifests as: red areas of skin. gross distention. localized pain. dark purple marks.

red areas of skin.

A technique used to gain insight into a patient's thinking, which involves repeating in question form what the patient has said, is called: intuitive listening. reflective listening. active listening. passive listening.

reflective listening.

General guidelines for safe ambulance driving include all of the following, except: avoiding routes with heavy traffic congestion. regularly using the siren as much as possible. assuming that other drivers will not see you. avoiding one-way streets whenever possible.

regularly using the siren as much as possible.

If an incident increases in scope and complexity, the incident commander should consider: ceasing all operations until a revised plan is established. requesting a second incident commander to assist him or her. maintaining responsibility for all of the command functions. relinquishing command to someone with more experience.

relinquishing command to someone with more experience.

You respond to a call for an unknown emergency. When you arrive at the scene, the patient's husband meets you at the door and states that his wife has been depressed and has locked herself in an upstairs bedroom. He further tells you that he keeps his handgun in the bedroom. You should: go upstairs with caution and attempt to talk to the patient. ask the husband to attempt to reason with his wife. get in your ambulance and leave the scene immediately. remain in a safe place and request law enforcement.

remain in a safe place and request law enforcement.

A 19-year-old female is found unconscious by her roommate. Your primary assessment reveals that her breathing is inadequate. As you insert an oropharyngeal airway, she begins to gag violently. You should: insert the airway no further but leave it in place as a bite block. continue to insert the airway as you suction her oropharynx. select a smaller oropharyngeal airway and attempt to insert it. remove the airway and be prepared to suction her oropharynx.

remove the airway and be prepared to suction her oropharynx.

EMTs are assessing a 2-year-old child who was riding in a car seat when the vehicle struck a tree while traveling at 45 MPH. The child is conscious and appears alert. There are no obvious signs of trauma to the child, and the car seat does not appear to be damaged. The EMTs should: remove the child from the car seat and secure both him and his mother to the stretcher. leave the child in the car seat, but pad around the child's head with towels or a pillow. remove the child from the car seat and secure him to a pediatric immobilization device. secure the child in the car seat with straps and apply a pediatric-sized cervical collar.

remove the child from the car seat and secure him to a pediatric immobilization device.

General care for a patient with a tracheostomy tube includes all of the following, except: removing the tube if the area around it appears infected. maintaining the patient in a position of comfort when possible. suctioning the tube as needed to clear a thick mucous plug. ensuring adequate oxygenation and ventilation at all times.

removing the tube if the area around it appears infected.

A 19-year-old male was stung multiple times by fire ants. He is experiencing obvious signs and symptoms of anaphylactic shock. You administer high-flow oxygen and give him epinephrine via intramuscular injection. Upon reassessment, you determine that his condition has not improved. You should: repeat the epinephrine injection after consulting with medical control. request a paramedic unit that is stationed approximately 15 miles away. consider that he may actually be experiencing an acute asthma attack. transport him immediately and provide supportive care while en route.

repeat the epinephrine injection after consulting with medical control.

After receiving online orders from medical control to perform a patient care intervention, you should: perform the intervention as ordered. ask the physician to repeat the order. confirm the order in your own words. repeat the order to medical control word for word.

repeat the order to medical control word for word.

A 56-year-old male has an incomplete avulsion to his right forearm. After controlling any bleeding from the wound, you should: thoroughly irrigate the wound with sterile water and cover it with a sterile dressing. replace the avulsed flap to its original position and cover it with a sterile dressing. carefully probe the wound to determine if the bleeding is venous or arterial. carefully remove the avulsed flap and wrap it in a moist, sterile trauma dressing.

replace the avulsed flap to its original position and cover it with a sterile dressing.

After assessing a patient's blood glucose level, you accidentally stick yourself with the contaminated lancet. You should: discontinue patient care and seek medical attention. get immunized against hepatitis as soon as possible. immerse your wound in an alcohol-based solution. report the incident to your supervisor after the call.

report the incident to your supervisor after the call.

Upon arriving at the scene of a law enforcement tactical situation, you should ensure your own safety and then: begin immediate triage of any injured personnel. report to the incident commander for instructions. apprise medical control of the tactical situation. locate all injured personnel and begin treatment.

report to the incident commander for instructions.

You and your partner respond to the scene of a 49-year-old male with acute abdominal pain. As you enter his residence, you find him lying on the floor in severe pain. He is conscious and alert. The patient appears to weigh in excess of 350 pounds. Your first action should be to: request the fire department if one attempt to move him fails. request additional personnel before making any attempts to lift him. assess him and then move him to the stretcher with a direct carry. encourage the patient to walk himself to the awaiting ambulance.

request additional personnel before making any attempts to lift him.

You have administered one dose of epinephrine to a 40-year-old female to treat an allergic reaction that she developed after being stung by a scorpion. Your reassessment reveals that she is still having difficulty breathing, has a decreasing mental status, and has a blood pressure of 80/50 mm Hg. You should: monitor her en route to the hospital and call medical control if she worsens. crush up an antihistamine tablet and place it in between her cheek and gum. request permission from medical control to give another dose of epinephrine. administer a nebulized bronchodilator to improve the status of her breathing.

request permission from medical control to give another dose of epinephrine.

A Level 4 hazardous material: is mildly toxic but still requires the use of a self-contained breathing apparatus. requires specialized gear designed for protection against that particular hazard. requires protective gear to ensure that no part of the skin's surface is exposed. causes temporary damage or injury unless prompt medical care is provided.

requires specialized gear designed for protection against that particular hazard.

You are assessing a 13-month-old female who is running a fever and has been vomiting. While you are performing your physical examination on this child, you will most likely find that she: has bulging fontanelles secondary to severe dehydration. responds to her name but is fearful of your presence. is unable to track your movements with her eyes. will readily allow you to separate her from her mother.

responds to her name but is fearful of your presence.

A 38-year-old male with a history of schizophrenia is reported by neighbors to be screaming and throwing things in his house. You are familiar with the patient and have cared for him in the past for unrelated problems. Law enforcement officers escort you into the residence when you arrive. The patient tells you that he sees vampires and is attempting to ward them off by screaming and throwing things at them. He has several large lacerations to his forearms that are actively bleeding. The MOST appropriate way to manage this situation is to: approach the patient and calm him by placing your hand on his shoulder. restrain the patient with appropriate force in order to treat his injuries. try to gain the patient's trust by telling him that you see the vampires too. request that the police officers arrest him and take him to the hospital.

restrain the patient with appropriate force in order to treat his injuries.

A 75-year-old male with a terminal illness has died at home. As you and your partner enter the residence, a family member becomes verbally abusive, pushes you, and states that you took too long to get there. You should: tell the family member that it is not your fault. ignore the family member and assess the patient. retreat and notify law enforcement personnel. subdue the family member until the police arrive.

retreat and notify law enforcement personnel.

A patient who cannot remember the events that preceded his or her head injury is experiencing: anterograde amnesia. prograde amnesia. posttraumatic amnesia. retrograde amnesia.

retrograde amnesia.

All of the following biologic agents or diseases can be transmitted from person to person, except: ricin. pneumonic plague. smallpox. viral hemorrhagic fevers.

ricin.

Deoxygenated blood from the body returns to the: right atrium. left ventricle. left atrium. right ventricle.

right atrium.

The part of the brain that controls the left side of the body is the: left parietal lobe. right temporal lobe. right-side cerebrum. left-side cerebrum.

right-side cerebrum.

When assisting an ALS worker, a BLS worker who performs a skill outside his or her level of certification: risks a lawsuit. needs incident command authorization. is striving for promotion to ALS. plays a team role.

risks a lawsuit.

Common complications associated with central venous catheters include all of the following, except: bleeding around the line. a local infection. rupture of a central vein. clotting of the line.

rupture of a central vein.

All of the following are vesicant agents, except: phosgene oxime. lewisite. sarin. sulfur mustard.

sarin.

Functions of the liver include: secretion of bile and filtration of toxic substances. absorption of nutrients and toxins. release of amylase, which breaks down starches into sugar. production of hormones that regulate blood sugar levels.

secretion of bile and filtration of toxic substances.

A 70-year-old female was recently discharged from the hospital following a total hip replacement. Today, she presents with restlessness, tachycardia, and a blood pressure of 90/64 mm Hg. Her skin is hot and moist. You should be MOST suspicious that she is experiencing: septic shock. pump failure. decompensated shock. a local infection.

septic shock.

A pregnant patient is diagnosed with hyperemesis gravidarum. This means that: her unborn baby is very large. her respiratory rate is rapid. she has excessive vomiting. she has frequent urination.

she has excessive vomiting.

During your treatment of a woman in cardiac arrest, you apply the AED, analyze her cardiac rhythm, and receive a "no shock advised" message. This indicates that: the AED detected patient motion. she has a pulse and does not need CPR. the AED has detected asystole. she is not in ventricular fibrillation.

she is not in ventricular fibrillation.

Bruising to the _________ is least suggestive of child abuse. buttocks back shins face

shins

Hypoperfusion is another name for: cyanosis. shock. hypoxemia. cellular death.

shock.

Dyspnea is MOST accurately defined as: a complete cessation of respiratory effort. labored breathing with reduced tidal volume. shortness of breath or difficulty breathing. a marked increase in the exhalation phase.

shortness of breath or difficulty breathing.

The presence of tachycardia following a significant abdominal injury: is always accompanied by hypotension. indicates a state of decompensated shock. is most commonly caused by severe pain. should be assumed to be a sign of shock.

should be assumed to be a sign of shock.

A tube from the brain to the abdomen that drains excessive cerebrospinal fluid is called a: cerebral bypass. CS tube. shunt. G-tube.

shunt.

The use of lights and siren on an ambulance: legally gives the emergency vehicle operator the right of way. signifies a request for other drivers to yield the right of way. allows other drivers to hear and see you from a great distance. is required any time a patient is being transported to the hospital.

signifies a request for other drivers to yield the right of way.

When assessing a patient with a hemothorax, you will most likely find: distant or muffled heart tones. signs and symptoms of shock. ipsilateral tracheal deviation. jugular venous engorgement.

signs and symptoms of shock.

An attack on an abortion clinic would most likely be carried out by a(n): single-issue group. violent religious group. doomsday cult. extremist political group.

single-issue group.

Jugular venous distention suggests a problem with blood returning to the heart if the patient is: in a prone position. in a supine position. sitting up at a 45° angle. in a recumbent position.

sitting up at a 45° angle.

While providing care to a patient, the EMT informs her partner that a shotgun is leaning against the wall in the corner of the room. In making this observation, the EMT has demonstrated: constructive intervention. closed-loop communication. situational awareness. crew resource management.

situational awareness.

Early signs and symptoms of smallpox include all of the following, except: headaches. body aches. skin blisters. high fever.

skin blisters.

You are dispatched to a call for a 4-month-old infant with respiratory distress. While you prepare to take care of this child, you must remember that: assisted ventilations in infants often need to be forceful to inflate their lungs. an infant's head should be placed in a flexed position to prevent obstruction. the infant's proportionately small tongue often causes an airway obstruction. small infants are nose breathers and require clear nasal passages at all times.

small infants are nose breathers and require clear nasal passages at all times.

The primary organ responsible for absorption of products of digestion is the: small intestine. large intestine. gallbladder. pancreas.

small intestine.

A 52-year-old male presents with a fever of 102.5°F and a severe headache. As you assess him, you note the presence of multiple blisters on his face and chest, which are all identical in shape and size. This patient's clinical presentation is most consistent with: sarin toxicity. smallpox. cutaneous anthrax. yellow fever virus.

smallpox.

Common associated conditions in patients with spina bifida include all of the following, except: loss of bladder control. spastic limb movement. hydrocephalus. extreme latex allergy.

spastic limb movement.

Significant trauma to the face should increase the EMT's index of suspicion for a(n): airway obstruction. displaced mandible. basilar skull fracture. spinal column injury.

spinal column injury.

Internal bleeding into a fractured extremity is most often controlled by: applying a tourniquet. splinting the extremity. keeping the patient warm. applying chemical ice pack.

splinting the extremity.

A 22-year-old female was ejected from her car after striking a tree head-on. As you approach her, you note obvious closed deformities to both of her femurs. She is not moving and does not appear to be conscious. You should: assess for a carotid pulse and assist her ventilations. stabilize her head and perform a primary assessment. apply manual stabilization to both of her femurs. administer oxygen and perform a rapid assessment.

stabilize her head and perform a primary assessment.

During your secondary assessment of a 30-year-old male who fell 25 feet, you note crepitus when palpating his pelvis. Your partner advises you that the patient's blood pressure is 80/50 mm Hg, and his heart rate is 120 beats/min and weak. After completing your assessment, you should: defer spinal immobilization and transport to a trauma center. stabilize the pelvis with a pelvic binder and protect the spine. perform a focused physical exam with emphasis on the pelvis. log roll the patient onto a long backboard and transport at once.

stabilize the pelvis with a pelvic binder and protect the spine.

The __________ area is where incoming ambulances meet and await further instructions at the scene of a multiple-casualty incident. triage staging transportation support

staging

A critical function of the safety officer is to: brief responders during the demobilization phase of an incident. determine the most efficient approach to extricate a victim. monitor emergency responders for signs of stress and anxiety. stop an emergency operation whenever a rescuer is in danger.

stop an emergency operation whenever a rescuer is in danger.

A 67-year-old female with severe chest pain becomes unresponsive, pulseless, and apneic during transport. You should: alert the receiving hospital and perform CPR for the duration of the transport. perform CPR for 1 to 2 minutes and then analyze her rhythm with an AED. stop the ambulance, begin CPR, and attach the AED as soon as possible. defibrillate with the AED while continuing transport to the hospital.

stop the ambulance, begin CPR, and attach the AED as soon as possible.

As you and your partner are carrying a stable patient down a flight of stairs in a stair chair, you feel a sudden, sharp pain in your lower back. You should: guide your partner while moving the chair backwards. stop the move and request additional lifting assistance. stop the move and have the patient walk down the stairs. reposition your hands and continue to move the patient.

stop the move and request additional lifting assistance.

A high-pitched inspiratory sound that indicates a partial upper airway obstruction is called: wheezing. rhonchi. stridor. grunting.

stridor.

You are caring for a 70-year-old female with signs and symptoms of an acute stroke. She is conscious, has secretions in her mouth, is breathing at a normal rate with adequate depth, and has an oxygen saturation of 96%. You should: assist her ventilations with a bag-valve mask. insert an oral airway, apply oxygen, and transport. suction her oropharynx and transport immediately. administer one tube of oral glucose and transport.

suction her oropharynx and transport immediately.

A 30-year-old female was robbed and assaulted as she was leaving a nightclub. She has massive facial trauma and slow, gurgling respirations. As your partner manually stabilizes her head, you should: begin immediate ventilatory assistance. visualize her mouth for obvious wounds. apply oxygen via a nonrebreathing mask. suction her oropharynx.

suction her oropharynx.

During your primary assessment of a 19-year-old unconscious male who experienced severe head trauma, you note that his respirations are rapid, irregular, and shallow. He has bloody secretions draining from his mouth and nose. You should: pack his nostrils to stop the drainage of blood. suction his oropharynx for up to 15 seconds. immobilize his spine and transport immediately. assist his ventilations with a bag valve mask.

suction his oropharynx for up to 15 seconds.

Proper technique for suctioning the oropharynx of an adult patient includes: suctioning for up to 1 minute if the patient is well oxygenated. removing large, solid objects with a tonsil-tip suction catheter. suctioning while withdrawing the catheter from the oropharynx. continuously suctioning patients with copious oral secretions.

suctioning while withdrawing the catheter from the oropharynx.

Burns associated with lightning strikes are typically: third-degree. partial-thickness. superficial. full-thickness.

superficial.

A 31-year-old male was bitten on the leg by an unidentified snake. The patient is conscious and alert and in no apparent distress. Your assessment of his leg reveals two small puncture marks with minimal pain and swelling. In addition to administering oxygen and providing reassurance, further care for this patient should include: supine positioning, splinting the leg, and transporting. transporting only with close, continuous monitoring. elevating the lower extremities and giving antivenin. applying ice to the wound and transporting quickly.

supine positioning, splinting the leg, and transporting.

As you and your team are removing an unresponsive patient from her wrecked car, you note that she has deformities to both of her legs and a deformity to her left humerus. You should: support the injured extremities and continue removal. splint the deformities before moving her any further. realign the deformed extremities before continuing. assess distal neurovascular functions in her extremities.

support the injured extremities and continue removal.

A 42-year-old male is found unresponsive on his couch by a neighbor. During your assessment, you find no signs of trauma, and the patient's blood glucose level is 75 mg/dL. His blood pressure is 168/98 mm Hg, his heart rate is 45 beats/min and bounding, and his respirations are 8 breaths/min and irregular. The patient is wearing a medical alert bracelet that states he has hemophilia. You should: suspect that he has intracranial bleeding, assist his ventilations, and transport rapidly to an appropriate hospital. administer oxygen via a nonrebreathing mask, apply oral glucose in between his cheek and gum, and transport. suspect that he has internal bleeding and is in shock, administer high-flow oxygen, and transport at once. administer high-flow oxygen, perform a detailed secondary assessment at the scene, and transport promptly.

suspect that he has intracranial bleeding, assist his ventilations, and transport rapidly to an appropriate hospital.

When caring for a patient who is visually impaired, it is important to: stand to the side of the patient when speaking if his or her peripheral vision is impaired. tell him or her what is happening, identify noises, and describe the situation and surroundings. leave items such as canes and walkers at the residence if the patient will be carried on a gurney. allow a service dog to remain with the patient at all times, even if the patient is critically ill.

tell him or her what is happening, identify noises, and describe the situation and surroundings.

Situational awareness is most accurately defined as: predicting the presence of certain hazards at the scene after receiving initial information from the dispatcher. performing an initial scan of the scene to identify hazards that will pose an immediate threat to you and your crew. an ongoing process of information gathering and scene evaluation to determine appropriate strategies and tactics. the ability to recognize any possible issues once you arrive at the scene and act proactively to avoid a negative impact.

the ability to recognize any possible issues once you arrive at the scene and act proactively to avoid a negative impact.

A unique consideration when dealing with a hybrid vehicle is that: you must locate the ignition switch and cut it to prevent a fire. the battery has higher voltage than a traditional vehicle battery. cutting the battery cables often results in an explosion or fire. rescue teams should disconnect the positive battery cable first.

the battery has higher voltage than a traditional vehicle battery.

With increasing age, the heart must work harder to move the blood effectively because: the arteries dilate significantly. diastolic blood pressure decreases. the blood vessels become stiff. the blood thickens as a person ages.

the blood vessels become stiff.

In contrast to younger patients, older patients are more prone to a decrease in blood pressure (BP) upon standing because: their red blood cells are destroyed at a faster than normal rate. the body is less able to adapt the BP to rapid postural changes. the aging process results in an overall increase in blood volume. any change in position causes blood to be shunted to the brain.

the body is less able to adapt the BP to rapid postural changes.

As time progresses following a significant injury: the patient's injuries will most likely be irreparable. the patient's blood pressure elevates significantly. the body's ability to compensate for shock decreases. most patients will die secondary to internal bleeding.

the body's ability to compensate for shock decreases.

Heatstroke occurs when: a person becomes dehydrated secondary to excess water loss. the body's heat-eliminating mechanisms are overwhelmed. the ambient temperature exceeds 90°F (32°C) and the humidity is high. a person's core body temperature rises above 103°F (39°C).

the body's heat-eliminating mechanisms are overwhelmed.

The concept of consistent care across the entire health care team from first patient contact to patient discharge is called: the scope of practice. patient care advocacy. the standard of care. the continuum of care.

the continuum of care.

An air embolism associated with diving occurs when: the diver hyperventilates prior to entering the water. the alveoli completely collapse due to high pressure. the diver holds his or her breath during a rapid ascent. high water pressure forces air into the mediastinum.

the diver holds his or her breath during a rapid ascent.

The main benefit of using a mechanical piston device for chest compressions is: the elimination of rescuer fatigue that results from manual compressions. its ability to be used with any patient, regardless of age, weight, or body size. the elimination of the need to place a firm, flat device under the patient. the minimal training required to correctly operate the device.

the elimination of rescuer fatigue that results from manual compressions.

Structures of the lower airway include all of the following, EXCEPT: alveoli. bronchioles. the epiglottis. the trachea.

the epiglottis.

When determining the exact location and position of the patient(s) in a wrecked vehicle, you and your team should routinely consider all of the following, except: the position of the crashed vehicle. the possibility of vehicle instability. hazards that pose a risk to rescuers. the make and model of the vehicle.

the make and model of the vehicle.

During delivery, it is most important to position your partner at the mother's head because: the mother might become nauseated and vomit. the mother needs to be apprised of the situation. she might need emotional support during the delivery. mothers often need assisted ventilation during delivery.

the mother might become nauseated and vomit.

An infant or small child's airway can be occluded if it is overextended or overflexed because: the tongue is proportionately small and can fall back into the throat. the occiput is proportionately large, and the trachea is flexible. he or she has a long neck, which makes the trachea prone to collapse. the back of the head is flat, which prevents a neutral position.

the occiput is proportionately large, and the trachea is flexible.

If a dislocated shoulder has spontaneously reduced before your arrival, the only way to confirm the injury is by noting: distal circulation. the patient history. the presence of deformity. bruising to the shoulder.

the patient history.

A 77-year-old female presents with an acute onset of altered mental status. Her son is present and advises that she has a history of hypertension, atrial fibrillation, type 2 diabetes, and glaucoma. He further advises that she takes numerous medications and is normally alert. When you assess this patient, it is important to note that: the patient is experiencing delirious behavior, which suggests a new health problem. because of her age and medical history, you should suspect Alzheimer disease. dementia typically presents as an acute onset of deterioration of cognitive function. her mental status is likely the result of hypoglycemia and you should give her sugar.

the patient is experiencing delirious behavior, which suggests a new health problem.

EMTs receive a call for a possible sexual assault. The patient is a young female who is conscious and alert and has no apparent injuries. She states, "I can't remember anything, but I know I was raped." The EMTs should suspect that: the traumatic experience has created a mental block. the patient was given a drug prior to the incident. the patient knew her attacker, but is afraid to say. an underlying head injury is causing her amnesia.

the patient was given a drug prior to the incident.

Abruptio placenta occurs when: the placenta prematurely separates from the uterine wall. a tear in the placenta causes severe internal hemorrhage. the placenta affixes itself to the outer layer of the uterus. the placenta develops over and covers the cervical opening.

the placenta prematurely separates from the uterine wall.

After delivery of the placenta, the mother is experiencing vaginal bleeding. After massaging the uterine fundus and allowing the mother to breastfeed, the bleeding stops. This occurred because: breastfeeding causes uterine blood vessels to dilate. a portion of the placenta was retained in the uterus. uterine massage increases blood flow to the uterus. the production of oxytocin caused uterine contraction.

the production of oxytocin caused uterine contraction.

The term "pericardiocentesis" means: a surgical opening made in the heart. the removal of fluid from around the heart. narrowing of the arteries supplying the heart. surgical repair of the sac around the heart.

the removal of fluid from around the heart.

The left cerebral hemisphere controls: the right side of the body. breathing and blood pressure. the right side of the face. heart rate and pupil reaction.

the right side of the body.

To protect vital organs, the body compensates by directing blood flow away from organs that are more tolerant of low flow, such as: the lungs. the skin. the brain. the heart.

the skin.

Regardless of the type of tourniquet used, it is important to remember that: the tourniquet should only be removed at the hospital because bleeding might return if the tourniquet is released. bulky dressings should be securely applied over the tourniquet to further assist in controlling the bleeding. you should try to control the bleeding by applying pressure to a proximal arterial pressure point first. the tourniquet should be applied directly over a joint if possible because this provides better bleeding control.

the tourniquet should only be removed at the hospital because bleeding might return if the tourniquet is released.

Cardiac output may decrease if the heart beats too rapidly because: the volume of blood that returns to the heart is not sufficient with fast heart rates. a rapid heartbeat causes a decrease in the strength of cardiac contractions. there is not enough time in between contractions for the heart to refill completely. as the heart rate increases, more blood is pumped from the ventricles than the atria

there is not enough time in between contractions for the heart to refill completely.

Many older victims of physical abuse might make false statements or lie about the origin of their injuries because: they fear retribution from the abuser. most elderly patients have dementia. they do not want to be bothersome. they are protective of the abuser.

they fear retribution from the abuser.

When transporting a patient with a facial injury, it is most important to be as descriptive as possible with the hospital regarding the patient's injuries because: they must make arrangements for an ICU bed. they might need to call a specialist to see the patient. it saves time on repeat assessments at the hospital. most patients with facial trauma will need surgery.

they might need to call a specialist to see the patient.

Most of the serious injuries associated with scuba diving are caused by: too rapid of an ascent. alcohol consumption. cold water temperature. too rapid of a descent.

too rapid of an ascent.

Acute pulmonary edema would MOST likely develop as the result of: severe hyperventilation. an upper airway infection. toxic chemical inhalation. right-sided heart failure.

toxic chemical inhalation.

If you are exposed to a patient's blood or other bodily fluid, your first action should be to: vigorously clean the area with soap and water. report the incident to the infection control officer. transfer care of the patient to another EMS provider. abandon patient care and seek medical attention.

transfer care of the patient to another EMS provider.

A 22-year-old male was kicked in the abdomen several times. You find him lying on his left side with his knees drawn up. He is conscious and alert and complains of increased pain and nausea when he tries to straighten his legs. His blood pressure is 142/82 mm Hg, his pulse rate is 110 beats/min and strong, and his respirations are 22 breaths/min and regular. In addition to administering high-flow oxygen, you should: keep him on his side but gently straighten his legs. transport him in the position in which you found him. place him supine but allow him to keep his knees bent. apply full spinal motion restriction precautions.

transport him in the position in which you found him.

The ____________ supervisor is responsible for notifying area hospitals and determining their availability and capabilities. medical treatment transportation triage

transportation

After assessing your patient, you determine that his condition is stable. You provide the appropriate treatment and then load him into the ambulance. While en route to the hospital, you should: keep your emergency lights on but avoid using the siren. use your lights and siren but drive slowly and defensively. turn your emergency lights off and obey all traffic laws. drive slowly and remain in the far left-hand lane, if possible.

turn your emergency lights off and obey all traffic laws.

A patient who is experiencing aphasia is: unable to produce or understand speech. not able to swallow without choking. usually conscious but has slurred speech. experiencing a right hemispheric stroke.

unable to produce or understand speech.

A teenage boy who was involved in a bicycle accident has a puncture wound where the bicycle kickstand impaled his leg. The most appropriate method for treating this injury is to: unbolt the kickstand from the bike frame and stabilize it with bulky dressings. leave the kickstand attached to the bike until the physician can remove it safely. remove the kickstand in a circular motion and apply a dry, sterile dressing. cut the kickstand off just above the skin, and stabilize it with sterile dressings.

unbolt the kickstand from the bike frame and stabilize it with bulky dressings.

Abdominal thrusts in a conscious child or adult with a severe upper airway obstruction are performed: about 1 inch below the xiphoid process. until he or she loses consciousness. in sets of five followed by reassessment. until he or she experiences cardiac arrest.

until he or she loses consciousness.

To facilitate a safe and coordinated move, the team leader should: speak softly but clearly to avoid startling the patient. use preparatory commands to initiate any moves. be positioned at the feet so the team can hear. never become involved in the move, only direct the move.

use preparatory commands to initiate any moves.

You and your partner are dispatched to a residence for an "ill person." When you arrive, you find that the patient, a 44-year-old man, does not speak English. There are no relatives or bystanders present who can act as an interpreter. You should: use short, simple questions and point to specific parts of your body to try to determine the source of the patient's complaint. give the patient oxygen, assess his vital signs, and transport him to the hospital in a position of comfort. speak to the patient with a moderately louder voice to facilitate his ability to understand what you are saying. refrain from performing any assessment or treatment until you can contact someone who can function as an interpreter.

use short, simple questions and point to specific parts of your body to try to determine the source of the patient's complaint.

A medical transport helicopter is incoming, and you are responsible for setting up the landing zone (LZ). You should: ask bystanders to stand at all four corners of the LZ. use yellow caution tape to mark off the LZ perimeter. use weighted cones to mark all four corners of the LZ. place four flares 100 feet apart in an "X" pattern.

use weighted cones to mark all four corners of the LZ.

Heat loss from the body through respiration occurs when: air temperature is greater than body temperature. warm air is exhaled into the atmosphere. the core body temperature is greater than 98°F (37°C). cool air is inhaled and displaces warm air.

warm air is exhaled into the atmosphere.

The main objective of traffic control at the scene of a motor vehicle crash is to: get oncoming traffic past the scene as soon as possible. warn oncoming traffic and prevent another crash. facilitate a route for the media to access the scene. prevent curious onlookers from observing the scene.

warn oncoming traffic and prevent another crash.

The simplest, yet most effective method of preventing the spread of an infectious disease is to: undergo annual testing for tuberculosis and hepatitis. wash your hands in between patient contacts. ensure that your immunizations are up-to-date. undergo an annual physical examination.

wash your hands in between patient contacts.

If you use a waterless handwashing substitute in the field, it is important to: avoid donning another pair of gloves for at least 10 minutes. wash your hands with soap and water at the hospital. wait at least five minutes before touching another patient. immediately dry your hands with a paper towel.

wash your hands with soap and water at the hospital.

In determining the potential for a terrorist attack, you should routinely observe all of the following on every call, except: weather conditions. the location type. victim's statements. the type of call.

weather conditions.

An infant is premature if it: is born before 38 weeks' gestation or weighs less than 6 pounds. weighs less than 5.5 pounds or is born before 37 weeks' gestation. is born before 40 weeks' gestation or weighs less than 7 pounds. weighs less than 5 pounds or is born before 36 weeks' gestation.

weighs less than 5 pounds or is born before 36 weeks' gestation.

Epinephrine is indicated for patients with an allergic reaction when: wheezing and hypotension are present. the patient is anxious and tachycardic. a paramedic is present at the scene. the reaction produces severe urticaria.

wheezing and hypotension are present.

Critical incident stress management (CISM) can occur at an ongoing scene in all of the following circumstances, except: when patients are actively being assessed or treated. before personnel are preparing to reenter the scene. when personnel are assessed during periods of rest. before leaving the scene after the incident is resolved.

when patients are actively being assessed or treated.

As soon as you leave the hospital and are en route back to your station, you should inform the dispatcher: that you are prepared for another call. of the name of the accepting physician. about the patient's clinical condition. whether you are back in service.

whether you are back in service.

Distributive shock occurs when: temporary but severe vasodilation causes a decrease in blood supply to the brain. severe bleeding causes tachycardia in order to distribute blood to the organs faster. widespread dilation of the blood vessels causes blood to pool in the vascular beds. an injury causes restriction of the heart muscle and impairs its pumping function.

widespread dilation of the blood vessels causes blood to pool in the vascular beds.

A nasopharyngeal airway is inserted: into the larger nostril with the tip pointing away from the septum. with the bevel pointing downward if inserted into the left nare. into the smaller nostril with the tip following the roof of the nose. with the bevel facing the septum if inserted into the right nare.

with the bevel facing the septum if inserted into the right nare.

A positive TB skin test indicates that: you are actively infected with TB. you have never been exposed to TB. the TB disease is currently dormant but might later become active. you have been exposed to TB.

you have been exposed to TB.

An index of suspicion is MOST accurately defined as: determining the underlying cause of a patient's medical condition based on signs and symptoms. the EMT's prediction of the type of illness a patient has based on how the call is dispatched. ruling out specific medical conditions based on the absence of certain signs and symptoms. your awareness and concern for potentially serious underlying and unseen injuries or illness.

your awareness and concern for potentially serious underlying and unseen injuries or illness.


Conjuntos de estudio relacionados

Equity in the distribution of income

View Set

4/18 MCQ Corrections- By Jonathan Escobar

View Set

PSY 1001: Intro Psychology Module 13 Quiz

View Set

Chapter 10: Post-traumatic Stress Disorder

View Set

Test Chapter 7:9 Lymphatic System

View Set

Module 80 - Altruism, Conflict, and Peacemaking

View Set

2602: Law/Licensees & Regulation

View Set

ECON MICRO MARKET FAILURES (CH 5)

View Set

AST 20 Advanced Engine Diagnosis

View Set